Monenlaista matematiikkaa?

Jotkut filosofisesti orientoituneet ovat sitä mieltä., että on monenlaista matematiikkaa sen mukaan, miten premissit valitaan, vaikka heillä ei ole yhtään esimerkkiä kertoa, miten kahdella validilla eri matematiikalla saavutettaisiin kaksi eri tulosta.

Todellisuudessa on yksi ja vain yksi matematiikka, joka antaa kaikissa tapauksissa saman tuloksen.

Opintojeni alkuvaiheessa eräs oli jopa sitä mieltä, että jos kantaluvuksi valitaan joku muu kuin 10, esimerkiksi 7, saadaan aivan erilaista matematiikkaa.

Tosin hän ei osannut sanoa tätä selkeästi, mutta hän oli aivan vakuuttunut, että 7-järjestelmässä esimerkiksi kymmenjärjestelmän 5 6 = 11 olisi todella 5 6 = 14, ikään kuin lukisimme taas 10-järjestelmää. Ja että todellisuus olisi eri sen mukaan, mitä "matematiikkaa" käytettiin.

Tämä on vain yksinkertainen esimerkki, mutta juuri tämänkaltaisiin sudenkuoppiin filosofisesti orientoituvat putoavat. Todellisuudesta matematiikka on aina yhtä ja samaa, muuttumatonta totuutta, jota kyllä voidaan kirjoittaa eri tavalla, mutta sisältö ei muutu.

Tähän perustuu myös erehdys, että matematiikka olisi riippuvainen materiasta ja sen prosesseista. Ei ole. Se on täysin riippumaton mahdollisesta aineen olemassaolosta ja sen laadusta, sekä ihmisen tekemistä "valinnoista".

Matemaattista tutkimusta on eri puolilla maailma. Sen virheettömyyden AINOA kriteeri on, että se on yhteneväinen ajattoman ja muuttumattoman matematiikan kanssa.

Siksi matemaattinen tutkimus tuottaa täysin yhteneviä tuloksia kaikkialla, vaikka missään ei suoriteta empiiristä tutkimusta. Jos ilmenee poikkeama, on varmasti virhe. Se etsitään ja poistetaan.

Matemaattisen totuuden totuusarvo (odotusarvo) on täydet 1.

x x x

Empiirinen tutkimus suoriteaan aina pienemmällä odotusarvolla kuin 1, siis <1. Käytetyistä menetelmistä voidaan päätellä, kuinka lähellä 1:tä ollaan. Yleensä kuitenkin hyvin lähellä.

Koska empiirisen tutkimuksen tulokset muodostuvat erehtymättömästä matematiikka (odotusarvo 1) ja empiirisestä havainosta (odotusarvo (<1, mutta lähellä 1:tä), loogis-empiirinen tieto on parasta ja varminta saatavilla olevaa tietoa.

Minkään suuntaisella tai tasoja erottelevalla filosofialla, kuten "tieteenfilosofialla" tai "ontologialla"ei ole mitään kosketuspintaa tieteen kanssa, kuten ei myöskään uskonnolla tai common sensellä.

62

<50

    Vastaukset

    Anonyymi (Kirjaudu / Rekisteröidy)
    5000
    • Olen kanssasi samaa mieltä: On vain yhdenlaista matematiikkaa. Mutta filosofit huomauttavat, että matematiikassakin lähdetään liikkeelle perustotuuksista eli aksioomista, joiden katsotaan olevan tosia ilman, että niitä on todistettu.

      Nämä matemaattiset perustotuudet ovat kuitenkin sellaisia, että käytännössä jokainen hyväksyy ne intuitiivisesti. Esimerkki matemaattisesta aksioomasta: ”A = A” eli jokainen luku on yhtäsuuri itsensä kanssa.

      • Tarkoitukseni oli tuoda esiin matematiikan status, eli mitä se on ja mitä ei. Todet asiat ovat tosia ja epätodet epätosia, pystymmepä niitä todistamaan tai emme. Matematiikassa on myös tosia väittämiä, joita ei voi todistaa.

        Kun toteamme, että keppi on pitempi kuin toinen, ei mieleemme nouse, että tämä pitemmyys olisi jotenkin olemassa. mutta heti kun aivomme luovat monimutkaisemman konstruktion, rupeamme ajattelemaan, että se on jollakin tavalla olemassa. Olemassaolo on vain aivojemme harhakuvitelmaa. Ei matematiikka, eikä mikään osa siitä, ole millään tavalla olemassa.

        Filosofia menee pysyvästi solmuun, kun arkikielen sanoja huomaamattomasti käytetään tarkoittamassa tyystin eri asioita. Kun esimerkiksi puhumme aineellisesta oliosta, sanomme, että se "on olemassa". Kun taas käytämme aivan samaa ilmaisua logiikassa, "on olemassa", tarkoitamme, ettei se ole loogisen totuuden vastainen, siis voimme kuvitella sen.

        Sen sijaan emme voi kuvitella "niiden joukkojen joukkoa, jotka eivät ole itsensä alkiona". Sellaista kun ei todellakaan voi olla olemassa.

        Filosofi rakentaa huomaamattaan löysän sanankäytön varaan kuvitelmia, joissa lainautuu vapaasti merkityksiä täysin keskenään yhteenkuulumattomista asioista. Kun tälläinen tura on syntynyt arkikielen ja epätarkan ajattelun pohjalta, sitä ei enää selvitä edes nainen Gruusiasta.

        Matemaattinen tutkimus on olemassa, se on aivoissa, sinikantisissa vihoissa ja yliopistoissa sun muissa, mutta itse matematiikka on vain loogisesti paikkansa pitävää totuutta. Se ei sijaitse missään, mutta pitää paikkansa.


      • Firdawsi kirjoitti:

        Tarkoitukseni oli tuoda esiin matematiikan status, eli mitä se on ja mitä ei. Todet asiat ovat tosia ja epätodet epätosia, pystymmepä niitä todistamaan tai emme. Matematiikassa on myös tosia väittämiä, joita ei voi todistaa.

        Kun toteamme, että keppi on pitempi kuin toinen, ei mieleemme nouse, että tämä pitemmyys olisi jotenkin olemassa. mutta heti kun aivomme luovat monimutkaisemman konstruktion, rupeamme ajattelemaan, että se on jollakin tavalla olemassa. Olemassaolo on vain aivojemme harhakuvitelmaa. Ei matematiikka, eikä mikään osa siitä, ole millään tavalla olemassa.

        Filosofia menee pysyvästi solmuun, kun arkikielen sanoja huomaamattomasti käytetään tarkoittamassa tyystin eri asioita. Kun esimerkiksi puhumme aineellisesta oliosta, sanomme, että se "on olemassa". Kun taas käytämme aivan samaa ilmaisua logiikassa, "on olemassa", tarkoitamme, ettei se ole loogisen totuuden vastainen, siis voimme kuvitella sen.

        Sen sijaan emme voi kuvitella "niiden joukkojen joukkoa, jotka eivät ole itsensä alkiona". Sellaista kun ei todellakaan voi olla olemassa.

        Filosofi rakentaa huomaamattaan löysän sanankäytön varaan kuvitelmia, joissa lainautuu vapaasti merkityksiä täysin keskenään yhteenkuulumattomista asioista. Kun tälläinen tura on syntynyt arkikielen ja epätarkan ajattelun pohjalta, sitä ei enää selvitä edes nainen Gruusiasta.

        Matemaattinen tutkimus on olemassa, se on aivoissa, sinikantisissa vihoissa ja yliopistoissa sun muissa, mutta itse matematiikka on vain loogisesti paikkansa pitävää totuutta. Se ei sijaitse missään, mutta pitää paikkansa.

        ”Todet asiat ovat tosia ja epätodet epätosia, pystymmepä niitä todistamaan tai emme.”

        Tuo ei kuitenkaan auta yhtään, kun yrität selvittää, onko jokin väittämä tosi. Yleensä tuon argumentin kuulee, kun joku uskoo perusteettomaan asiaan eikä kykene perustelemaan harhauskomustansa.

        ”Sen sijaan emme voi kuvitella "niiden joukkojen joukkoa, jotka eivät ole itsensä alkiona". Sellaista kun ei todellakaan voi olla olemassa.”

        Oleppas tarkkana. Se, ettemme kykene kuvittelemaan jotakin, ei tarkoita, että asia olisi epätosi. Kyseessä on argumentti tietämättömyydestä.

        Matemaattiset aksioomat ovat premissejä ja periaatteessa todistus kaatuu siihen, jos joku kyseenalaistaa aksioomat. Mutta en ole vielä tavannut henkilöä, joka näin tekisi minkään matemaattisen aksiooman osalta. Samoin matemaattisista perustotuuksista voimme johtaa asioita, joiden toimivuus voidaan todeta myös käytännössä, mikä on vahva todistus aksioomien todenpitävyydestä.


      • Järkisyitä kirjoitti:

        ”Todet asiat ovat tosia ja epätodet epätosia, pystymmepä niitä todistamaan tai emme.”

        Tuo ei kuitenkaan auta yhtään, kun yrität selvittää, onko jokin väittämä tosi. Yleensä tuon argumentin kuulee, kun joku uskoo perusteettomaan asiaan eikä kykene perustelemaan harhauskomustansa.

        ”Sen sijaan emme voi kuvitella "niiden joukkojen joukkoa, jotka eivät ole itsensä alkiona". Sellaista kun ei todellakaan voi olla olemassa.”

        Oleppas tarkkana. Se, ettemme kykene kuvittelemaan jotakin, ei tarkoita, että asia olisi epätosi. Kyseessä on argumentti tietämättömyydestä.

        Matemaattiset aksioomat ovat premissejä ja periaatteessa todistus kaatuu siihen, jos joku kyseenalaistaa aksioomat. Mutta en ole vielä tavannut henkilöä, joka näin tekisi minkään matemaattisen aksiooman osalta. Samoin matemaattisista perustotuuksista voimme johtaa asioita, joiden toimivuus voidaan todeta myös käytännössä, mikä on vahva todistus aksioomien todenpitävyydestä.

        Firdawsi:”Todet asiat ovat tosia ja epätodet epätosia, pystymmepä niitä todistamaan tai emme.”

        Järkisyitä:"Tuo ei kuitenkaan auta yhtään, kun yrität selvittää, onko jokin väittämä tosi."

        En ole kertomassa, miten tosi väite on erotettavissa epätodesta, vaan kysymyksestä, MIKÄ ON MATEMATIIKAN STATUS. Esitin lauseeni siksi, että tarjoamasi todistamiskysymys ei ole statuksen kannalta relevantti.

        Epälooginen ei ole argumentti "tietämättömyydestä" vaan mahdottomuudesta. Mainitsemani joukko oli esimerkki epäloogisen, siis mahdottoman, joukon muodostamisesta.

        Näyttää olevan vaikea keskittyä matematiikan stuskysymykseen, siis onko matematiikka olemassa vai ei. Sen sijaan tarjoat kysymyksiä todistettavuudesta tai sen puutteesta. Näistä en ole sanonut mitään, enkä sano.

        "Oleppas tarkkana." Sopimatonta kieltä. Hylättävää.


      • Firdawsi kirjoitti:

        Firdawsi:”Todet asiat ovat tosia ja epätodet epätosia, pystymmepä niitä todistamaan tai emme.”

        Järkisyitä:"Tuo ei kuitenkaan auta yhtään, kun yrität selvittää, onko jokin väittämä tosi."

        En ole kertomassa, miten tosi väite on erotettavissa epätodesta, vaan kysymyksestä, MIKÄ ON MATEMATIIKAN STATUS. Esitin lauseeni siksi, että tarjoamasi todistamiskysymys ei ole statuksen kannalta relevantti.

        Epälooginen ei ole argumentti "tietämättömyydestä" vaan mahdottomuudesta. Mainitsemani joukko oli esimerkki epäloogisen, siis mahdottoman, joukon muodostamisesta.

        Näyttää olevan vaikea keskittyä matematiikan stuskysymykseen, siis onko matematiikka olemassa vai ei. Sen sijaan tarjoat kysymyksiä todistettavuudesta tai sen puutteesta. Näistä en ole sanonut mitään, enkä sano.

        "Oleppas tarkkana." Sopimatonta kieltä. Hylättävää.

        Matematiikka on eksaktia laskentaa, mikä lähtee liikkeelle pienestä joukosta aksioomia eli premissejä. Näiden pohjalta johdetaan kaikki muu käyttäen eksakteja määritelmiä. Tässä suhteessa matematiikka muistuttaa mitä tahansa eksaktia filosofista päättelyä.

        Matematiikka ei ole vain yksi asia. Esimerkiksi geometrian aksioomilla ei ole mitään tekemistä reaalilukujen aksioomien kanssa. Imaginääriluvut ovat realilukujen laajennus ja joku voi perustellusti olla eri mieltä siitä, onko reaalilukujen laajennus imaginääri-akselille lainkaan perusteltua. Imaginäärilukuja voi perustellusti pitää täysin hatusta vedettynä asiana, mille ei ole mitään vastinetta reaalimaailmassa. Ne ovat vain meidän mielikuvituksen tuote - keksintö.

        Jos olemassa olemisella tarkoitetaan vain fyysisiä asioita, niin silloin abstraktit asiat kuten matematiikka ei ole olemassa ellei jokin tietoinen olento ajattele niitä, jolloin kyseiset käsitteet ovat olemassa aivojen tilana tai prosessina.


      • Anonyymi
        Järkisyitä kirjoitti:

        Matematiikka on eksaktia laskentaa, mikä lähtee liikkeelle pienestä joukosta aksioomia eli premissejä. Näiden pohjalta johdetaan kaikki muu käyttäen eksakteja määritelmiä. Tässä suhteessa matematiikka muistuttaa mitä tahansa eksaktia filosofista päättelyä.

        Matematiikka ei ole vain yksi asia. Esimerkiksi geometrian aksioomilla ei ole mitään tekemistä reaalilukujen aksioomien kanssa. Imaginääriluvut ovat realilukujen laajennus ja joku voi perustellusti olla eri mieltä siitä, onko reaalilukujen laajennus imaginääri-akselille lainkaan perusteltua. Imaginäärilukuja voi perustellusti pitää täysin hatusta vedettynä asiana, mille ei ole mitään vastinetta reaalimaailmassa. Ne ovat vain meidän mielikuvituksen tuote - keksintö.

        Jos olemassa olemisella tarkoitetaan vain fyysisiä asioita, niin silloin abstraktit asiat kuten matematiikka ei ole olemassa ellei jokin tietoinen olento ajattele niitä, jolloin kyseiset käsitteet ovat olemassa aivojen tilana tai prosessina.

        Jousi värähtelee lineaarisesti imaginäärisessä koordinaatistossa - esim auton jouset. Imaginääri luvuilla on paljon todellisia fysikaalisia käyttötarkoituksia - elektroniikkaa ei voida edes suunnitella/laskea ilman imaginäärilukuja ja siinä sinä kirjoitat koneella jota ei voida valmistaa käyttämättä imaginäärilukuja...


      • Anonyymi
        Järkisyitä kirjoitti:

        Matematiikka on eksaktia laskentaa, mikä lähtee liikkeelle pienestä joukosta aksioomia eli premissejä. Näiden pohjalta johdetaan kaikki muu käyttäen eksakteja määritelmiä. Tässä suhteessa matematiikka muistuttaa mitä tahansa eksaktia filosofista päättelyä.

        Matematiikka ei ole vain yksi asia. Esimerkiksi geometrian aksioomilla ei ole mitään tekemistä reaalilukujen aksioomien kanssa. Imaginääriluvut ovat realilukujen laajennus ja joku voi perustellusti olla eri mieltä siitä, onko reaalilukujen laajennus imaginääri-akselille lainkaan perusteltua. Imaginäärilukuja voi perustellusti pitää täysin hatusta vedettynä asiana, mille ei ole mitään vastinetta reaalimaailmassa. Ne ovat vain meidän mielikuvituksen tuote - keksintö.

        Jos olemassa olemisella tarkoitetaan vain fyysisiä asioita, niin silloin abstraktit asiat kuten matematiikka ei ole olemassa ellei jokin tietoinen olento ajattele niitä, jolloin kyseiset käsitteet ovat olemassa aivojen tilana tai prosessina.

        Nimimerkki: Jumalan-kumous (ei-rekisteröitynyt professori)

        Järkisyitä: "Jos olemassa olemisella tarkoitetaan vain fyysisiä asioita, niin silloin abstraktit asiat kuten matematiikka ei ole olemassa ellei jokin tietoinen olento ajattele niitä, jolloin kyseiset käsitteet ovat olemassa aivojen tilana tai prosessina."

        Eräs relevantti seikka tähän "todellisen tai mielekkään olemassaolon laajaan teemaan" liittyen on se, että mitkä ovat käsitteiden ja mm. siis tietysti matemaattisten abstraktioiden määrittelyn *minimiedellytykset*. Ensinnäkin, käsitteet; kuten esim. matematiikka, totuus, kuvaus ja käsite, voidaan määritellä vain siten, kuin miten todellisuus *pakottaa* ne määrittelemään; liittyen aiemmin mainitsemaani Wigner:in dilemmaan.

        Tarkemmin kuvaten tosin, todellisuus ei kuitenkaan havaintojen mukaan pakota määrittelemään ainakaan kaikkia niitä s.e. kaikki niiden määrittelyyn kykenevät osajärjestelmät määrittelisivät ne samalla tavalla tai edes välttämättä ajan funktiona vakioisella tavallakaan, mikä on nähtävillä mm. tälläkin palstalla; etenkin jos sen sisältöä tarkastelee kokonaisuutena, eikä vain tietyllä ajanhetkellä.

        Oleellisempaa tässä tietysti kuitenkin on se, että määritelmien tuottaminen ei käytännössä ole ylipäätään mahdollista, jos ne tuottava osajärjestelmä ei ole vuorovaikutuksessa kausaalisia vaikutuksia omaavan todellisuuden kanssa ja siis myös ole osa tuota järjestelmää. Lisäksi, käytännön kannalta optimaaliset määritelmät ilmeisesti ovat sellaisia, että niillä on ainakin välillisesti käyttökelpoinen yhteys *havaintojen* tulkintaan; eli että nuo määritelmät mahdollistavat; omalta osaltaan, ainakin välillisesti jonkinlaista "koettua lisäarvoa"; joidenkin tulkitsijoiden toimesta, kuten esim. sellaisten kuvausten osalta, jotka ovat omalta osaltaan käyttökelpoisia fyysisten ilmiöiden käyttäytymisen ennustamisessa; osana reaalista laskentaa.

        Kaikenkaikkiaan, kuitenkin, koska todellisuuden kuvaukset; jotka siis ovat; aiemmin kuvatulla tavalla, aina abstraktioita; mikä on sellainen seikka, joka ansaitsisi *paljon* laajemmankin käsittelyn; ja myös meta-tason käsittelyn, merkityksensä vuoksi, niin ne tosiaan ovat *tietyssä merkityksessä* vähemmän todellisia kuin se, että mitä ne kuvaavat, mutta jos ne havaitaan tai niillä on vaikutusta havaitsemiseen, niin ne ovat *silti* todellisia ainakin subjektiivisessa, kokemuksellisessa merkityksessä; eli ymmärrämme esim. sen, että mitä totuus kvaliteettina tarkoittaa; ainakin jossain määrin.

        Kyse lienee tältä osin siitä, että mitä painotetaan, eli painotetaanko kuvaukset aikaansaavia reaalisia vaikutuksia; jotka ovat tietysti omalta osaltaan syy siihen, että noita kuvauksia ylipäätään on olemassa, vai kuvauksia itseään, jotka taas ovat potentiaalisesti siinä merkityksessä hyvinkin relevantteja, että "jäsentämätön todellisuus" ei ole käyttökelpoinen. Tai tarkemmin ilmaisten, todellisuuden hyödyntäminen edellyttää ainakin mm. jonkinlaista "ohjaavaa järjestystä", vaikka tuota järjestystä ei eksplikoitaisikaan, vaan "annettaisiin" sen vain vaikuttaa omalta osaltaan siihen, että järjestelmän vuorovaikutus ympäristönsä kanssa mm. ylipäätään mahdollistaa tuon järjestelmän olemassaolon…

        Esim. Carlos Castenadan (CC); varsin "epäpopulistinen", näkökulma todellisuuden jäsentämiseen; josta keskustelin tässä ketjussa kursorisesti toisaalla Belisario:n kanssa, on sellainen, että sen osana painotetaan vuorollaan *sekä* A) "todellisuuden kohtaamista sellaisenaan"; eli aiemmista tulkinnoista mahdollisimman riippumatonta havaitsemista; mikä siis esim. CC:n tapauksessa edellytti mm. psykoaktiivisten sienien käyttöä, *että* B) "todellisuuden oleellisten aspektien selittämistä". Tämä on tässä relevanttia liittyen mm. siihen, että miten fyysisen todellisuuden ilmiöt voidaan tulkita hyvin monilla erilaisilla tavoilla, jotka tulkinnat kokemuksellisina tapahtumina ovat subjektiivisesti relevantteja ja muodostavat välttämättömän perustan tietysti siis myös mm. määritelmille, aksioomille, teoreemoilla jne.

        CC:n opettajan tradition kanta tämän toiminnan organisointiin on ollut se, että optimaalinen järjestely on se, että oppilaalla on sekä opettaja ("teacher"), jonka tehtävä on tuon B-kohdan mahdollistaminen, että hyväntekijä ("benefactor"), jonka tehtävänä on tuon A-kohdan mahdollistaminen, koska etenkin hyväntekijän kohtaaminen on "hyvin raskasta järjelle" ja oli sitä erityisesti CC:n itsensä tapauksessa, koska CC oli erittäin taipuvainen etsimään kokemilleen ilmiöille *omien ennakko-oletustensa* mukaisesti järkiperäisiä selityksiä ja opettajan tehtävänä on oppilaan "järjen rauhoittaminen", tarjoamalla tradition mukaisia ja tuon tradition mukaan asetettujen käytännöllisten tavoitteiden saavuttamisen mahdollistamisen kannalta myös nimenomaan optimaalisia selityksiä koetuille ilmiöille, kuten tietysti myös ohjaaminen tunnistamaan se, että mikä on relevanttia tavoitteiden saavuttamisen kannalta...


      • Anonyymi
        Järkisyitä kirjoitti:

        Matematiikka on eksaktia laskentaa, mikä lähtee liikkeelle pienestä joukosta aksioomia eli premissejä. Näiden pohjalta johdetaan kaikki muu käyttäen eksakteja määritelmiä. Tässä suhteessa matematiikka muistuttaa mitä tahansa eksaktia filosofista päättelyä.

        Matematiikka ei ole vain yksi asia. Esimerkiksi geometrian aksioomilla ei ole mitään tekemistä reaalilukujen aksioomien kanssa. Imaginääriluvut ovat realilukujen laajennus ja joku voi perustellusti olla eri mieltä siitä, onko reaalilukujen laajennus imaginääri-akselille lainkaan perusteltua. Imaginäärilukuja voi perustellusti pitää täysin hatusta vedettynä asiana, mille ei ole mitään vastinetta reaalimaailmassa. Ne ovat vain meidän mielikuvituksen tuote - keksintö.

        Jos olemassa olemisella tarkoitetaan vain fyysisiä asioita, niin silloin abstraktit asiat kuten matematiikka ei ole olemassa ellei jokin tietoinen olento ajattele niitä, jolloin kyseiset käsitteet ovat olemassa aivojen tilana tai prosessina.

        Nimimerkki: Jumalan-kumous (ei-rekisteröitynyt professori)

        [jatkuu…] Koska todellisuuden kuvausten reaalisesti määritelmäänsä vastaavan määrittelytoiminnan esiehto on se, että ne kuvaukset ylipäätään on määritelty *jossakin vaiheessa* *jonkin* sisältöisiksi, niin tarvitaan vähintään yksi sellainen järjestelmä; eli käytännössä todellisuuden osajärjestelmä, joka *kykenee* tuohon määrittelytoimintaan.

        Toisaalta, koska määrittely on määrittelyprosessin tulos ja tuo tulos on informaatiota, niin kyseessä on tietojenkäsittelyprosessi ja toisaalta, koska ns. irreversiibeli; palautumaton tietojenkäsittely, kuluttaa aina tietojenkäsittelyn kannalta käyttökelpoista energiaa; viime kädessä johtuen siitä, että käytettävissä ei ole rajattomasti muistia ja oletusarvoisesti tuota energiaa tulisi tietysti olla käytettävissä tuohon tarkoitukseen *ainakin* jollain määrittelyn tuottamisen mahdollistavan pitkänä ajanjaksona, niin tuon määrittelyä suorittavan järjestelmän täytyy olla fyysinen, koska *pelkät* abstraktiot tai mielikuvitukseen perustuvat prosessit eivät kykene *suorittamaan* laskentaa, vaan niiden muodostaminen päinvastoin edellyttää laskentaa.

        Erityisesti sen täytyy olla fyysinen siinä merkityksessä, että se mahdollistaa; omalta osaltaan, fyysisen todellisuuden rakenteiden *tulkinnan* sellaiseen muotoon, että tuo muoto mahdollistaa sen, että esim. määritelmiä voidaan käyttää niihin tarkoituksiin joihin määritelmiä; " tavanomaisen" määritelmänsä mukaan, pitää voida käyttää...

        Tietoisuus liittyy tähän kuvioon välillisemmin "relevanssin kautta", eli jotta nuo määrittelyt voisivat olla kokemuksellisesti ja subjektiivisesti *relevantteja*, niin niillä tulisi voida olla *vähintään* jotain *vaikutusta* siihen, että miten ainakin jokin järjestelmä todellisuuden kokee. Eli, niillä voi olla merkitystä, vaikka *niitä itseään* ei tiedostettaisikaan, edellyttäen tietenkin, että ne ylipäätään ovat olemassa; edellä kuvatussa merkityksessä. Käytännössä ne ovat tuossa merkityksessä olemassa vain jos ne kuuluvat todellisuuden tilamuutosten kausaalisten syiden joukkoon, esim. (1) sitä kautta, että niitä käytetään jossain sellaisessa "algoritmiksi tulkittavissa olevassa fyysisessä prosessissa", jolla on; on omalta osaltaan, ohjaavaa vaikutusta noihin tilasiirtymiin.

        Kuitenkin; kuten olen aiemmin todennut, niin mm. oma todellisuuskäsitykseni on se, että koska todellisuudesta ei voida *reaalisesti* irrottaa mitään erilleen, niin tuo (1):kin on vain todellisten tapahtumien abstraktio; eli siis puutteellinen, ja *jos* siis fyysisen todellisuuden kompleksisuus on ääretön, niin äärimmäisen puutteellinen kuvaus...

        Käytännön tasolla, oleellista relevanssin kannalta on määritelmien *tulkinta*, koska käytännössä kaikki käyttämämme määritelmät ovat "ihmisten tuottamia", eli niihin liittyy tuollaista tulkintatoimintaa, eli olemassa on fysikaalinen järjestelmä, joka suorittaa mm. todellisuuden kuvausten tulkintaa, joka prosessi tuottaa kuvauksia, eli esim. kuten tässä tapauksessa, sellaisia {kuvauksen A} ja {kuvauksen A kohteen B} välisiä kuvauksia, joissa B:n sisältönä on {{kuvaus C}, {kuvauksen C kohde D}}, jotka vastaavat; jossain merkityksessä, toisiaan, eli ovat esim. totuuden korrespondenssiteorian mukaisessa merkityksessä paikkansapitäviä; määritelmällisesti, käytännön kannalta riittävässä määrin.

        Käytännössä todellisuus on siis "vähän niin kuin sipuli", mikä on myös mm. esim. mainitsemani CC:n tradition näkemys. Tuon tämän esille, koska; kuten totesin, olen kiinnostunut mm. erilaisten näkemysten synteeseistä. Käytännössä, esim. tuon tradition mukaan kaikilla oppilailla ei ole riittävästi "käyttökelpoista voimaa", joka taas on tuon tradition eräs *kaikkein keskeisimmistä käsitteistä* ja myös eräs keskeisistä tavoitteista, että he kykenisivät löytämään edes aiemmin mainitsemani tradition mukaisen opettajan; tai tarkemmin ilmaisten "selittämätön voima"; joka voidaan tulkita "fyysisten voimien nettovaikutukseksi", johtaa joissakin tapauksissa siihen, että jotkut oppilaat kohtaavat opettajan ja/tai myös hyväntekijän. CC:n maailmankuvan kuvauksissa ei käytetä matematiikkaa; vaikka olisikin sinällään mielenkiintoista ja ehkä hyvin origenelliakin kuvata sitä matemaattisesti, eikä siihen pitäisi olla periaatteellista estettä...

        *Pääasiallinen* este CC:n maailmankuvan omaksumiseen ei kuitenkaan liene sen kuvauksen monimerkityksisyys, koska se on ns. "sisäisesti varsin konsistentti", vaan lähtökohtainen kyvyttömyys kokea todellisuus tuon aiemmin mainitsemani A-kohdan merkityksessä empiirisesti "puhtaan havaitsemisen" kautta ja siten myöskään mieltää ko. maailmankuvan relevanssi, joskin semantiikkaankin liittyvät tekijät ovat suuri este, koska käytetyillä käsitteillä on omat erityiset tradition mukaiset merkityksensä, eivätkä nuo merkitykset myöskään usein ole samat, kuin ne merkitykset, joita ko. käsitteillä yleensä oletetaan olevan ja etenkin oppilasajan alussa ja etenkin mm. CC:n itsensä kaltaisille "rationaalisille tyypeille"...


      • Anonyymi

        Eikö matematiikka ole aika onneton tiede, jos siinä lähdetään liikkeelle perustotuuksista. Mikäs tiede se sellainen on. Vai onko se uskontoa. Eikö matematiikassa juuri jokainen matemaatikko lähde itsestään ja omasta järjestään käsin. Perustotuudetkin muuttuvat jos siltä näyttää. Eikös tämä ole oikea asenne. Ei kait perustotuudet ole matemaatikon ulkopuolella, jossain. Eikö ne ole juuri perustotuuksia vain sikäli kuin matemaatikko ne pystyy todistamaan.


    • Anonyymi

      Ratkaisemattomia ongelmia on vielä "List of unsolved problems in mathematics". Ne eivät kuitenkaan ole mielipidekysymyksiä eivätkä ratkea spekuloimalla.

      • Vaikka yhtään ongelmaa ei ratkaistaisi, kieristelisin jo hunajassa ja sokerissa, jos filosofit ymmärtäisivät, että matematiikalla ei ole mitään yhteyttä olemassaoloon.


    • Anonyymi

      Kuka filosofi tuollaista on väittänyt? Onko tuo taas yksi epätoivoinen yritys mustamaalata filosofiaa ja filosofeja. Mikä kauna teitä ajaa tuollaiseen toimintaan?
      Filosofit ovat nimen omaan kehittäneet matematiikkaa aina Pythagorasta lähtien. Voisit ottaa hieman asioista selvää, ennen kuin kirjoittelet sammakoita.

    • Anonyymi

      Nimimerkki: Jumalan-kumous (ei-rekisteröitynyt professori)

      Firdawsi: "[…] Todellisuudesta matematiikka on aina yhtä ja samaa, muuttumatonta totuutta, jota kyllä voidaan kirjoittaa eri tavalla, mutta sisältö ei muutu. […] Tähän perustuu myös erehdys, että matematiikka olisi riippuvainen materiasta ja sen prosesseista. Ei ole. Se on täysin riippumaton mahdollisesta aineen olemassaolosta ja sen laadusta, sekä ihmisen tekemistä "valinnoista"."

      Matematiikka; kuten *kaikki* muukin, on päinvastoin *täysin riippuvaista* fysikaalisista edellytyksistänsä, ensinnäkin koska *mikään* matemaattinen *sisältö* ei ole mahdollista ilman käytettävissä olevaa muistia, joka on aina fysikaalista. Myöskään *minkäänlaisen* matemaattisen notaation käyttö, minkään-sisältöisten määrittelyjen tekeminen, minkäänlaisten matemaattisten rakenteiden/prosessien havaitseminen ja/tai kuvittelu ja/tai itseasiassa ylipäätään mikään muukaan, ei olisi mahdollista ilman fysikaalisia edellytyksiänsä.

      Tässä kohtaa tosiaan vallitsee; valitettavasti varsin laajallekin levinnyt, erehdys, mutta erehdys on juuri siinä, että kuvitellaan, että matematiikka voisi olla jotenkin irrallista todellisuudesta; eli edellytyksistänsä, joka todellisuus kuitenkin on nimenomaan fysikaalinen. Matemaattisen esitystavan ja notaation avulla voidaan *kuvata* mm. yksiselitteisyyden kannalta optimaalisella tavalla mm. fysikaalisen todellisuuden rakenteita ja prosesseja, mutta kuvaus on silti vain kuvaus.

      Kuvaus on eri asia kuin kuvauksen kohde ja kuvaustapa, notaatio ja noudatettavat matematiikan harjoittamisen toimintatavat, proseduurit ja algoritmit ovat nimenomaan sopimuksenvaraisia ja määritelmällisiä. Ne voivat toki olla *optimaalisia* mm. todellisuuden tieteellisen kuvauksen kannalta (ns. Wigner:in dilemma), mutta ne eivät ole millään mielekkäällä tavalla edes olemassakaan irrallaan edellytyksistänsä. Perustuvaalaatua olevana syynä tuohon optimaalisuuteen ei ole se, että ne on määritelty optimaalisesti, vaan se, mikä on mahdollistanut; eli käytännössä itseasiassa *pakottanut*, määrittelemään ne optimaalisesti ja se on nimenomaan fysikaalinen todellisuus, eikä sen kuvaus.

      Fysikaalisen todellisuuden ensisijaisuus matematiikkaan ja tietysti kaikkiin muihinkin kuvaustapoihin/kuvausprosesseihin jne. verrattuna on ilmeinen, kun havaitaan, että kaikki matematiikan harjoittaminen edellyttää laskennallisia resursseja ja mikään matemaattinen prosessi ei kykene tuottamaan mitään laskennallisia resursseja. Tämän hahmottaminen lienee helpointa niille, joilla on kompetenssia *sekä* kovien luonnontieteiden, että laskennallisten tieteiden aloilla ja ehkä myös jonkinverran filosofista mielenlaatua; sanan positiivisessa merkityksessä, jolloin tarkoituksena on pyrkiä välttämään käsitteellisiä sekavuuksia ja käytännön kannalta hyödyttömiä umpikujia.

      Jos esim. todellisuutta kuvaavat matemaattiset mallit ovat sellaisia, että niihin ei olla sisällytetty; luonnollisesti kuvaustasolla, havaitun ja havaintojen edellyttämän kompleksisuuden edellyttämiä *reaalisia* fysikaalisia laskennallisia resursseja, niin tuloksena voi hyvin helposti olla esim. sellainen kvanttimekaaninen todellisuuden tulkinta, joka itseasiassa sisältää implisiittisesti epätieteellisiä mystisiä elementtejä. Kaikenkaikkiaan, matemaattiset oliot eivät suorita mitään laskentaa, eivätkä tuota mitään matemaattista kontribuutiota, vaan vain fysikaaliset oliot suorittavat/tuottavat sitä.

      Juttelen muuten ihan mielelläni sekä esim. sinun ja esim. Belisario:n kanssa esim. näistäkin teemoista; jos ehdin, vaikka esim. teillä on hyvin erilaisia näkemyksiä ja minulla ja Belisario:lla on myös aika erilaisia näkemyksiä ja myös meillä kahdella on nyt siis aika erilaisia näkemyksiä *nimenomaan* tämän "matematiikka-teeman osan osalta"; ainakin tällä hetkellä, vaikka olenkin hyvin samoilla linjoilla kanssasi, esim. tieteellisen menetelmän merkityksestä, sekavan filosofian hyödyttömyydestä jne. Tämä "kolmiomme" kyllä muodostaa *debatin muodostuksen* kannalta mielenkiintoisen "mix":in, vaikka konsensusta "tuskin" saavutetaankaan. Tavoitteeni onkin tässä lähinnä ehkä vain opetella kirjoittamaan ja argumentoimaan - entistä paremmin. :D

      • Itse matematiikka ei ole vähääkään riippuvainen fyysisistä edellytyksistä, sen sijaan matemaattinen tutkimus ja myös laskenta ovat. Matematiikka on vain aineetonta totuutta.

        Luultavasti filosofisesti orientoituvat eivät osaa näitä erottaa toisistaan ja siksi lyövät päätään seinään. Näyttää myös siltä, ettei voi oppiakaan erottamaan, jos on jo alkujaan joutunut sivuraiteelle.

        En luule, että saisin käännettyä jonkin pään. Pyrkimykseni on paljon vaatimattomampi. Pyrin vain pitämään sellaisen kanssaihmisen pään selvänä, jolla on edellytykset ymmärtää, mutta on vaarassa joutua kielellisen prujun eli filosofoinnin saaliiksi.

        Loogis-empiirisen tutkimuksen metodologia on perin yksinkertainen. Tehdään välillisesti tai välittömästi mitään edellyttämättömia havaintoja, jotka ovat riittävän runsaat. Selvitetään, onko matemaattinen tutkimus jo löytänyt sellaisen matemaattisen totuuden, jolla voidaan kuvata havaintojen esiintymistä.

        Syntynyt näkemys on hypoteesi, jota vielä testataan. Jos virhettä ei ole, hypoteesi korottuu teoriaksi. Mikäli useampi hypoteesi on yhtäpitävä todellisuuden kanssa, valitaan yksinkertaisin, koska yksinkertainen on todennäköisempi kuin mutkikas, ihan todennäköisyysmatematiikan mukaan.

        Ei ole vaihtoehtoista tapaa, jolla voisi saada validia tietoa. Filosofisia edellytyksiä ei ole, koska se on ainoa mahdollinen.


      • Anonyymi
        Firdawsi kirjoitti:

        Itse matematiikka ei ole vähääkään riippuvainen fyysisistä edellytyksistä, sen sijaan matemaattinen tutkimus ja myös laskenta ovat. Matematiikka on vain aineetonta totuutta.

        Luultavasti filosofisesti orientoituvat eivät osaa näitä erottaa toisistaan ja siksi lyövät päätään seinään. Näyttää myös siltä, ettei voi oppiakaan erottamaan, jos on jo alkujaan joutunut sivuraiteelle.

        En luule, että saisin käännettyä jonkin pään. Pyrkimykseni on paljon vaatimattomampi. Pyrin vain pitämään sellaisen kanssaihmisen pään selvänä, jolla on edellytykset ymmärtää, mutta on vaarassa joutua kielellisen prujun eli filosofoinnin saaliiksi.

        Loogis-empiirisen tutkimuksen metodologia on perin yksinkertainen. Tehdään välillisesti tai välittömästi mitään edellyttämättömia havaintoja, jotka ovat riittävän runsaat. Selvitetään, onko matemaattinen tutkimus jo löytänyt sellaisen matemaattisen totuuden, jolla voidaan kuvata havaintojen esiintymistä.

        Syntynyt näkemys on hypoteesi, jota vielä testataan. Jos virhettä ei ole, hypoteesi korottuu teoriaksi. Mikäli useampi hypoteesi on yhtäpitävä todellisuuden kanssa, valitaan yksinkertaisin, koska yksinkertainen on todennäköisempi kuin mutkikas, ihan todennäköisyysmatematiikan mukaan.

        Ei ole vaihtoehtoista tapaa, jolla voisi saada validia tietoa. Filosofisia edellytyksiä ei ole, koska se on ainoa mahdollinen.

        Nimimerkki: Jumalan-kumous (ei-rekisteröitynyt professori)

        Firdawsi: "Itse matematiikka ei ole vähääkään riippuvainen fyysisistä edellytyksistä, sen sijaan matemaattinen tutkimus ja myös laskenta ovat. Matematiikka on vain aineetonta totuutta."

        Jatkat aiemmista varsin yksityiskohtaisesti perustelluista vastaväitteistäni huolimatta nyt tuossa sen väittämistä, että matematiikka ei muka olisi riippuvaista fyysisistä edellytyksistänsä, mutta väitteeltäsi puuttuu tuossa perustelu, eikä se ole mm. tieteellisesti uskottava, koska ei-fysikaalisten asioiden havaitseminen ei ylipäätään ole edes mahdollistakaan.

        Matematiikka ei myöskään taatusti ole "vain aineetonta totuutta". Totuus on kvaliteetti ja sillä kuten muillakaan kvaliteeteilla ei ole mitään sisältöä, jos olemassa ei ole fysikaalisia elementtejä, joilla *tulkitaan* olevan kyseiseen kvaliteettiin liittyen jotain arvoja. Mitään tuollaista tulkintaa ei edes voida tehdä ilman fysikaalisia edellytyksiänsä jne. Fysiikka siis "rulettaa" tässäkin asiassa. Matematiikka ei muutenkaan ole totuutta, vaan fysikaalisten entiteettien tietyt ominaisuudet voidaan *tulkita* sellaisiksi attribuuteiksi, joilla on totuusarvo.

        "Luultavasti filosofisesti orientoituvat eivät osaa näitä erottaa toisistaan ja siksi lyövät päätään seinään. Näyttää myös siltä, ettei voi oppiakaan erottamaan, jos on jo alkujaan joutunut sivuraiteelle."

        Matematiikka on käsite, jolle on monenlaisia määritelmiä ja esittämäsi määritelmät ovat korkeintaan vain tietynlaisia matematiikan erilaisten aspektien luonnehdintoja, eivätkä onnistuneita, etenkään tuo viimeisin matematiikan luonnehdinta "vain aineettomaksi totuudeksi", koska matematiikka ei ole vain tuollaista, ei ole totuutta, eikä ole aineetontakaan. Tuollainen määritelmä ei vaikuta edes ollenkaan matemaattiseltakaan, vaan pikemmin uskonnollis-sävyiseltä epämääräiseltä luonnehdinnalta.

        Sivuraiteelle mennään jo siinä, jos oletetaan, että ideat ja informaatio ovat aineettomia. Eivät ole, vaan niissä on kyse sellaisista fysikaalisista prosesseista, jotka tuottavat fysikaalisten rakenteiden ja ilmiöiden *tulkinnan* ideoiksi ja informaatioksi. Aiemmin olet muistaakseni pyrkinyt määrittelemään; tai luonnehtimaan tms., matematiikkaa myös mm. jopa olemattomaksikin, eivätkä nuo luonnehdinnat todellakaan tunnu johtavan matematiikan olemusta selventävään lopputulokseen; *lievästi* ilmaisten...

        "En luule, että saisin käännettyä jonkin pään. Pyrkimykseni on paljon vaatimattomampi. Pyrin vain pitämään sellaisen kanssaihmisen pään selvänä, jolla on edellytykset ymmärtää, mutta on vaarassa joutua kielellisen prujun eli filosofoinnin saaliiksi."

        Tuo on asiallinen tavoite, mutta mielestäni olet jo itsekin joutunut juuri tuollaisen prujun tms. tai jonkin muun saaliiksi, jos oletat, että olemassa olisi "fysikaalisista edellytyksistänsä irrallisia aineettomia ideoita"; mukaan lukien tietysti *mm.* tuo matematiikan idea, eli matematiikan idea on tietysti vain ns. "idea muiden joukossa". Koskaan ei olla havaittu mitään ideoita kellumassa avaruudessa, vaan ne ovat vain ihmisaivojen tuottamia fysikaalisten prosessien tulkintoja ja tietysti siis itsekin fyysisiä...

        "Loogis-empiirisen tutkimuksen metodologia on perin yksinkertainen. Tehdään välillisesti tai välittömästi mitään edellyttämättömia havaintoja, jotka ovat riittävän runsaat. Selvitetään, onko matemaattinen tutkimus jo löytänyt sellaisen matemaattisen totuuden, jolla voidaan kuvata havaintojen esiintymistä."

        Uh! *Mitään* "mitään edellyttämättömiä havaintoja" ei ylipäätään voi olla olemassakaan, vaan havaitsemisella nimenomaan on *aivan erityisen tiukat* fysikaaliset edellytykset. Matematiikka on "vain" todellisuuden mallintamisen apuväline; joskin siis erityisen hyödyllinen apuväline, mm. ja erityisesti pyrittäessä kuvaamaan esim. havaintoja suhteellisen tiiviillä ja yksiselitteisellä tavalla.

        Käytännössä sen, että mitä tietty osajärjestelmä kykenee havaitsemaan ja sen, että mitä se kykenee mallintamaan matemaattisesti tai muutenkaan ja/tai ylipäätään tekemään, rajaa yksikäsitteisesti tuon osajärjestelmän ja kokonaisjärjestelmän välinen suhde, eli esim. tietyn universumin osien keskimääräinen käyttäytyminen mahdollistaa vain nimenomaan tietyntyyppisen matematiikan.

        "Syntynyt näkemys on hypoteesi, jota vielä testataan. Jos virhettä ei ole, hypoteesi korottuu teoriaksi. [...]"

        Kannatan sitä, että väistämättä aina rajalliset tutkimusresurssit kohdennetaan asianmukaisessa määrin mm. todennäköisimmin paikkansapitävien hypoteesien testaukseen sen sijaan, että ne kohdennettaisiin esim. havaintoihin perustumattomien filosofisten vaihtoehtojen analysointiin.


      • Anonyymi kirjoitti:

        Nimimerkki: Jumalan-kumous (ei-rekisteröitynyt professori)

        Firdawsi: "Itse matematiikka ei ole vähääkään riippuvainen fyysisistä edellytyksistä, sen sijaan matemaattinen tutkimus ja myös laskenta ovat. Matematiikka on vain aineetonta totuutta."

        Jatkat aiemmista varsin yksityiskohtaisesti perustelluista vastaväitteistäni huolimatta nyt tuossa sen väittämistä, että matematiikka ei muka olisi riippuvaista fyysisistä edellytyksistänsä, mutta väitteeltäsi puuttuu tuossa perustelu, eikä se ole mm. tieteellisesti uskottava, koska ei-fysikaalisten asioiden havaitseminen ei ylipäätään ole edes mahdollistakaan.

        Matematiikka ei myöskään taatusti ole "vain aineetonta totuutta". Totuus on kvaliteetti ja sillä kuten muillakaan kvaliteeteilla ei ole mitään sisältöä, jos olemassa ei ole fysikaalisia elementtejä, joilla *tulkitaan* olevan kyseiseen kvaliteettiin liittyen jotain arvoja. Mitään tuollaista tulkintaa ei edes voida tehdä ilman fysikaalisia edellytyksiänsä jne. Fysiikka siis "rulettaa" tässäkin asiassa. Matematiikka ei muutenkaan ole totuutta, vaan fysikaalisten entiteettien tietyt ominaisuudet voidaan *tulkita* sellaisiksi attribuuteiksi, joilla on totuusarvo.

        "Luultavasti filosofisesti orientoituvat eivät osaa näitä erottaa toisistaan ja siksi lyövät päätään seinään. Näyttää myös siltä, ettei voi oppiakaan erottamaan, jos on jo alkujaan joutunut sivuraiteelle."

        Matematiikka on käsite, jolle on monenlaisia määritelmiä ja esittämäsi määritelmät ovat korkeintaan vain tietynlaisia matematiikan erilaisten aspektien luonnehdintoja, eivätkä onnistuneita, etenkään tuo viimeisin matematiikan luonnehdinta "vain aineettomaksi totuudeksi", koska matematiikka ei ole vain tuollaista, ei ole totuutta, eikä ole aineetontakaan. Tuollainen määritelmä ei vaikuta edes ollenkaan matemaattiseltakaan, vaan pikemmin uskonnollis-sävyiseltä epämääräiseltä luonnehdinnalta.

        Sivuraiteelle mennään jo siinä, jos oletetaan, että ideat ja informaatio ovat aineettomia. Eivät ole, vaan niissä on kyse sellaisista fysikaalisista prosesseista, jotka tuottavat fysikaalisten rakenteiden ja ilmiöiden *tulkinnan* ideoiksi ja informaatioksi. Aiemmin olet muistaakseni pyrkinyt määrittelemään; tai luonnehtimaan tms., matematiikkaa myös mm. jopa olemattomaksikin, eivätkä nuo luonnehdinnat todellakaan tunnu johtavan matematiikan olemusta selventävään lopputulokseen; *lievästi* ilmaisten...

        "En luule, että saisin käännettyä jonkin pään. Pyrkimykseni on paljon vaatimattomampi. Pyrin vain pitämään sellaisen kanssaihmisen pään selvänä, jolla on edellytykset ymmärtää, mutta on vaarassa joutua kielellisen prujun eli filosofoinnin saaliiksi."

        Tuo on asiallinen tavoite, mutta mielestäni olet jo itsekin joutunut juuri tuollaisen prujun tms. tai jonkin muun saaliiksi, jos oletat, että olemassa olisi "fysikaalisista edellytyksistänsä irrallisia aineettomia ideoita"; mukaan lukien tietysti *mm.* tuo matematiikan idea, eli matematiikan idea on tietysti vain ns. "idea muiden joukossa". Koskaan ei olla havaittu mitään ideoita kellumassa avaruudessa, vaan ne ovat vain ihmisaivojen tuottamia fysikaalisten prosessien tulkintoja ja tietysti siis itsekin fyysisiä...

        "Loogis-empiirisen tutkimuksen metodologia on perin yksinkertainen. Tehdään välillisesti tai välittömästi mitään edellyttämättömia havaintoja, jotka ovat riittävän runsaat. Selvitetään, onko matemaattinen tutkimus jo löytänyt sellaisen matemaattisen totuuden, jolla voidaan kuvata havaintojen esiintymistä."

        Uh! *Mitään* "mitään edellyttämättömiä havaintoja" ei ylipäätään voi olla olemassakaan, vaan havaitsemisella nimenomaan on *aivan erityisen tiukat* fysikaaliset edellytykset. Matematiikka on "vain" todellisuuden mallintamisen apuväline; joskin siis erityisen hyödyllinen apuväline, mm. ja erityisesti pyrittäessä kuvaamaan esim. havaintoja suhteellisen tiiviillä ja yksiselitteisellä tavalla.

        Käytännössä sen, että mitä tietty osajärjestelmä kykenee havaitsemaan ja sen, että mitä se kykenee mallintamaan matemaattisesti tai muutenkaan ja/tai ylipäätään tekemään, rajaa yksikäsitteisesti tuon osajärjestelmän ja kokonaisjärjestelmän välinen suhde, eli esim. tietyn universumin osien keskimääräinen käyttäytyminen mahdollistaa vain nimenomaan tietyntyyppisen matematiikan.

        "Syntynyt näkemys on hypoteesi, jota vielä testataan. Jos virhettä ei ole, hypoteesi korottuu teoriaksi. [...]"

        Kannatan sitä, että väistämättä aina rajalliset tutkimusresurssit kohdennetaan asianmukaisessa määrin mm. todennäköisimmin paikkansapitävien hypoteesien testaukseen sen sijaan, että ne kohdennettaisiin esim. havaintoihin perustumattomien filosofisten vaihtoehtojen analysointiin.

        Ehkä lopetamme tähän. Olen vanha, sairas ja hyvin, hyvin väsynyt. Luonnontieteellinen tutkimus etenee edelleen. Olen aikoinani opiskellut filosofiaa, joten olen keskustellut aiheesta jo aivan riittävästi. Hyvää syksyä ja talvea sinulle ja Belisariolle.


      • Anonyymi
        Firdawsi kirjoitti:

        Ehkä lopetamme tähän. Olen vanha, sairas ja hyvin, hyvin väsynyt. Luonnontieteellinen tutkimus etenee edelleen. Olen aikoinani opiskellut filosofiaa, joten olen keskustellut aiheesta jo aivan riittävästi. Hyvää syksyä ja talvea sinulle ja Belisariolle.

        Nimimerkki: Jumalan-kumous (ei-rekisteröitynyt professori)

        "Hyvää syksyä ja talvea sinulle ja Belisariolle."

        Hyvää syksyä ja talvea myös sinulle - ja Belisariolle.


      • Firdawsi kirjoitti:

        Itse matematiikka ei ole vähääkään riippuvainen fyysisistä edellytyksistä, sen sijaan matemaattinen tutkimus ja myös laskenta ovat. Matematiikka on vain aineetonta totuutta.

        Luultavasti filosofisesti orientoituvat eivät osaa näitä erottaa toisistaan ja siksi lyövät päätään seinään. Näyttää myös siltä, ettei voi oppiakaan erottamaan, jos on jo alkujaan joutunut sivuraiteelle.

        En luule, että saisin käännettyä jonkin pään. Pyrkimykseni on paljon vaatimattomampi. Pyrin vain pitämään sellaisen kanssaihmisen pään selvänä, jolla on edellytykset ymmärtää, mutta on vaarassa joutua kielellisen prujun eli filosofoinnin saaliiksi.

        Loogis-empiirisen tutkimuksen metodologia on perin yksinkertainen. Tehdään välillisesti tai välittömästi mitään edellyttämättömia havaintoja, jotka ovat riittävän runsaat. Selvitetään, onko matemaattinen tutkimus jo löytänyt sellaisen matemaattisen totuuden, jolla voidaan kuvata havaintojen esiintymistä.

        Syntynyt näkemys on hypoteesi, jota vielä testataan. Jos virhettä ei ole, hypoteesi korottuu teoriaksi. Mikäli useampi hypoteesi on yhtäpitävä todellisuuden kanssa, valitaan yksinkertaisin, koska yksinkertainen on todennäköisempi kuin mutkikas, ihan todennäköisyysmatematiikan mukaan.

        Ei ole vaihtoehtoista tapaa, jolla voisi saada validia tietoa. Filosofisia edellytyksiä ei ole, koska se on ainoa mahdollinen.

        Olen tässä kyllä Firdawsin kanssa samaa mieltä. Matematiikan abstraktien käsitteiden olemassa olo ei edellytä fyysistä todellisuutta. Tässä 'olemassa olo' ei ole fyysistä eikä edes henkistä/kokemuksellista vaan sitä, että kyseinen totuus pätee aina.

        Nämä (matemaattiset) totuudet ovat pakosti kuten ne ovat - ne eivät voi olla mitenkään muuten. Ne ovat perimmäisiä totuuksia.

        Tälläisien perustavaa laatua olevien totuuksien olemassa olo on mielenkiintoista, koska niiden kautta voi olla mahdollista hakea vastausta filosofeja askarruttaneeseen kysymykseen: "Miksi on olemassa jotakin sen sijaan, ettei olisi olemassa mitään?"


      • Anonyymi kirjoitti:

        Nimimerkki: Jumalan-kumous (ei-rekisteröitynyt professori)

        Firdawsi: "Itse matematiikka ei ole vähääkään riippuvainen fyysisistä edellytyksistä, sen sijaan matemaattinen tutkimus ja myös laskenta ovat. Matematiikka on vain aineetonta totuutta."

        Jatkat aiemmista varsin yksityiskohtaisesti perustelluista vastaväitteistäni huolimatta nyt tuossa sen väittämistä, että matematiikka ei muka olisi riippuvaista fyysisistä edellytyksistänsä, mutta väitteeltäsi puuttuu tuossa perustelu, eikä se ole mm. tieteellisesti uskottava, koska ei-fysikaalisten asioiden havaitseminen ei ylipäätään ole edes mahdollistakaan.

        Matematiikka ei myöskään taatusti ole "vain aineetonta totuutta". Totuus on kvaliteetti ja sillä kuten muillakaan kvaliteeteilla ei ole mitään sisältöä, jos olemassa ei ole fysikaalisia elementtejä, joilla *tulkitaan* olevan kyseiseen kvaliteettiin liittyen jotain arvoja. Mitään tuollaista tulkintaa ei edes voida tehdä ilman fysikaalisia edellytyksiänsä jne. Fysiikka siis "rulettaa" tässäkin asiassa. Matematiikka ei muutenkaan ole totuutta, vaan fysikaalisten entiteettien tietyt ominaisuudet voidaan *tulkita* sellaisiksi attribuuteiksi, joilla on totuusarvo.

        "Luultavasti filosofisesti orientoituvat eivät osaa näitä erottaa toisistaan ja siksi lyövät päätään seinään. Näyttää myös siltä, ettei voi oppiakaan erottamaan, jos on jo alkujaan joutunut sivuraiteelle."

        Matematiikka on käsite, jolle on monenlaisia määritelmiä ja esittämäsi määritelmät ovat korkeintaan vain tietynlaisia matematiikan erilaisten aspektien luonnehdintoja, eivätkä onnistuneita, etenkään tuo viimeisin matematiikan luonnehdinta "vain aineettomaksi totuudeksi", koska matematiikka ei ole vain tuollaista, ei ole totuutta, eikä ole aineetontakaan. Tuollainen määritelmä ei vaikuta edes ollenkaan matemaattiseltakaan, vaan pikemmin uskonnollis-sävyiseltä epämääräiseltä luonnehdinnalta.

        Sivuraiteelle mennään jo siinä, jos oletetaan, että ideat ja informaatio ovat aineettomia. Eivät ole, vaan niissä on kyse sellaisista fysikaalisista prosesseista, jotka tuottavat fysikaalisten rakenteiden ja ilmiöiden *tulkinnan* ideoiksi ja informaatioksi. Aiemmin olet muistaakseni pyrkinyt määrittelemään; tai luonnehtimaan tms., matematiikkaa myös mm. jopa olemattomaksikin, eivätkä nuo luonnehdinnat todellakaan tunnu johtavan matematiikan olemusta selventävään lopputulokseen; *lievästi* ilmaisten...

        "En luule, että saisin käännettyä jonkin pään. Pyrkimykseni on paljon vaatimattomampi. Pyrin vain pitämään sellaisen kanssaihmisen pään selvänä, jolla on edellytykset ymmärtää, mutta on vaarassa joutua kielellisen prujun eli filosofoinnin saaliiksi."

        Tuo on asiallinen tavoite, mutta mielestäni olet jo itsekin joutunut juuri tuollaisen prujun tms. tai jonkin muun saaliiksi, jos oletat, että olemassa olisi "fysikaalisista edellytyksistänsä irrallisia aineettomia ideoita"; mukaan lukien tietysti *mm.* tuo matematiikan idea, eli matematiikan idea on tietysti vain ns. "idea muiden joukossa". Koskaan ei olla havaittu mitään ideoita kellumassa avaruudessa, vaan ne ovat vain ihmisaivojen tuottamia fysikaalisten prosessien tulkintoja ja tietysti siis itsekin fyysisiä...

        "Loogis-empiirisen tutkimuksen metodologia on perin yksinkertainen. Tehdään välillisesti tai välittömästi mitään edellyttämättömia havaintoja, jotka ovat riittävän runsaat. Selvitetään, onko matemaattinen tutkimus jo löytänyt sellaisen matemaattisen totuuden, jolla voidaan kuvata havaintojen esiintymistä."

        Uh! *Mitään* "mitään edellyttämättömiä havaintoja" ei ylipäätään voi olla olemassakaan, vaan havaitsemisella nimenomaan on *aivan erityisen tiukat* fysikaaliset edellytykset. Matematiikka on "vain" todellisuuden mallintamisen apuväline; joskin siis erityisen hyödyllinen apuväline, mm. ja erityisesti pyrittäessä kuvaamaan esim. havaintoja suhteellisen tiiviillä ja yksiselitteisellä tavalla.

        Käytännössä sen, että mitä tietty osajärjestelmä kykenee havaitsemaan ja sen, että mitä se kykenee mallintamaan matemaattisesti tai muutenkaan ja/tai ylipäätään tekemään, rajaa yksikäsitteisesti tuon osajärjestelmän ja kokonaisjärjestelmän välinen suhde, eli esim. tietyn universumin osien keskimääräinen käyttäytyminen mahdollistaa vain nimenomaan tietyntyyppisen matematiikan.

        "Syntynyt näkemys on hypoteesi, jota vielä testataan. Jos virhettä ei ole, hypoteesi korottuu teoriaksi. [...]"

        Kannatan sitä, että väistämättä aina rajalliset tutkimusresurssit kohdennetaan asianmukaisessa määrin mm. todennäköisimmin paikkansapitävien hypoteesien testaukseen sen sijaan, että ne kohdennettaisiin esim. havaintoihin perustumattomien filosofisten vaihtoehtojen analysointiin.

        Ymmärrän mielestäni kaiken, mitä 'Jumalan-kumous' kirjoitti, mutta silti en koe, että hän ottaa kantaa Firdawsin-argumentteihin. On kuin puhuisitte kahdesta eri asiasta.

        Tietenkään matemaattiset totuudet eivät ole fyysisiä tai edes kokemuksellisia asioita eli tässä mielessä niitä ei ole olemassa. Idealistit voisivat sanoa, että jopa fyysistä maailmaa ei ole olemassa ellei joku ole sitä kokemassa. Kenties tässä takerrumme siihen, mitä ymmärrämme olemassa olon tarkoittavan.

        Lakkaako fyysinen todellisuus olemasta silloin, jos kukaan ei koe sitä tietoisesti? Entä lakkaako matemaattinen totuus olemasta silloin, kun sitä ei ajattele kukaan? Miten lähdemme todistamaan, että joku asia on silloinkin, kun emme havaitse sitä?

        Sanotaan, että näet puun. Suljet silmät. Avaat silmät ja näet puun edelleen. Eikö tästä ole johdonmukaista olettaa, että puu on olemassa riippumatta siitä, että sitä havainnoidaan koko ajan?

        Sovelletaan samaa matemaattiseen totuuteen. Sekin pysyy muuttumattomana täysin riippumatta siitä havainnoiko sitä joku. Jokainen tietoinen olento havaitsee sen samanlaisena. Täten katsoisin, että se on abstraktina käsitteenä aina olemassa. Se ei katoa ja jokainen tietoinen olento kykenee havaitsemaan sen olemassa olon aina ajatellessaan sitä.


      • Anonyymi
        Järkisyitä kirjoitti:

        Olen tässä kyllä Firdawsin kanssa samaa mieltä. Matematiikan abstraktien käsitteiden olemassa olo ei edellytä fyysistä todellisuutta. Tässä 'olemassa olo' ei ole fyysistä eikä edes henkistä/kokemuksellista vaan sitä, että kyseinen totuus pätee aina.

        Nämä (matemaattiset) totuudet ovat pakosti kuten ne ovat - ne eivät voi olla mitenkään muuten. Ne ovat perimmäisiä totuuksia.

        Tälläisien perustavaa laatua olevien totuuksien olemassa olo on mielenkiintoista, koska niiden kautta voi olla mahdollista hakea vastausta filosofeja askarruttaneeseen kysymykseen: "Miksi on olemassa jotakin sen sijaan, ettei olisi olemassa mitään?"

        "Nämä (matemaattiset) totuudet ovat pakosti kuten ne ovat - ne eivät voi olla mitenkään muuten. Ne ovat perimmäisiä totuuksia."

        Niin ne tuntuvat pätevän ainakin tässä meidän mahdollisessa ja aktualisoituneessa maailmassa riittävän hyvin mutta pystyn aika helposti kuvittelemaan sellaisenkin mahdollisen maailman jossa tuntemamme matematiikka ei toimi juuri lainkaan.

        Maailma jossa ei ole mitään selkeästi erillisiä luokiteltavissa olevia olioita eikä mitään selkeitä säännönmukaisuuksia on maailma jossa tuntemallamme matematiikalla ei ole juurikaan käyttöä. Samoin maailma jossa on pelkkiä laatuja mutta ei lainkaan määriä eikä mitään selkeästi vertailukelpoisia asioita jää matematiikan ulottumattomiin.

        Matematiikan näennäinen "totuus" tarkoittaa vain sitä että matemaattinen aksiomaattinen järjestelmä on loogisesti ristiriidaton mutta ei sen silti tarvitse vastata mitään fysikaalisesti olemassaolevaa asiaa.

        Belisario

        ps. hyvät syksyt ja talvet myös Firwdasille ja Jumalan kumoukselle. Veikkaan kyllä että Firwdasi palaa jonkin ajan kuluttua takaisin. Erimielisten kanssa väittely vaatii aika paljon energiaa ja hyviä hermoja sekä kestävyyttä - usein enemmän kuin mikään fyysinen ponnistelu.


      • Anonyymi
        Järkisyitä kirjoitti:

        Ymmärrän mielestäni kaiken, mitä 'Jumalan-kumous' kirjoitti, mutta silti en koe, että hän ottaa kantaa Firdawsin-argumentteihin. On kuin puhuisitte kahdesta eri asiasta.

        Tietenkään matemaattiset totuudet eivät ole fyysisiä tai edes kokemuksellisia asioita eli tässä mielessä niitä ei ole olemassa. Idealistit voisivat sanoa, että jopa fyysistä maailmaa ei ole olemassa ellei joku ole sitä kokemassa. Kenties tässä takerrumme siihen, mitä ymmärrämme olemassa olon tarkoittavan.

        Lakkaako fyysinen todellisuus olemasta silloin, jos kukaan ei koe sitä tietoisesti? Entä lakkaako matemaattinen totuus olemasta silloin, kun sitä ei ajattele kukaan? Miten lähdemme todistamaan, että joku asia on silloinkin, kun emme havaitse sitä?

        Sanotaan, että näet puun. Suljet silmät. Avaat silmät ja näet puun edelleen. Eikö tästä ole johdonmukaista olettaa, että puu on olemassa riippumatta siitä, että sitä havainnoidaan koko ajan?

        Sovelletaan samaa matemaattiseen totuuteen. Sekin pysyy muuttumattomana täysin riippumatta siitä havainnoiko sitä joku. Jokainen tietoinen olento havaitsee sen samanlaisena. Täten katsoisin, että se on abstraktina käsitteenä aina olemassa. Se ei katoa ja jokainen tietoinen olento kykenee havaitsemaan sen olemassa olon aina ajatellessaan sitä.

        "Lakkaako fyysinen todellisuus olemasta silloin, jos kukaan ei koe sitä tietoisesti? "

        Se ns. "fyysinen todellisuus" on aina tietoisuuden sisältö ja emme voi kokea eikä havaita tietoisuudemme sisältöjen ulkopuolella mahdollisesti olevia asioita. En myöskään voi tietää onko kokemani väri täsmälleen sama kuin sinun kokemasi väri vaikka käytätkin sille samaa käsitettä (esim. punainen) vaikka toisaaalta periaatteessa tietyt värien väliset säännönmukaisuudet voivat päteä havaitsijasta riippumatta.

        "Sanotaan, että näet puun. Suljet silmät. Avaat silmät ja näet puun edelleen."

        Voi myös olla että näet hyvin todentuntuista unta puusta ja sen katsomisesta. Vasta herättyäsi ns. valvemaailmaan huomaat eron. Voi myös olla että heräätkin vain toisenlaiseen uneen eli pelkästään tietoisuuden datastream vaihtuu toiseksi.

        "Sovelletaan samaa matemaattiseen totuuteen. Sekin pysyy muuttumattomana täysin riippumatta siitä havainnoiko sitä joku. "

        Kyllä mutta ei välttämättä havaitsijasta riippumatta. On aivan hyvin mahdollista hahmottaa kaikki kaikki todellisuus prosessien kautta (esim. A.N. Whitehead ja prosessiontologia) eikä kappaleina/hiukkasina jolloin kausaliteettikin joudutaan hahmottamaan toisin ajallisesti ulotteisina tapahtumasykleinä jotka muodostavat identitettinsä alkunsa ja loppunsa kautta.

        Sen sijaan että tapahtuma A aiheuttaa tapahtuman B niin A & B muodostavat yhdessä prosessisysteemin jossa ei ole selkeästi erillisiä syitä eikä seurauksia eikä se prosessisysteemikään ole riippumaton ja erillinen koko muusta samaan kokonaisuuteen kuuluvasta kaikkeudesta.

        Matemaattiset "totuudet" ovat täysin riippuvaisia hahmotustavasta ja sama tietoinen olio voi soveltaa useampiakin keskenään yhteismitattomia hahmotustapoja saman tietoisuuden objektin käsittelemiseen ja ne hahmotustavat voivat olla toisiaan täydentäviä vaikka ovatkin erilaisia.

        Belisario


      • Anonyymi kirjoitti:

        "Nämä (matemaattiset) totuudet ovat pakosti kuten ne ovat - ne eivät voi olla mitenkään muuten. Ne ovat perimmäisiä totuuksia."

        Niin ne tuntuvat pätevän ainakin tässä meidän mahdollisessa ja aktualisoituneessa maailmassa riittävän hyvin mutta pystyn aika helposti kuvittelemaan sellaisenkin mahdollisen maailman jossa tuntemamme matematiikka ei toimi juuri lainkaan.

        Maailma jossa ei ole mitään selkeästi erillisiä luokiteltavissa olevia olioita eikä mitään selkeitä säännönmukaisuuksia on maailma jossa tuntemallamme matematiikalla ei ole juurikaan käyttöä. Samoin maailma jossa on pelkkiä laatuja mutta ei lainkaan määriä eikä mitään selkeästi vertailukelpoisia asioita jää matematiikan ulottumattomiin.

        Matematiikan näennäinen "totuus" tarkoittaa vain sitä että matemaattinen aksiomaattinen järjestelmä on loogisesti ristiriidaton mutta ei sen silti tarvitse vastata mitään fysikaalisesti olemassaolevaa asiaa.

        Belisario

        ps. hyvät syksyt ja talvet myös Firwdasille ja Jumalan kumoukselle. Veikkaan kyllä että Firwdasi palaa jonkin ajan kuluttua takaisin. Erimielisten kanssa väittely vaatii aika paljon energiaa ja hyviä hermoja sekä kestävyyttä - usein enemmän kuin mikään fyysinen ponnistelu.

        "Matematiikan näennäinen "totuus" tarkoittaa vain sitä että matemaattinen aksiomaattinen järjestelmä on loogisesti ristiriidaton mutta ei sen silti tarvitse vastata mitään fysikaalisesti olemassaolevaa asiaa."

        Tismalleen! Sitähän minä yritän sanoa. Matemaattiset totuudet ovat olemassa täysin riippumatta siitä millaisessa maailmassa satumme olemaan!

        Se, ettei jossakin maailmassa ole lukumääriä on täysin irrelevanttia. Meidänkään maailmassa ei ole olemassa viisiulotteista palloa. Tästä huolimatta voin laskea kyseisen pallon pinta-alan ja tilavuuden suhteessa sen säteeseen.

        Joudut keksimään maailman, missä logiikka ei päde. Kokonaisluku aritmetiikka voidaan johtaa joukko-opista eli joudut keksimään maailman, missä joukko-opin aksioomat ovat epätosia.

        Tässä pääsemmekin kysymykseen: Onko logiikka universaalia vai voiko olla maailmoja, jossa logiikkakin on epätosi? Mehän voisimme jopa todistaa tuollaisen maailman mahdottomaksi, koska se olisi logiikan vastainen.


      • Anonyymi kirjoitti:

        "Lakkaako fyysinen todellisuus olemasta silloin, jos kukaan ei koe sitä tietoisesti? "

        Se ns. "fyysinen todellisuus" on aina tietoisuuden sisältö ja emme voi kokea eikä havaita tietoisuudemme sisältöjen ulkopuolella mahdollisesti olevia asioita. En myöskään voi tietää onko kokemani väri täsmälleen sama kuin sinun kokemasi väri vaikka käytätkin sille samaa käsitettä (esim. punainen) vaikka toisaaalta periaatteessa tietyt värien väliset säännönmukaisuudet voivat päteä havaitsijasta riippumatta.

        "Sanotaan, että näet puun. Suljet silmät. Avaat silmät ja näet puun edelleen."

        Voi myös olla että näet hyvin todentuntuista unta puusta ja sen katsomisesta. Vasta herättyäsi ns. valvemaailmaan huomaat eron. Voi myös olla että heräätkin vain toisenlaiseen uneen eli pelkästään tietoisuuden datastream vaihtuu toiseksi.

        "Sovelletaan samaa matemaattiseen totuuteen. Sekin pysyy muuttumattomana täysin riippumatta siitä havainnoiko sitä joku. "

        Kyllä mutta ei välttämättä havaitsijasta riippumatta. On aivan hyvin mahdollista hahmottaa kaikki kaikki todellisuus prosessien kautta (esim. A.N. Whitehead ja prosessiontologia) eikä kappaleina/hiukkasina jolloin kausaliteettikin joudutaan hahmottamaan toisin ajallisesti ulotteisina tapahtumasykleinä jotka muodostavat identitettinsä alkunsa ja loppunsa kautta.

        Sen sijaan että tapahtuma A aiheuttaa tapahtuman B niin A & B muodostavat yhdessä prosessisysteemin jossa ei ole selkeästi erillisiä syitä eikä seurauksia eikä se prosessisysteemikään ole riippumaton ja erillinen koko muusta samaan kokonaisuuteen kuuluvasta kaikkeudesta.

        Matemaattiset "totuudet" ovat täysin riippuvaisia hahmotustavasta ja sama tietoinen olio voi soveltaa useampiakin keskenään yhteismitattomia hahmotustapoja saman tietoisuuden objektin käsittelemiseen ja ne hahmotustavat voivat olla toisiaan täydentäviä vaikka ovatkin erilaisia.

        Belisario

        "Matemaattiset "totuudet" ovat täysin riippuvaisia hahmotustavasta"

        Eivät ole. Toki aina voit ajatella väärin, mutta se ei tarkoita, että loogiset totuudet olisivat muuttuneet.


      • Anonyymi
        Järkisyitä kirjoitti:

        "Matematiikan näennäinen "totuus" tarkoittaa vain sitä että matemaattinen aksiomaattinen järjestelmä on loogisesti ristiriidaton mutta ei sen silti tarvitse vastata mitään fysikaalisesti olemassaolevaa asiaa."

        Tismalleen! Sitähän minä yritän sanoa. Matemaattiset totuudet ovat olemassa täysin riippumatta siitä millaisessa maailmassa satumme olemaan!

        Se, ettei jossakin maailmassa ole lukumääriä on täysin irrelevanttia. Meidänkään maailmassa ei ole olemassa viisiulotteista palloa. Tästä huolimatta voin laskea kyseisen pallon pinta-alan ja tilavuuden suhteessa sen säteeseen.

        Joudut keksimään maailman, missä logiikka ei päde. Kokonaisluku aritmetiikka voidaan johtaa joukko-opista eli joudut keksimään maailman, missä joukko-opin aksioomat ovat epätosia.

        Tässä pääsemmekin kysymykseen: Onko logiikka universaalia vai voiko olla maailmoja, jossa logiikkakin on epätosi? Mehän voisimme jopa todistaa tuollaisen maailman mahdottomaksi, koska se olisi logiikan vastainen.

        "Joudut keksimään maailman, missä logiikka ei päde. "

        Voimme kuvitella sellaisia maailmoja jotka ovat loogisesti ristiriidattomia. Voimme myös kehitää sellaisen logiikan joka ei ole kaavamaisesti 2-arvoinen, binäärinen. joko tai logiikka vaan esim. jonkinlainen sekä-että logiikka kuten esim. kvanttifysikkan superpositiot.

        "Kokonaisluku aritmetiikka voidaan johtaa joukko-opista eli joudut keksimään maailman, missä joukko-opin aksioomat ovat epätosia."

        Ei ole mielekästä suorittaa kokonaislukuoperaatioita keskenään hyvin erilaisten asioiden välillä. Vain samaan luokitteluun liittyviä asioita on hyödyllistä laskea yhteen. vähentää toisistaan jne. Esim. omenoiden, rahojen, apinoiden ja kolmioiden yhteenlaskeminen keskenään ei ole kovin mielekästä. Sen lisäksi että yhteenlaskettavien asioiden pitää olla samaan joukkoon kuuluvia niin niiden pitää olla myös selkeästi erillisiä.

        Kokonaisluvut liittyvät lähinnä kiinteisiin kappaleisiin. Kaasumaiset ja nestemäiset oliot ovat hankalampia yhteenlaskettavia varsinkin jos hahmottaa ne sillä tavalla että ei "riko" niiden asioiden tiettyyn kokonaisuuteen liittyviä kemiallisia ominaisuuksia pilkkomalla ne protoneiksi, neutroneiksi ja elektroneiksi tai kvarkeiksi. Esim, molekyylit ja atomit ovat pienimpiä yksiköitä joilla on juuri tietyt "yksilölliset" kemialliset ominaisuudet.

        Matematiikka siis edellyttää jonkinlaista luokittelua ja identiteetin määrittelyä ennen kuin voidaan suorittaa mielekkäästi laskutoimituksia ja ne luokittelut ja määrittelyt ovat pohjimmiltaan metafysiikkaan ja filosofisiin oletuksiin liittyviä eli siis pojhimmiltaan vain tapoja hahmottaa koettua todellisuutta.

        Loogiset totuudet eivät tietenkään muutu koska niillä ei ole aikaan liittyvää komponenttia kuten aineellisilla kappaleilla mutta pohjimmiltaan sekä logiikka että matematiikka ovat luovan mielikuvituksen tuotteita ja jos se luova mielikuvitus on ristiriidatonta niin se kuvitellun asian voi toteuttaa ns. aineellisella tasolla kuten voimme huomata varsinkin kulttuurin, yhteiskunnan, tekniikan ja taiteen tasolla. Kuvitellun asian pitää olla myös yhteensopiva kokonaisuuden (kokonaissysteemi) kanssa vaikka toisaalta se kokonaisuuskin vähitellen muuttuu samalla kuin sen osasysteemit muuttuvat ja saavuttavat ns. kriittisen massan.

        Intentio ristiriidaton mielikuvitus= todellisuus

        Belisario


      • Anonyymi
        Anonyymi kirjoitti:

        "Joudut keksimään maailman, missä logiikka ei päde. "

        Voimme kuvitella sellaisia maailmoja jotka ovat loogisesti ristiriidattomia. Voimme myös kehitää sellaisen logiikan joka ei ole kaavamaisesti 2-arvoinen, binäärinen. joko tai logiikka vaan esim. jonkinlainen sekä-että logiikka kuten esim. kvanttifysikkan superpositiot.

        "Kokonaisluku aritmetiikka voidaan johtaa joukko-opista eli joudut keksimään maailman, missä joukko-opin aksioomat ovat epätosia."

        Ei ole mielekästä suorittaa kokonaislukuoperaatioita keskenään hyvin erilaisten asioiden välillä. Vain samaan luokitteluun liittyviä asioita on hyödyllistä laskea yhteen. vähentää toisistaan jne. Esim. omenoiden, rahojen, apinoiden ja kolmioiden yhteenlaskeminen keskenään ei ole kovin mielekästä. Sen lisäksi että yhteenlaskettavien asioiden pitää olla samaan joukkoon kuuluvia niin niiden pitää olla myös selkeästi erillisiä.

        Kokonaisluvut liittyvät lähinnä kiinteisiin kappaleisiin. Kaasumaiset ja nestemäiset oliot ovat hankalampia yhteenlaskettavia varsinkin jos hahmottaa ne sillä tavalla että ei "riko" niiden asioiden tiettyyn kokonaisuuteen liittyviä kemiallisia ominaisuuksia pilkkomalla ne protoneiksi, neutroneiksi ja elektroneiksi tai kvarkeiksi. Esim, molekyylit ja atomit ovat pienimpiä yksiköitä joilla on juuri tietyt "yksilölliset" kemialliset ominaisuudet.

        Matematiikka siis edellyttää jonkinlaista luokittelua ja identiteetin määrittelyä ennen kuin voidaan suorittaa mielekkäästi laskutoimituksia ja ne luokittelut ja määrittelyt ovat pohjimmiltaan metafysiikkaan ja filosofisiin oletuksiin liittyviä eli siis pojhimmiltaan vain tapoja hahmottaa koettua todellisuutta.

        Loogiset totuudet eivät tietenkään muutu koska niillä ei ole aikaan liittyvää komponenttia kuten aineellisilla kappaleilla mutta pohjimmiltaan sekä logiikka että matematiikka ovat luovan mielikuvituksen tuotteita ja jos se luova mielikuvitus on ristiriidatonta niin se kuvitellun asian voi toteuttaa ns. aineellisella tasolla kuten voimme huomata varsinkin kulttuurin, yhteiskunnan, tekniikan ja taiteen tasolla. Kuvitellun asian pitää olla myös yhteensopiva kokonaisuuden (kokonaissysteemi) kanssa vaikka toisaalta se kokonaisuuskin vähitellen muuttuu samalla kuin sen osasysteemit muuttuvat ja saavuttavat ns. kriittisen massan.

        Intentio ristiriidaton mielikuvitus= todellisuus

        Belisario

        "Loogiset totuudet eivät tietenkään muutu koska..."

        Matemaattiset totuudet ovat loogisia totuuksia.


      • Anonyymi
        Anonyymi kirjoitti:

        "Loogiset totuudet eivät tietenkään muutu koska..."

        Matemaattiset totuudet ovat loogisia totuuksia.

        "Matemaattiset totuudet ovat loogisia totuuksia."

        Samoin kuin kaikki muutkin sisäisesti ristiriidattomat ajatukset ja ideajärjestelmät. Matematiikkaan ja logiikkaan ei liity mitään mystiikkaa. Ne ovat vain todellisuuden hahmottamisen työkaluja ja abstraktioita joita voi enemmän tai vähemmän menestyksellisesti soveltaa oman kokemusmaailmansa jäsentämiseen. Itse asiassa matematiikan ja logiikan soveltaminen on aika mekaanista ja automaattista ja verattain kömpelöä ja hidasta.

        Fysiikka ei ole matemaattista sen takia että tiedetään niin paljon vaan sen takia että tiedetään niin äärettömän vähän ja siitä vähästä sitten yritetään pusertaa irti mahdollisimman paljon loogisia seurauksia joilla saattaa olla jonkintasoinen vastine todellisuudessa mutta välttämättä ei ole.

        Voi myös matematiikan ja logiikan avulla tuottaa loogisesti koherenttia fiktiota tai tarinaa ja on huomattavasti epätodennäköisempää että se matemaattinen kuvaus olisi oikeasti toimiva analogia tai vertauskuva suhteessa siihen todellisuuteen vaikka voi siltä jonkin aikaa näyttää.

        Belisario


      • Anonyymi kirjoitti:

        "Joudut keksimään maailman, missä logiikka ei päde. "

        Voimme kuvitella sellaisia maailmoja jotka ovat loogisesti ristiriidattomia. Voimme myös kehitää sellaisen logiikan joka ei ole kaavamaisesti 2-arvoinen, binäärinen. joko tai logiikka vaan esim. jonkinlainen sekä-että logiikka kuten esim. kvanttifysikkan superpositiot.

        "Kokonaisluku aritmetiikka voidaan johtaa joukko-opista eli joudut keksimään maailman, missä joukko-opin aksioomat ovat epätosia."

        Ei ole mielekästä suorittaa kokonaislukuoperaatioita keskenään hyvin erilaisten asioiden välillä. Vain samaan luokitteluun liittyviä asioita on hyödyllistä laskea yhteen. vähentää toisistaan jne. Esim. omenoiden, rahojen, apinoiden ja kolmioiden yhteenlaskeminen keskenään ei ole kovin mielekästä. Sen lisäksi että yhteenlaskettavien asioiden pitää olla samaan joukkoon kuuluvia niin niiden pitää olla myös selkeästi erillisiä.

        Kokonaisluvut liittyvät lähinnä kiinteisiin kappaleisiin. Kaasumaiset ja nestemäiset oliot ovat hankalampia yhteenlaskettavia varsinkin jos hahmottaa ne sillä tavalla että ei "riko" niiden asioiden tiettyyn kokonaisuuteen liittyviä kemiallisia ominaisuuksia pilkkomalla ne protoneiksi, neutroneiksi ja elektroneiksi tai kvarkeiksi. Esim, molekyylit ja atomit ovat pienimpiä yksiköitä joilla on juuri tietyt "yksilölliset" kemialliset ominaisuudet.

        Matematiikka siis edellyttää jonkinlaista luokittelua ja identiteetin määrittelyä ennen kuin voidaan suorittaa mielekkäästi laskutoimituksia ja ne luokittelut ja määrittelyt ovat pohjimmiltaan metafysiikkaan ja filosofisiin oletuksiin liittyviä eli siis pojhimmiltaan vain tapoja hahmottaa koettua todellisuutta.

        Loogiset totuudet eivät tietenkään muutu koska niillä ei ole aikaan liittyvää komponenttia kuten aineellisilla kappaleilla mutta pohjimmiltaan sekä logiikka että matematiikka ovat luovan mielikuvituksen tuotteita ja jos se luova mielikuvitus on ristiriidatonta niin se kuvitellun asian voi toteuttaa ns. aineellisella tasolla kuten voimme huomata varsinkin kulttuurin, yhteiskunnan, tekniikan ja taiteen tasolla. Kuvitellun asian pitää olla myös yhteensopiva kokonaisuuden (kokonaissysteemi) kanssa vaikka toisaalta se kokonaisuuskin vähitellen muuttuu samalla kuin sen osasysteemit muuttuvat ja saavuttavat ns. kriittisen massan.

        Intentio ristiriidaton mielikuvitus= todellisuus

        Belisario

        Edelleenkään en ymmärrä, miksi etsit fyysisestä maailmasta matematiikan sovelluksia, kun olemme selvästikin samaa mieltä, ettei matemaattisen logiikan pätevyys liity fyysiseen maailmaan millään tavalla.

        Matemaattinen logiikka on aina olemassa muuttumattomana - kuten totesit. Se ei siis ole mielen tuotos, vaikka mieli voikin ymmärtää sen ja käyttää sitä. Logiikan olemassa olo on ajattelun edellytys. Ilman logiikkaa ajatuksesi olisivat vain yhtä kokemuksellista sekamelskaa, missä ei ole järkeä.


      • Anonyymi kirjoitti:

        "Matemaattiset totuudet ovat loogisia totuuksia."

        Samoin kuin kaikki muutkin sisäisesti ristiriidattomat ajatukset ja ideajärjestelmät. Matematiikkaan ja logiikkaan ei liity mitään mystiikkaa. Ne ovat vain todellisuuden hahmottamisen työkaluja ja abstraktioita joita voi enemmän tai vähemmän menestyksellisesti soveltaa oman kokemusmaailmansa jäsentämiseen. Itse asiassa matematiikan ja logiikan soveltaminen on aika mekaanista ja automaattista ja verattain kömpelöä ja hidasta.

        Fysiikka ei ole matemaattista sen takia että tiedetään niin paljon vaan sen takia että tiedetään niin äärettömän vähän ja siitä vähästä sitten yritetään pusertaa irti mahdollisimman paljon loogisia seurauksia joilla saattaa olla jonkintasoinen vastine todellisuudessa mutta välttämättä ei ole.

        Voi myös matematiikan ja logiikan avulla tuottaa loogisesti koherenttia fiktiota tai tarinaa ja on huomattavasti epätodennäköisempää että se matemaattinen kuvaus olisi oikeasti toimiva analogia tai vertauskuva suhteessa siihen todellisuuteen vaikka voi siltä jonkin aikaa näyttää.

        Belisario

        ”Fysiikka ei ole matemaattista sen takia että tiedetään niin paljon vaan sen takia että tiedetään niin äärettömän vähän”

        Matematiikka menestyskulku fysikaalisen maailman mallintamisessa on musertava. Tämä ei taatusti tule muuttumaan tiedon myötä. Se, miten hyvin tietyt luonnonvakiot kuten elektronin magneettisen momentin vakio Gs voidaan selittää matemaattisten kvanttifysiikanmallien kautta on ajoittain jopa hämmästyttävä. Dirac:han katsoi, että Gs=2, mutta kokeet osoittivat sen poikkeavan tästä noin parin tuhannesosan verran. Nykyinen kvanttifysiikan malli on yhtenevä mittauksien kanssa aina 10 (!) desimaalin tarkkuuteen. Näyttää siis, että mittauksien kautta määritetyille luonnonvakioillekin on olemassa syynsä ja kaikki nousee matemaattisten totuuksien pohjalta kunhan vain ymmärrämme kaiken alla piilevän mekanismin.


      • Anonyymi
        Järkisyitä kirjoitti:

        Edelleenkään en ymmärrä, miksi etsit fyysisestä maailmasta matematiikan sovelluksia, kun olemme selvästikin samaa mieltä, ettei matemaattisen logiikan pätevyys liity fyysiseen maailmaan millään tavalla.

        Matemaattinen logiikka on aina olemassa muuttumattomana - kuten totesit. Se ei siis ole mielen tuotos, vaikka mieli voikin ymmärtää sen ja käyttää sitä. Logiikan olemassa olo on ajattelun edellytys. Ilman logiikkaa ajatuksesi olisivat vain yhtä kokemuksellista sekamelskaa, missä ei ole järkeä.

        "Edelleenkään en ymmärrä, miksi etsit fyysisestä maailmasta matematiikan sovelluksia, kun olemme selvästikin samaa mieltä, ettei matemaattisen logiikan pätevyys liity fyysiseen maailmaan millään tavalla."

        Siksi että mielestäni kaikki satunnaisuudesta poikkeavat ilmiöt luonnossa ovat jonkinlaisen tietoisen älyn tuottamia (joko ihmisten tai joidenkin muiden). Kyllä ihmiskunnan käyttämän matematiikan pätevyys liittyy luontoon koska se käyttämämme matematiikka on nimenomaan kehitetty toimimaan tuntemassamme luonnossa joko teorioiden tai tekniikan muodossa.

        "Se ei siis ole mielen tuotos, vaikka mieli voikin ymmärtää sen ja käyttää sitä. "

        Se nimenomaan aina on jonkinlaisen mielen tuotos samoin kuin kaikki luonto jonka me tunnemme.


        "Logiikan olemassa olo on ajattelun edellytys. Ilman logiikkaa ajatuksesi olisivat vain yhtä kokemuksellista sekamelskaa, missä ei ole järkeä."

        Itse käytän lähinnä intuitiota ja keksin sitten sen logiikan jälkeenpäin samalla kun yritän muuntaa sitä intuitiota käsitteelliseen kielellisesti ilmaistavissa olevaan muotoon.

        Se kokemuksellinen sekamelska johtuu lähinnä siitä että varsinkin nykyaikana ihmisten mieli toimii epänormaalilla ja "rikkinäisellä" tavalla. Ns. luonnonkansoilla tätä kokemuksellisuuden rikkinäisyyttä esiintyy paljon vähemmän koska luonnonoloissa on elintärkeää kehittää tasapainoinen mieli jos aikoo säilyä hengissä. Länsimaisessa yhteiskunnassa sekoilu ei ole läheskään niin fataalia kuin metsästäjä-keräilijöillä.

        Belisario


      • Anonyymi
        Järkisyitä kirjoitti:

        ”Fysiikka ei ole matemaattista sen takia että tiedetään niin paljon vaan sen takia että tiedetään niin äärettömän vähän”

        Matematiikka menestyskulku fysikaalisen maailman mallintamisessa on musertava. Tämä ei taatusti tule muuttumaan tiedon myötä. Se, miten hyvin tietyt luonnonvakiot kuten elektronin magneettisen momentin vakio Gs voidaan selittää matemaattisten kvanttifysiikanmallien kautta on ajoittain jopa hämmästyttävä. Dirac:han katsoi, että Gs=2, mutta kokeet osoittivat sen poikkeavan tästä noin parin tuhannesosan verran. Nykyinen kvanttifysiikan malli on yhtenevä mittauksien kanssa aina 10 (!) desimaalin tarkkuuteen. Näyttää siis, että mittauksien kautta määritetyille luonnonvakioillekin on olemassa syynsä ja kaikki nousee matemaattisten totuuksien pohjalta kunhan vain ymmärrämme kaiken alla piilevän mekanismin.

        "Matematiikka menestyskulku fysikaalisen maailman mallintamisessa on musertava. Tämä ei taatusti tule muuttumaan tiedon myötä. "

        On olemassa menestystä ja näennäismenestystä. Newtonin ja Leibnizin kehottämä matematiikka oli hyödyllistä varsinkin arkielämän tasolla. Luonnosta ei koskaan löydy täydellisiä geometrisia kuvioita (esim. täydellistä ympyrää tai neliötä).

        Nykyisessä fysiikkatieteessä on kuljettu matematiikka edellä jo yli 100 vuotta ja epätoivon vimmalla sitten etsitty äärimmäisen suurta tarkkuutta edellyttävien matemaattisten mallien ennusteiden vastineita luonnossa (esim. LIGO) .

        Tämä mielestäni täysin väärä tapa toimia. Tieteen tehtävä on tutkia selkeästi havaittavissa olevia ilmiöitä eikä yrittää etsiä sopivia ilmiöitä teor. fysiikan tai kosmologian kaavoille. Matematiikka ja matemaattiset simulaatiot eivät voi koskaan korvata havaintoja.

        "Se, miten hyvin tietyt luonnonvakiot kuten elektronin magneettisen momentin vakio Gs voidaan selittää matemaattisten kvanttifysiikanmallien kautta on ajoittain jopa hämmästyttävä. "

        En ole tuohon perehtynyt mutta itselleni tulee lähinnä mieleen se että jo pelkästään se että ilmiötä tulkitaan jonkin matemaattisen mallin ilmentymänä helposti tuottaa illuusion
        empiirisestä tiedosta vaikka kyse on pelkästään siitä että tietty tulos seuraa jo pelkästään loogisena seurauksena sen mallin oletuksista ja kun ne ns. havainnot ko, asiasta tulkitaan myös sen matemaattisen mallin puitteissa niin ne suurta mittaustarkkuutta vaativat kokeiden tulokset tulkitaan sen teorian puitteissa eli syntyy eräänlainen kehäpäätelmä. Lisäksi PEAR LABS:n kokeissa todettiin että intention avulla voi muuttaa erilaisten satunnaisgeneraattorien tuottamia tuloksia haluttuun suuntaan niin ainakin minulla herää epäilys että tutkijoiden harras intentio jo sellaisenaan voi tuottaa havaintoja jotka ovat heidän itsensä aiheuttamia ja heidän kehittämäänsä mallia tukevia.

        Suurin osa nykyisestä teor. fysiikasta ja kaikesta muustakin pitkälle matematisoidusta tieteestä voi olla pielessä. Kyseessä voi olla placebo-efekti mikä tarkoittaa myös sitä että nykyinen tieteenfilosofia (esim. Popper) voi olla hyvin pitkälle pelkkää haaveilua ja todellisuudelle vierasta.

        Belisario


      • Anonyymi
        Järkisyitä kirjoitti:

        Olen tässä kyllä Firdawsin kanssa samaa mieltä. Matematiikan abstraktien käsitteiden olemassa olo ei edellytä fyysistä todellisuutta. Tässä 'olemassa olo' ei ole fyysistä eikä edes henkistä/kokemuksellista vaan sitä, että kyseinen totuus pätee aina.

        Nämä (matemaattiset) totuudet ovat pakosti kuten ne ovat - ne eivät voi olla mitenkään muuten. Ne ovat perimmäisiä totuuksia.

        Tälläisien perustavaa laatua olevien totuuksien olemassa olo on mielenkiintoista, koska niiden kautta voi olla mahdollista hakea vastausta filosofeja askarruttaneeseen kysymykseen: "Miksi on olemassa jotakin sen sijaan, ettei olisi olemassa mitään?"

        Nimimerkki: Jumalan-kumous (ei-rekisteröitynyt professori)

        Jatkanpa vielä tästä minäkin, koska keskustelu näyttääkin jatkuneen. Kaikenkaikkiaan, tämä on kyllä hyvin mielenkiintoinen, relevantti, filosofinen ja ehkä hiukan haastavakin aihe ja tällä porukalla siihen saadaan kyllä hyvin erilaisia näkökulmia, mutta siitä johtuen ehkä tuskin kovinkaan paljoa konsensusta; näin ennakoin: veni, vidi, vici. :D

        Järkisyitä: "Olen tässä kyllä Firdawsin kanssa samaa mieltä. Matematiikan abstraktien käsitteiden olemassa olo ei edellytä fyysistä todellisuutta. Tässä 'olemassa olo' ei ole fyysistä eikä edes henkistä/kokemuksellista vaan sitä, että kyseinen totuus pätee aina. Nämä (matemaattiset) totuudet ovat pakosti kuten ne ovat - ne eivät voi olla mitenkään muuten. Ne ovat perimmäisiä totuuksia."

        Vaan, kun eivät olekaan, koska vaikka *jos* ne olisivatkin "pakosti" *aina ja kaikkialla* jotakin, minkä hypoteesin paikkansapitävyyden; mahdollisesti varsin haasteellisen, todistamisen osalta, todistustaakka on tietenkin kyseisen oletuksen tekijällä, niin ne ovat "pakosti" sitä, mitä ne sitten ovatkin, koska niihin on *jokin syy* ja *reaalinen kausaalinen syy* voi olla vain *fysikaalinen*, ei matemaattinen tai looginen.

        Toistan, että koskaan ei olla *havaittu*, että matemaattinen käsite tai matemaattinen totuus tai mikään muukaan *kuvaus* *sinällään* olisi tuottanut mitään reaalisia vaikutuksia ja jos/kun kuitenkin pyritään lisäksi erityisesti korostamaan sitä, että pyritään tarkastelemaan todellisuutta tieteellisen menetelmän mukaisesti; kuten tässä ketjussa on tehty, niin seurauksena on selvä ristiriita, koska tieteellinen menetelmä perustuu nimenomaan *reaalisia* asioita koskeviin havaintoihin, eikä määritelmällisiin ja/tai edes empiiris-induktiivisesti pääteltyihinkään väitelauseiden totuusarvoihin.

        Eli, kuvaukset ovat "korkeintaan" ainakin *suhteellisen* paikkansapitäviä fysikaalisten rakenteiden/ilmiöiden kuvauksia; jostain näkökulmasta, eivätkä suinkaan mitään "perimmäisiä totuuksia". En ylipäätään usko, että mitään "perimmäisiä totuuksia" ylipäätään kyetään koskaan tietämäänkään; sikäli kuin niitä ylipäätään on olemassakaan, ainakaan missään mielekkäässä ja/tai ristiriidattomassa merkityksessä.

        Esim. Max Tegmark varmaankin on näistä asioista eri mieltä kanssani ainakin joiltakin osin, vaikka hänkin on multiversumi-näkökantaa oikeana todellisuuden kuvauksena pitävä, mutta hänen; varsin radikaalit, näkemyksensä siitä, että kaikki *olisi* perimmiltään matematiikkaa, ovat tuottaneet myös monenlaista; ja mielestäni monilta osin hyvin perusteltua ja vakuuttavaa kritiikkiä; jota kritiikkiä on kuvattu mm. oheisella, häntä kuvaavalla Wikipedia-sivulla. Eli; kuten aiemmin jo totesin, fysikaalista todellisuutta voidaan *kuvata* matemaattisesti ja se tietysti mahdollistaa myös esim. logiikan käytön kuvauksissa, mutta koska yksikään matemaattinen totuus tai käsite ei saa aikaan *mitään* kausaalisia vaikutuksia, niin ne eivät voi olla "perimmäisiä totuuksia".
        https://en.wikipedia.org/wiki/Max_Tegmark

        "Tälläisien perustavaa laatua olevien totuuksien olemassa olo on mielenkiintoista, koska niiden kautta voi olla mahdollista hakea vastausta filosofeja askarruttaneeseen kysymykseen: "Miksi on olemassa jotakin sen sijaan, ettei olisi olemassa mitään?""

        Olen samaa mieltä siitä, että mm. tuo kysymys on hyvin mielenkiintoinen ja muistan itse pohtineeni ensimmäisen kerran sitä jo noin 6-vuotiaana; täysin ilman *erityistä tuohon suuntaavaa ohjausta*; muilta *ihmisiltä*. Pohdin sitä, että olisiko mielekästä esittää kysymys koskien sitä, että mitä se merkitsisi *kokemuksellisesti*, jos *koskaan* ei olisi ollut *mitään* ja muodostettuani tuon kysymyksen, niin aivoiltani vei tuolloin vain noin sekunnin tuottaa vastaus tuohon kysymykseen. :D

        Kokemuksellisesti tuntemuksena oli se, että aivojen läpi kulki ikään kuin oikosulku ja ymmärsin heti, että koska olisi *fyysisesti* mahdotonta esittää kysymyksiä/vastauksia, jos aivojakaan ei olisi olemassa, niin aivojen prosessit olivat "jakautuneet kahtia"; prosessoimaan erikseen noita kahta vaihtoehtoa ja kun ne noin sekuntia myöhemmin olivat sitten saaneet työnsä päätökseen ja tulokset yhdistyivät; epäilemättä jonkinlaisessa automatisoidussa deterministisessä prosessissa, niin seurauksena oli *mitä kummallisin* tuntemus; ikään kuin nuo prosessit olisivat kumonneet toisensa.

        Jos olisin tuolloin ollut ns. "indoktrinoitu" esim. "platonistiseen ideaoppiin"; edes implisiittisesti, niin minulta olisi todennäköisesti vienyt enemmän aikaa tajuta se, että looginen päättely tosiaan on *fyysinen* prosessi ja totuusarvotkin ovat pohjimmiltaan nimenomaan fyysisiä asioita, jotka jotkut sitten *tulkitsevat* joksikin muuksi...


      • Anonyymi
        Järkisyitä kirjoitti:

        Ymmärrän mielestäni kaiken, mitä 'Jumalan-kumous' kirjoitti, mutta silti en koe, että hän ottaa kantaa Firdawsin-argumentteihin. On kuin puhuisitte kahdesta eri asiasta.

        Tietenkään matemaattiset totuudet eivät ole fyysisiä tai edes kokemuksellisia asioita eli tässä mielessä niitä ei ole olemassa. Idealistit voisivat sanoa, että jopa fyysistä maailmaa ei ole olemassa ellei joku ole sitä kokemassa. Kenties tässä takerrumme siihen, mitä ymmärrämme olemassa olon tarkoittavan.

        Lakkaako fyysinen todellisuus olemasta silloin, jos kukaan ei koe sitä tietoisesti? Entä lakkaako matemaattinen totuus olemasta silloin, kun sitä ei ajattele kukaan? Miten lähdemme todistamaan, että joku asia on silloinkin, kun emme havaitse sitä?

        Sanotaan, että näet puun. Suljet silmät. Avaat silmät ja näet puun edelleen. Eikö tästä ole johdonmukaista olettaa, että puu on olemassa riippumatta siitä, että sitä havainnoidaan koko ajan?

        Sovelletaan samaa matemaattiseen totuuteen. Sekin pysyy muuttumattomana täysin riippumatta siitä havainnoiko sitä joku. Jokainen tietoinen olento havaitsee sen samanlaisena. Täten katsoisin, että se on abstraktina käsitteenä aina olemassa. Se ei katoa ja jokainen tietoinen olento kykenee havaitsemaan sen olemassa olon aina ajatellessaan sitä.

        Nimimerkki: Jumalan-kumous (ei-rekisteröitynyt professori)

        Järkisyitä: "Tietenkään matemaattiset totuudet eivät ole fyysisiä tai edes kokemuksellisia asioita eli tässä mielessä niitä ei ole olemassa. Idealistit voisivat sanoa, että jopa fyysistä maailmaa ei ole olemassa ellei joku ole sitä kokemassa. Kenties tässä takerrumme siihen, mitä ymmärrämme olemassa olon tarkoittavan."

        Tietenkin totuudet; kuten kaikki muutkin asiat; ovat *vähintään myös* fyysisiä asioita, vaikka tämä näkemys ei taatusti olekaan suosittu; mikä taas on sellainen seikka, jolle ei yleensäkään kannata antaa paljoa painoarvoa, siinä tapauksessa siis, että on oikeassa; kuten tietysti lähes kaikki, ainakin täällä, tietysti ovat, ainakin omasta mielestään...

        Esim. idealistit eivät kykene tuottamaan tieteellistä lisäarvoa edes pohdittaessa sitä, että mikä tuottaa ne laskennalliset resurssit, jotka *mahdollistavat* esim. havaitsemisen, joka kuitenkin tietenkin on; heidän ilmeisesti erittäin tärkeänä pitämänsä tietoisuudenkin, kannalta keskeistä.

        "Lakkaako fyysinen todellisuus olemasta silloin, jos kukaan ei koe sitä tietoisesti? Entä lakkaako matemaattinen totuus olemasta silloin, kun sitä ei ajattele kukaan? Miten lähdemme todistamaan, että joku asia on silloinkin, kun emme havaitse sitä? Sanotaan, että näet puun. Suljet silmät. Avaat silmät ja näet puun edelleen. Eikö tästä ole johdonmukaista olettaa, että puu on olemassa riippumatta siitä, että sitä havainnoidaan koko ajan?"

        Fyysisen todellisuuden olemassaolo ei edellytä havaitsemista, mutta matemaattisen totuuden; ja tietysti muidenkin *todellisuuden kuvausten* olemassaolo, edellyttää fyysistä todellisuutta ja nimenomaan sellaista fyysistä todellisuutta, joka mahdollistaa kyseisen kuvauksen. Olen siis samaa mieltä siitä, että fysikaaliset asiat ovat olemassa havainnoistamme riippumatta ja syy tuohon on se, että niillä on; tai ne ovat, *energiaa* ja energia on eräs edellytys sille, että voidaan ylipäätään edes kyetä kuvittelemaan mitään kuvauksia jne. Fyysisen todellisuuden; nimenomaan *vain ja ainoastaan*, reaalinen, olemassaolo, on luonnollisesti hyvin yhteensopiva; mm. ymmärryksen mahdollistajana käyttökelpoisimpana pitämäni vaihtoehdon, eli kvanttimekaniikan monimaailmatulkinnan, kanssa...

        Tietenkin; toisaalta, kannaltamme relevantit asiat ovat sellaisia, että kykenemme ne havaitsemaan tai ainakin tietämään jotakin niihin liittyvää, esim. sitä kautta, että niitä on tehty havaintoja joidenkin muiden toimesta, tai joidenkin mittalaitteiden avulla tai havaitsemme niiden vaikutuksi tai kykenemme päättelemään niistä jotakin muun käytettävissämme olevan informaation perusteella; käyttäen loogista päättelyä jne.

        Tästä kaikeasta ei siis kuitenkaan tietenkään seuraa, että "matemaattiset totuudet" olisivat olemassa riippumatta fysikaalisista edellytyksistänsä, koska siis niillä *ei* ole energiaa ja jos taas vedämme yhtäläisyysmerkin matemaattisten totuuksien ja niiden fysikaalisten edellytysten välille; eli "implisiittisesti annamme niille energiaa", niin kyseessä on jonkinlainen reifikaatio; joka on argumentointivirhe, ja tässä tapauksessa todellisuuden kuvauksen kannalta hyödytön samaistus, koska todellisuuden ns. "resoluutioltaan käyttökelpoinen" kuvaus nimenomaan edellyttää mm. edellytysten ja seurausten erottamista toisistaan ja muuten vaarana on myös "psykologisella kielellä" ilmaisten: "vaipuminen ns. Jungilaiseen pleroomaan" tuolta osin, josta konseptista ehkä esim. Belisario haluaa kertoa enemmänkin. :D


      • Anonyymi
        Järkisyitä kirjoitti:

        Ymmärrän mielestäni kaiken, mitä 'Jumalan-kumous' kirjoitti, mutta silti en koe, että hän ottaa kantaa Firdawsin-argumentteihin. On kuin puhuisitte kahdesta eri asiasta.

        Tietenkään matemaattiset totuudet eivät ole fyysisiä tai edes kokemuksellisia asioita eli tässä mielessä niitä ei ole olemassa. Idealistit voisivat sanoa, että jopa fyysistä maailmaa ei ole olemassa ellei joku ole sitä kokemassa. Kenties tässä takerrumme siihen, mitä ymmärrämme olemassa olon tarkoittavan.

        Lakkaako fyysinen todellisuus olemasta silloin, jos kukaan ei koe sitä tietoisesti? Entä lakkaako matemaattinen totuus olemasta silloin, kun sitä ei ajattele kukaan? Miten lähdemme todistamaan, että joku asia on silloinkin, kun emme havaitse sitä?

        Sanotaan, että näet puun. Suljet silmät. Avaat silmät ja näet puun edelleen. Eikö tästä ole johdonmukaista olettaa, että puu on olemassa riippumatta siitä, että sitä havainnoidaan koko ajan?

        Sovelletaan samaa matemaattiseen totuuteen. Sekin pysyy muuttumattomana täysin riippumatta siitä havainnoiko sitä joku. Jokainen tietoinen olento havaitsee sen samanlaisena. Täten katsoisin, että se on abstraktina käsitteenä aina olemassa. Se ei katoa ja jokainen tietoinen olento kykenee havaitsemaan sen olemassa olon aina ajatellessaan sitä.

        Nimimerkki: Jumalan-kumous (ei-rekisteröitynyt professori)

        Järkisyitä: "Sovelletaan samaa matemaattiseen totuuteen. Sekin pysyy muuttumattomana täysin riippumatta siitä havainnoiko sitä joku. Jokainen tietoinen olento havaitsee sen samanlaisena. Täten katsoisin, että se on abstraktina käsitteenä aina olemassa. Se ei katoa ja jokainen tietoinen olento kykenee havaitsemaan sen olemassa olon aina ajatellessaan sitä."

        Matemaattiset totuudet; kuten *kaikki muutkin olemassaolevat asiat*, edellyttävät tiettyjä asioita ja jos noita edellytyksiä ei ole olemassa, niin myöskään matemaattisia totuuksia ei ole olemassa. Olen samoilla linjoilla Belisario:n kanssa siltä osin, että olemassa ei ole mitään "perimmäisiä totuuksia", vaan vain todellisuuden "puutteellisia kuvauksia", koska todellisuus on *kokonaisuus* ja kokonaisuuden totuudellinen kuvaus ei ole mahdollista osajärjestelmistä käsin; eikä mitenkään muutenkaan...

        Kyse ei muutenkaan ole pelkästään siitä, että niitä; eli esim. "matemaattisia totuuksia", ei havaittaisi, vaan siitä, että niitä ei ylipäätään olisi edes olemassakaan, jos niiden edellytykset eivät olisi olemassa. Näkökantani on ns. materialistis-empiiris-holistis-deterministinen, eli en usko, että todellisuudesta oikeasti voidaan ns. ottaa osia pois tai lisätä siihen osia, eli matemaattiset totuudet; kuten muutkin asiat, edellyttävät sitä, että todellisuus on se mikä se on, eikä jotain muuta ja havaintojen mukaan todellisuus on fysikaalinen; tai ainakin todellisuuden kuvaaminen; sen ymmärrystä ja ainakin näennäistä hallittavuutta lisäävällä tavalla, on mm. luonnontieteiden edistymisen perusteella, selkeästi sellainen asia, jossa nimenomaan fysikaalisilla rakenteilla ja prosesseilla on keskeinen merkitys.

        Mitä taas tulee siihen, että *miksi* matematiikka on osoittautunut niin onnistuneeksi *kuvaustavaksi*; erityisesti fysikaalisten asioiden osalta; eli siis ns. Wignerin dilemmaan, niin vastaukseni on, että kykenemme havaitsemaan vain sellaisen universumin; monien reaalisten universumeiden joukosta, joka mahdollistaa havaitsemisen; ns. antrooppisen periaatteen ns. heikon muodon mukaisesti, eli emme havaitse suinkaan satunnaista universumia, vaan nimenomaan sellaisen universumin, jonka havaitsemisen kannalta relevantit fysikaaliset attribuutit korreloivat erittäin voimakkaasti sen kanssa, että kyetään muodostamaan tehokkaasti prosessoitavia todellisuuden kuvauksia ja tietojenkäsittelyn kannalta optimaalisesti kompressoidut kuvaukset ovat nimenomaan matemaattisia.

        On *todella* nurinkurista, jos multiversumin; tai jos ei usko sen olemassaoloon, niin vaihtoehtoisesti jonkin "kuvitteellisen todellisuuden", tarjoamilla fysikaalisilla laskennallisilla resursseilla, tuotamme todellisuuden joistakin omasta mielestämme relevanteista aspekteista; jotka aspektit olemme valinneet perustuen *ilmeisen rajoitteisiin* kognitiivisiin kykyihimme, optimaalisesti tai ainakin kognitiivisten kykyjemme avulla lähes optimaalisiksi arvioimiamme kuvauksia ja toteamme, että nuo kuvaukset ovatkin olemassa *riippumatta* noista laskennallisista resursseista; jotka ne omalta osaltaan ovat mahdollistaneet ja vieläpä, että ne ovat "lopullisia ja aina ja kaikkialla päteviä totuuksia".

        "Fanittamani" Einstein totesi joskus mm. jotensakin näin ikään, että: "Vain universumi ja ihmisen tyhmyys ovat äärettömiä - enkä ole siitä universumin äärettömyydestäkään varma."; kuten en ole minäkään ja en siis ole ollenkaan varma siitä, että ihminen kykenisi todistamaan, että nuo em. kuvaukset olisivat "lopullisia ja aina ja kaikkialla päteviä totuuksia". :D


      • Anonyymi
        Anonyymi kirjoitti:

        "Nämä (matemaattiset) totuudet ovat pakosti kuten ne ovat - ne eivät voi olla mitenkään muuten. Ne ovat perimmäisiä totuuksia."

        Niin ne tuntuvat pätevän ainakin tässä meidän mahdollisessa ja aktualisoituneessa maailmassa riittävän hyvin mutta pystyn aika helposti kuvittelemaan sellaisenkin mahdollisen maailman jossa tuntemamme matematiikka ei toimi juuri lainkaan.

        Maailma jossa ei ole mitään selkeästi erillisiä luokiteltavissa olevia olioita eikä mitään selkeitä säännönmukaisuuksia on maailma jossa tuntemallamme matematiikalla ei ole juurikaan käyttöä. Samoin maailma jossa on pelkkiä laatuja mutta ei lainkaan määriä eikä mitään selkeästi vertailukelpoisia asioita jää matematiikan ulottumattomiin.

        Matematiikan näennäinen "totuus" tarkoittaa vain sitä että matemaattinen aksiomaattinen järjestelmä on loogisesti ristiriidaton mutta ei sen silti tarvitse vastata mitään fysikaalisesti olemassaolevaa asiaa.

        Belisario

        ps. hyvät syksyt ja talvet myös Firwdasille ja Jumalan kumoukselle. Veikkaan kyllä että Firwdasi palaa jonkin ajan kuluttua takaisin. Erimielisten kanssa väittely vaatii aika paljon energiaa ja hyviä hermoja sekä kestävyyttä - usein enemmän kuin mikään fyysinen ponnistelu.

        Nimimerkki: Jumalan-kumous (ei-rekisteröitynyt professori)

        Belisario: "Matematiikan näennäinen "totuus" tarkoittaa vain sitä että matemaattinen aksiomaattinen järjestelmä on loogisesti ristiriidaton mutta ei sen silti tarvitse vastata mitään fysikaalisesti olemassaolevaa asiaa."

        Matematiikka on aksiomaattinen tiede, mutta Kurt Gödel:in todistaman ja hänen mukaansa nimetyn epätäydellisyysteoreeman mukaanhan matematiikalle ei voida kyetä luomaan edes sellaista *aksiomaattista perustaa*, jolla kyettäisiin kuvaamaan *edes* matematiikka itsekään ("eo ipso") *ristiriidattomasti*. Itseasiassa, matematiikalla ei kyetä kuvaamaan ristiriidattomasti edes *perus-aritmetiikkaakaan*...
        https://en.wikipedia.org/wiki/Gödel's_incompleteness_theorems

        "Erimielisten kanssa väittely vaatii aika paljon energiaa ja hyviä hermoja sekä kestävyyttä - usein enemmän kuin mikään fyysinen ponnistelu."

        Väittelyhän *on* fyysistä ponnistelua; kuten kaikki muukin ponnistelu ja aivot kuluttavat tutkimusten mukaan keskimäärin n. 20-25% ihmisen energiasta ja joidenkin täällä esiintyvien keskustelijoiden tapauksessa siis ehkä hyvinkin jopa 30-35%; silloin kun erimielisyys on ns. "päällä". :D


      • Anonyymi
        Järkisyitä kirjoitti:

        "Matematiikan näennäinen "totuus" tarkoittaa vain sitä että matemaattinen aksiomaattinen järjestelmä on loogisesti ristiriidaton mutta ei sen silti tarvitse vastata mitään fysikaalisesti olemassaolevaa asiaa."

        Tismalleen! Sitähän minä yritän sanoa. Matemaattiset totuudet ovat olemassa täysin riippumatta siitä millaisessa maailmassa satumme olemaan!

        Se, ettei jossakin maailmassa ole lukumääriä on täysin irrelevanttia. Meidänkään maailmassa ei ole olemassa viisiulotteista palloa. Tästä huolimatta voin laskea kyseisen pallon pinta-alan ja tilavuuden suhteessa sen säteeseen.

        Joudut keksimään maailman, missä logiikka ei päde. Kokonaisluku aritmetiikka voidaan johtaa joukko-opista eli joudut keksimään maailman, missä joukko-opin aksioomat ovat epätosia.

        Tässä pääsemmekin kysymykseen: Onko logiikka universaalia vai voiko olla maailmoja, jossa logiikkakin on epätosi? Mehän voisimme jopa todistaa tuollaisen maailman mahdottomaksi, koska se olisi logiikan vastainen.

        Nimimerkki: Jumalan-kumous (ei-rekisteröitynyt professori)

        Järkisyitä: "Tässä pääsemmekin kysymykseen: Onko logiikka universaalia vai voiko olla maailmoja, jossa logiikkakin on epätosi? Mehän voisimme jopa todistaa tuollaisen maailman mahdottomaksi, koska se olisi logiikan vastainen."

        Olen todistanut logiikan ("reductio ad absurdum":in) avulla mahdottomaksi mm. sellaisen "maailman", joka olisi "kaikenkattava absoluuttinen tyhjyys", mutta tietenkään tuokaan todistus ei olisi mahdollinen esim. tuossa "kumotussa maailmassa"; tai *muissakaan* sellaisissa maailmoissa, jotka eivät olisi ominaisuuksiltaan *täsmälleen sellaisia*, että nuo ominaisuudet johtivat kausaalisesti kyseiseen tapahtumaan, eli tuohon suorittamaani todistukseen.

        Pidän siis esim. "reductio ad absurdum":ia kuitenkin validina todistusmenetelmänä, ainakin käytännön kannalta; mm. pyrittäessä rajaamaan niitä todellisuuden aspekteja, joihin on tarkoituksenmukaista *kiinnittää huomiota*; eli jotka ovat tuossa suhteessa relevantteja, mutta tuo validiteetti siis *perustuu* siihen, että havaitsemamme fysikaalisen universumin ominaisuudet ovat ne, mitkä ne ovat, enkä osaa äkkiseltään sanoa, että kuinka paljon ja missä suhteissa niitä pitäisi muuttaa; siis teoreettisena ajatuskokeena, koska käytännössähän emme kykene niitä muuttamaan, jotta universumi tuottaisikin vain sellaisia havaitsevia olentoja, joiden erittäin merkittävän enemmistön kanta olisikin se, että "reductio ad absurdum" on *havaintojen mukaan* *invalidi* todistusmenetelmä ja jos taas olemassa on sellaisia universumeita, joissa tuntemamme logiikka ei pätisi, niin jos ne kyettäisiin havaitsemaan, niin nuo "ulkopuoliset" havaitsijat voisivat hyvinkin päätellä tulleensa hulluiksi, sen sijaan, että päättelisivät havaitsevansa sellaisia universumeita, joissa siis heidän tuntemansa logiikka ei päde tai ainakaan vakuuta muita; kuten esim. täällä filosofia-palstalla... :D


      • Anonyymi
        Anonyymi kirjoitti:

        Nimimerkki: Jumalan-kumous (ei-rekisteröitynyt professori)

        Jatkanpa vielä tästä minäkin, koska keskustelu näyttääkin jatkuneen. Kaikenkaikkiaan, tämä on kyllä hyvin mielenkiintoinen, relevantti, filosofinen ja ehkä hiukan haastavakin aihe ja tällä porukalla siihen saadaan kyllä hyvin erilaisia näkökulmia, mutta siitä johtuen ehkä tuskin kovinkaan paljoa konsensusta; näin ennakoin: veni, vidi, vici. :D

        Järkisyitä: "Olen tässä kyllä Firdawsin kanssa samaa mieltä. Matematiikan abstraktien käsitteiden olemassa olo ei edellytä fyysistä todellisuutta. Tässä 'olemassa olo' ei ole fyysistä eikä edes henkistä/kokemuksellista vaan sitä, että kyseinen totuus pätee aina. Nämä (matemaattiset) totuudet ovat pakosti kuten ne ovat - ne eivät voi olla mitenkään muuten. Ne ovat perimmäisiä totuuksia."

        Vaan, kun eivät olekaan, koska vaikka *jos* ne olisivatkin "pakosti" *aina ja kaikkialla* jotakin, minkä hypoteesin paikkansapitävyyden; mahdollisesti varsin haasteellisen, todistamisen osalta, todistustaakka on tietenkin kyseisen oletuksen tekijällä, niin ne ovat "pakosti" sitä, mitä ne sitten ovatkin, koska niihin on *jokin syy* ja *reaalinen kausaalinen syy* voi olla vain *fysikaalinen*, ei matemaattinen tai looginen.

        Toistan, että koskaan ei olla *havaittu*, että matemaattinen käsite tai matemaattinen totuus tai mikään muukaan *kuvaus* *sinällään* olisi tuottanut mitään reaalisia vaikutuksia ja jos/kun kuitenkin pyritään lisäksi erityisesti korostamaan sitä, että pyritään tarkastelemaan todellisuutta tieteellisen menetelmän mukaisesti; kuten tässä ketjussa on tehty, niin seurauksena on selvä ristiriita, koska tieteellinen menetelmä perustuu nimenomaan *reaalisia* asioita koskeviin havaintoihin, eikä määritelmällisiin ja/tai edes empiiris-induktiivisesti pääteltyihinkään väitelauseiden totuusarvoihin.

        Eli, kuvaukset ovat "korkeintaan" ainakin *suhteellisen* paikkansapitäviä fysikaalisten rakenteiden/ilmiöiden kuvauksia; jostain näkökulmasta, eivätkä suinkaan mitään "perimmäisiä totuuksia". En ylipäätään usko, että mitään "perimmäisiä totuuksia" ylipäätään kyetään koskaan tietämäänkään; sikäli kuin niitä ylipäätään on olemassakaan, ainakaan missään mielekkäässä ja/tai ristiriidattomassa merkityksessä.

        Esim. Max Tegmark varmaankin on näistä asioista eri mieltä kanssani ainakin joiltakin osin, vaikka hänkin on multiversumi-näkökantaa oikeana todellisuuden kuvauksena pitävä, mutta hänen; varsin radikaalit, näkemyksensä siitä, että kaikki *olisi* perimmiltään matematiikkaa, ovat tuottaneet myös monenlaista; ja mielestäni monilta osin hyvin perusteltua ja vakuuttavaa kritiikkiä; jota kritiikkiä on kuvattu mm. oheisella, häntä kuvaavalla Wikipedia-sivulla. Eli; kuten aiemmin jo totesin, fysikaalista todellisuutta voidaan *kuvata* matemaattisesti ja se tietysti mahdollistaa myös esim. logiikan käytön kuvauksissa, mutta koska yksikään matemaattinen totuus tai käsite ei saa aikaan *mitään* kausaalisia vaikutuksia, niin ne eivät voi olla "perimmäisiä totuuksia".
        https://en.wikipedia.org/wiki/Max_Tegmark

        "Tälläisien perustavaa laatua olevien totuuksien olemassa olo on mielenkiintoista, koska niiden kautta voi olla mahdollista hakea vastausta filosofeja askarruttaneeseen kysymykseen: "Miksi on olemassa jotakin sen sijaan, ettei olisi olemassa mitään?""

        Olen samaa mieltä siitä, että mm. tuo kysymys on hyvin mielenkiintoinen ja muistan itse pohtineeni ensimmäisen kerran sitä jo noin 6-vuotiaana; täysin ilman *erityistä tuohon suuntaavaa ohjausta*; muilta *ihmisiltä*. Pohdin sitä, että olisiko mielekästä esittää kysymys koskien sitä, että mitä se merkitsisi *kokemuksellisesti*, jos *koskaan* ei olisi ollut *mitään* ja muodostettuani tuon kysymyksen, niin aivoiltani vei tuolloin vain noin sekunnin tuottaa vastaus tuohon kysymykseen. :D

        Kokemuksellisesti tuntemuksena oli se, että aivojen läpi kulki ikään kuin oikosulku ja ymmärsin heti, että koska olisi *fyysisesti* mahdotonta esittää kysymyksiä/vastauksia, jos aivojakaan ei olisi olemassa, niin aivojen prosessit olivat "jakautuneet kahtia"; prosessoimaan erikseen noita kahta vaihtoehtoa ja kun ne noin sekuntia myöhemmin olivat sitten saaneet työnsä päätökseen ja tulokset yhdistyivät; epäilemättä jonkinlaisessa automatisoidussa deterministisessä prosessissa, niin seurauksena oli *mitä kummallisin* tuntemus; ikään kuin nuo prosessit olisivat kumonneet toisensa.

        Jos olisin tuolloin ollut ns. "indoktrinoitu" esim. "platonistiseen ideaoppiin"; edes implisiittisesti, niin minulta olisi todennäköisesti vienyt enemmän aikaa tajuta se, että looginen päättely tosiaan on *fyysinen* prosessi ja totuusarvotkin ovat pohjimmiltaan nimenomaan fyysisiä asioita, jotka jotkut sitten *tulkitsevat* joksikin muuksi...

        Nimimerkki: Jumalan-kumous (ei-rekisteröitynyt professori)

        Ohessa vielä tarkempi linkki tuohon mainitsemaani Tegmark:in ns. "matemaattisen universumin hypoteesin" kritiikkiin liittyen:
        https://en.wikipedia.org/wiki/Mathematical_universe_hypothesis#Criticisms_and_responses


      • Anonyymi
        Anonyymi kirjoitti:

        Nimimerkki: Jumalan-kumous (ei-rekisteröitynyt professori)

        Ohessa vielä tarkempi linkki tuohon mainitsemaani Tegmark:in ns. "matemaattisen universumin hypoteesin" kritiikkiin liittyen:
        https://en.wikipedia.org/wiki/Mathematical_universe_hypothesis#Criticisms_and_responses

        Minusta ns. matemaattiset ja loogiset totuudet eivät mitenkään luonteeltaan poikkea muista ihmiskunnan keksinnöistä ja innovaatioista.

        Esim. polkupyörän idea (kun se on kerran keksitty) niin pysyy voimassa niin kauan kuin se idea säilyy ihmiskunnan kollektiivisessa tietotaidossa joko ihmismuistin tai kirjallisen (lasken tähön mukaan myös tietotekniset tallenteet)n aineiston tai fyysisten esimerkkikappaleiden avulla. Itse konkreettiset aktualisoidut polkupyörät sen sijaan vähitellen hajoavat ihan samalla tavalla kuin kaikki muutkin fysikaaliset objektit mutta ne tekniset innovaatiot kestävät kauemmin jos niitä kopioidaan tai niiden periaate säilytetään fysikaalisen informaation muodossa.

        Sama periaate pätee matemaattisten keksintöjen suhteen jotka äärimmäisen pitkälle yleistettyjä ja pelkistettyjä keksintöjä. Logiikka taas voi olla moniarvoista tai sumeaa logiikkaa joka soveltuu paremmin esim. robotiikkaan kuin 2-arvoinen jäykkä kyllä-ei kytkentä.

        Tässä minusta on ihan sama analogia kuin kaksoisrakokokeen viivästetyssä versiossa
        eli jokin on todellista vain jos se on mukana ihmiskunnan kollektiivisessa tietoisuudessa (se kollektiivi voi olla 2 tai useampia yksilöitä aina miljardeihin asti jossain sellaisessa "paikassa" lokalisoitunena jossa heillä on yhteinen ympäristö). Erilaiset tietoisten olioiden kollektiivit voivat tuottaa itselleen ihan omanlaisensa todellisuuden joka sen kollektiivin ulkopuolisen kannalta näyttää anomalialta tai huijaukselta/hulluudelta mutta on silti sen yhteisön toiminnan kannalta mielekäs kokonaisuus (esim. luonnonkansojen ja länsimaisenkin kansanperinteesen liittyvien yhteisöjen jäsenten merkittävät ja usein länsimaisten kannalta varsin merkilliset kyvyt esim. luonnonlääkinnässä ja viestinnässä ennen kännyköiden aikaa )

        Tämä selittäisi mm. jotkut ns. parapsykologiset ilmiöt kuten esim. levitaatiot joita on havaittu suhteellisen usein esim. Intiassa ja länsimaissakin muutaman kerran ( esim. Daniel Dunglas Home ja eräs keskiaikana elänyt munkki joka haltioitui messun aikana niin paljon että alkoi liidellä lähellä kirkon kattoa:D)

        Ilmaisen tämän (tietysti taas) aika kiistanalaisella tavalla koska minun kannattamassani ontologiassa ns. aine on alisteista ns. tietoisuudelle eli aineellinen maailma tavallaan sijaitsee tietoisuuden maailman sisällä eikä niin kun fysikalismissa jossa tietoisuus ja kaikki mahdollinen kokemus sijaitsee ihmisten kallon sisällä fysikaalisten aivojen neuronien keskinäisessä vuorovaikutuksessa ja rakenteessa.

        En kuitenkaan kannata puhdasta idealismia koska siinä tulee vastaan 'tietoisuuden vaikean ongelman' sijasta tietynlainen 'aineen vaikea ongelma' ainakin joissain tilanteissa mitkä sattumoisin ovat ihan samoja tilanteita missä fysikalisteillakin ilmenee samantapainen ongelma.

        Itse en usko superdeterminismiin ja hahmotan ns. aineenkin hierarkkisena ja laadullisesti tiettyjen aineen aspektien suhteen suhteellisena ontologiana jossa tietoisuus on korkeamman tasoista aine-energiaa jossa on matalampi entropia ja vähemmän säännönmukaisuuksia (=luonnonlakeja) kuin aistein havaittavissa olevalla aineella.

        Matematiikalla ja logiikalla on suurin hyöty ja arvo ainoastaan suhteessa niihin aineen säännönmukaisuuksiin ja se matematiikan hyöty vähenee sitä mukaa kuin ne säännönmukaisuudet vähenevät siirryttäessä mekaanis-automaattisesta ontologiasta intention (=tahdon) ohjaamaan ontologiaan eli tietoisen itseohjautuvuuden tasolle,

        Belisario


      • Anonyymi kirjoitti:

        Nimimerkki: Jumalan-kumous (ei-rekisteröitynyt professori)

        Järkisyitä: "Tietenkään matemaattiset totuudet eivät ole fyysisiä tai edes kokemuksellisia asioita eli tässä mielessä niitä ei ole olemassa. Idealistit voisivat sanoa, että jopa fyysistä maailmaa ei ole olemassa ellei joku ole sitä kokemassa. Kenties tässä takerrumme siihen, mitä ymmärrämme olemassa olon tarkoittavan."

        Tietenkin totuudet; kuten kaikki muutkin asiat; ovat *vähintään myös* fyysisiä asioita, vaikka tämä näkemys ei taatusti olekaan suosittu; mikä taas on sellainen seikka, jolle ei yleensäkään kannata antaa paljoa painoarvoa, siinä tapauksessa siis, että on oikeassa; kuten tietysti lähes kaikki, ainakin täällä, tietysti ovat, ainakin omasta mielestään...

        Esim. idealistit eivät kykene tuottamaan tieteellistä lisäarvoa edes pohdittaessa sitä, että mikä tuottaa ne laskennalliset resurssit, jotka *mahdollistavat* esim. havaitsemisen, joka kuitenkin tietenkin on; heidän ilmeisesti erittäin tärkeänä pitämänsä tietoisuudenkin, kannalta keskeistä.

        "Lakkaako fyysinen todellisuus olemasta silloin, jos kukaan ei koe sitä tietoisesti? Entä lakkaako matemaattinen totuus olemasta silloin, kun sitä ei ajattele kukaan? Miten lähdemme todistamaan, että joku asia on silloinkin, kun emme havaitse sitä? Sanotaan, että näet puun. Suljet silmät. Avaat silmät ja näet puun edelleen. Eikö tästä ole johdonmukaista olettaa, että puu on olemassa riippumatta siitä, että sitä havainnoidaan koko ajan?"

        Fyysisen todellisuuden olemassaolo ei edellytä havaitsemista, mutta matemaattisen totuuden; ja tietysti muidenkin *todellisuuden kuvausten* olemassaolo, edellyttää fyysistä todellisuutta ja nimenomaan sellaista fyysistä todellisuutta, joka mahdollistaa kyseisen kuvauksen. Olen siis samaa mieltä siitä, että fysikaaliset asiat ovat olemassa havainnoistamme riippumatta ja syy tuohon on se, että niillä on; tai ne ovat, *energiaa* ja energia on eräs edellytys sille, että voidaan ylipäätään edes kyetä kuvittelemaan mitään kuvauksia jne. Fyysisen todellisuuden; nimenomaan *vain ja ainoastaan*, reaalinen, olemassaolo, on luonnollisesti hyvin yhteensopiva; mm. ymmärryksen mahdollistajana käyttökelpoisimpana pitämäni vaihtoehdon, eli kvanttimekaniikan monimaailmatulkinnan, kanssa...

        Tietenkin; toisaalta, kannaltamme relevantit asiat ovat sellaisia, että kykenemme ne havaitsemaan tai ainakin tietämään jotakin niihin liittyvää, esim. sitä kautta, että niitä on tehty havaintoja joidenkin muiden toimesta, tai joidenkin mittalaitteiden avulla tai havaitsemme niiden vaikutuksi tai kykenemme päättelemään niistä jotakin muun käytettävissämme olevan informaation perusteella; käyttäen loogista päättelyä jne.

        Tästä kaikeasta ei siis kuitenkaan tietenkään seuraa, että "matemaattiset totuudet" olisivat olemassa riippumatta fysikaalisista edellytyksistänsä, koska siis niillä *ei* ole energiaa ja jos taas vedämme yhtäläisyysmerkin matemaattisten totuuksien ja niiden fysikaalisten edellytysten välille; eli "implisiittisesti annamme niille energiaa", niin kyseessä on jonkinlainen reifikaatio; joka on argumentointivirhe, ja tässä tapauksessa todellisuuden kuvauksen kannalta hyödytön samaistus, koska todellisuuden ns. "resoluutioltaan käyttökelpoinen" kuvaus nimenomaan edellyttää mm. edellytysten ja seurausten erottamista toisistaan ja muuten vaarana on myös "psykologisella kielellä" ilmaisten: "vaipuminen ns. Jungilaiseen pleroomaan" tuolta osin, josta konseptista ehkä esim. Belisario haluaa kertoa enemmänkin. :D

        ”Tietenkin totuudet; kuten kaikki muutkin asiat; ovat *vähintään myös* fyysisiä asioita,”

        Olen eri mieltä. Totuus on väittämä-lauseen ominaisuus eikä fyysinen asia. Esimerkiksi matemaattiset lauseet ovat joko tosia tai epätosia. Fyysisillä asioilla kuten vaikkapa kivellä ei ole totuusarvoa.

        Jos tarkoitit totuudella olemassa olemista, niin se on asia, joka meidän pitäisi määritellä. Kuten sanoin matemaattiset totuudet ovat tietyllä tavalla olemassa, mutta huomaan, että sinä määrittelet olemassa olon seuraavasti:
        Vain asiat joilla on energiaa (=kykyä tehdä työtä fyysisessä todellisuudessa) ovat olemassa.

        Tuon määritelmän mukaisesti matematiikkaa ja loogisia totuuksia ei ole olemassa, mutta minun mielestäni ne ovat kuitenkin olemassa, vaikka niiden olemassa olo ei olekaan fyysistä olemista.

        Jos määrittelemme olemassa olon asiaksi, jonka tietoiset olennot voivat havaita ja se on kaikille havaitsijoille samaa (ei siis vain oma kuvitelma), niin matematiikka on olemassa.


      • Anonyymi kirjoitti:

        Nimimerkki: Jumalan-kumous (ei-rekisteröitynyt professori)

        Järkisyitä: "Tietenkään matemaattiset totuudet eivät ole fyysisiä tai edes kokemuksellisia asioita eli tässä mielessä niitä ei ole olemassa. Idealistit voisivat sanoa, että jopa fyysistä maailmaa ei ole olemassa ellei joku ole sitä kokemassa. Kenties tässä takerrumme siihen, mitä ymmärrämme olemassa olon tarkoittavan."

        Tietenkin totuudet; kuten kaikki muutkin asiat; ovat *vähintään myös* fyysisiä asioita, vaikka tämä näkemys ei taatusti olekaan suosittu; mikä taas on sellainen seikka, jolle ei yleensäkään kannata antaa paljoa painoarvoa, siinä tapauksessa siis, että on oikeassa; kuten tietysti lähes kaikki, ainakin täällä, tietysti ovat, ainakin omasta mielestään...

        Esim. idealistit eivät kykene tuottamaan tieteellistä lisäarvoa edes pohdittaessa sitä, että mikä tuottaa ne laskennalliset resurssit, jotka *mahdollistavat* esim. havaitsemisen, joka kuitenkin tietenkin on; heidän ilmeisesti erittäin tärkeänä pitämänsä tietoisuudenkin, kannalta keskeistä.

        "Lakkaako fyysinen todellisuus olemasta silloin, jos kukaan ei koe sitä tietoisesti? Entä lakkaako matemaattinen totuus olemasta silloin, kun sitä ei ajattele kukaan? Miten lähdemme todistamaan, että joku asia on silloinkin, kun emme havaitse sitä? Sanotaan, että näet puun. Suljet silmät. Avaat silmät ja näet puun edelleen. Eikö tästä ole johdonmukaista olettaa, että puu on olemassa riippumatta siitä, että sitä havainnoidaan koko ajan?"

        Fyysisen todellisuuden olemassaolo ei edellytä havaitsemista, mutta matemaattisen totuuden; ja tietysti muidenkin *todellisuuden kuvausten* olemassaolo, edellyttää fyysistä todellisuutta ja nimenomaan sellaista fyysistä todellisuutta, joka mahdollistaa kyseisen kuvauksen. Olen siis samaa mieltä siitä, että fysikaaliset asiat ovat olemassa havainnoistamme riippumatta ja syy tuohon on se, että niillä on; tai ne ovat, *energiaa* ja energia on eräs edellytys sille, että voidaan ylipäätään edes kyetä kuvittelemaan mitään kuvauksia jne. Fyysisen todellisuuden; nimenomaan *vain ja ainoastaan*, reaalinen, olemassaolo, on luonnollisesti hyvin yhteensopiva; mm. ymmärryksen mahdollistajana käyttökelpoisimpana pitämäni vaihtoehdon, eli kvanttimekaniikan monimaailmatulkinnan, kanssa...

        Tietenkin; toisaalta, kannaltamme relevantit asiat ovat sellaisia, että kykenemme ne havaitsemaan tai ainakin tietämään jotakin niihin liittyvää, esim. sitä kautta, että niitä on tehty havaintoja joidenkin muiden toimesta, tai joidenkin mittalaitteiden avulla tai havaitsemme niiden vaikutuksi tai kykenemme päättelemään niistä jotakin muun käytettävissämme olevan informaation perusteella; käyttäen loogista päättelyä jne.

        Tästä kaikeasta ei siis kuitenkaan tietenkään seuraa, että "matemaattiset totuudet" olisivat olemassa riippumatta fysikaalisista edellytyksistänsä, koska siis niillä *ei* ole energiaa ja jos taas vedämme yhtäläisyysmerkin matemaattisten totuuksien ja niiden fysikaalisten edellytysten välille; eli "implisiittisesti annamme niille energiaa", niin kyseessä on jonkinlainen reifikaatio; joka on argumentointivirhe, ja tässä tapauksessa todellisuuden kuvauksen kannalta hyödytön samaistus, koska todellisuuden ns. "resoluutioltaan käyttökelpoinen" kuvaus nimenomaan edellyttää mm. edellytysten ja seurausten erottamista toisistaan ja muuten vaarana on myös "psykologisella kielellä" ilmaisten: "vaipuminen ns. Jungilaiseen pleroomaan" tuolta osin, josta konseptista ehkä esim. Belisario haluaa kertoa enemmänkin. :D

        "Fyysisen todellisuuden olemassaolo ei edellytä havaitsemista,"

        Samaa mieltä. Vain äärimmäiset idealistit eivät hyväksy tuota asiaa. Voihan olla niin, että fyysinen todellisuus mallinnetaan vasta siinä vaiheessa, kun havaitsija sitä katsoo - eikä fyysinen todellisuus ole olemassa havaintojen välillä. Se *vaikuttaa* jatkuvalta, muttei ole.

        "mutta matemaattisen totuuden; ja tietysti muidenkin *todellisuuden kuvausten* olemassaolo, edellyttää fyysistä todellisuutta"

        Tuo on pelkkä väittämä. Yhtä hyvin voin väittää Max Tegmarkin tavoin, että fyysisen todellisuuden olemassa olo edellyttää matemaattisen logiikan olemassa oloa.

        Otan vertauskuvan: Edellyttääkö tietokone ohjelman olemassa olo sitä, että on olemassa prosessori joka tulkitsee sitä? Nähdäkseni väität, että logiikka on olemassa vain, jos on olemassa fyysinen todellisuus, jossa sitä tulkitaan. Minä taas katson, että ohjelma/logiikka on olemassa myös silloin, kun sillä ei ole tulkitsijaa.


      • Anonyymi kirjoitti:

        Nimimerkki: Jumalan-kumous (ei-rekisteröitynyt professori)

        Järkisyitä: "Tietenkään matemaattiset totuudet eivät ole fyysisiä tai edes kokemuksellisia asioita eli tässä mielessä niitä ei ole olemassa. Idealistit voisivat sanoa, että jopa fyysistä maailmaa ei ole olemassa ellei joku ole sitä kokemassa. Kenties tässä takerrumme siihen, mitä ymmärrämme olemassa olon tarkoittavan."

        Tietenkin totuudet; kuten kaikki muutkin asiat; ovat *vähintään myös* fyysisiä asioita, vaikka tämä näkemys ei taatusti olekaan suosittu; mikä taas on sellainen seikka, jolle ei yleensäkään kannata antaa paljoa painoarvoa, siinä tapauksessa siis, että on oikeassa; kuten tietysti lähes kaikki, ainakin täällä, tietysti ovat, ainakin omasta mielestään...

        Esim. idealistit eivät kykene tuottamaan tieteellistä lisäarvoa edes pohdittaessa sitä, että mikä tuottaa ne laskennalliset resurssit, jotka *mahdollistavat* esim. havaitsemisen, joka kuitenkin tietenkin on; heidän ilmeisesti erittäin tärkeänä pitämänsä tietoisuudenkin, kannalta keskeistä.

        "Lakkaako fyysinen todellisuus olemasta silloin, jos kukaan ei koe sitä tietoisesti? Entä lakkaako matemaattinen totuus olemasta silloin, kun sitä ei ajattele kukaan? Miten lähdemme todistamaan, että joku asia on silloinkin, kun emme havaitse sitä? Sanotaan, että näet puun. Suljet silmät. Avaat silmät ja näet puun edelleen. Eikö tästä ole johdonmukaista olettaa, että puu on olemassa riippumatta siitä, että sitä havainnoidaan koko ajan?"

        Fyysisen todellisuuden olemassaolo ei edellytä havaitsemista, mutta matemaattisen totuuden; ja tietysti muidenkin *todellisuuden kuvausten* olemassaolo, edellyttää fyysistä todellisuutta ja nimenomaan sellaista fyysistä todellisuutta, joka mahdollistaa kyseisen kuvauksen. Olen siis samaa mieltä siitä, että fysikaaliset asiat ovat olemassa havainnoistamme riippumatta ja syy tuohon on se, että niillä on; tai ne ovat, *energiaa* ja energia on eräs edellytys sille, että voidaan ylipäätään edes kyetä kuvittelemaan mitään kuvauksia jne. Fyysisen todellisuuden; nimenomaan *vain ja ainoastaan*, reaalinen, olemassaolo, on luonnollisesti hyvin yhteensopiva; mm. ymmärryksen mahdollistajana käyttökelpoisimpana pitämäni vaihtoehdon, eli kvanttimekaniikan monimaailmatulkinnan, kanssa...

        Tietenkin; toisaalta, kannaltamme relevantit asiat ovat sellaisia, että kykenemme ne havaitsemaan tai ainakin tietämään jotakin niihin liittyvää, esim. sitä kautta, että niitä on tehty havaintoja joidenkin muiden toimesta, tai joidenkin mittalaitteiden avulla tai havaitsemme niiden vaikutuksi tai kykenemme päättelemään niistä jotakin muun käytettävissämme olevan informaation perusteella; käyttäen loogista päättelyä jne.

        Tästä kaikeasta ei siis kuitenkaan tietenkään seuraa, että "matemaattiset totuudet" olisivat olemassa riippumatta fysikaalisista edellytyksistänsä, koska siis niillä *ei* ole energiaa ja jos taas vedämme yhtäläisyysmerkin matemaattisten totuuksien ja niiden fysikaalisten edellytysten välille; eli "implisiittisesti annamme niille energiaa", niin kyseessä on jonkinlainen reifikaatio; joka on argumentointivirhe, ja tässä tapauksessa todellisuuden kuvauksen kannalta hyödytön samaistus, koska todellisuuden ns. "resoluutioltaan käyttökelpoinen" kuvaus nimenomaan edellyttää mm. edellytysten ja seurausten erottamista toisistaan ja muuten vaarana on myös "psykologisella kielellä" ilmaisten: "vaipuminen ns. Jungilaiseen pleroomaan" tuolta osin, josta konseptista ehkä esim. Belisario haluaa kertoa enemmänkin. :D

        "Esim. idealistit eivät kykene tuottamaan tieteellistä lisäarvoa edes ..."

        Pidän idealismia jonkinasteisena filosofisena hömppänä. Silti jos pitäydymme vain rationaaliseen päättelyyn idealismia on vaikea kumota ennen kuin ymmärrämme filosofian "vaikean kysymyksen" eli sen miten tietoisuus voi emergoitua fysikaalisesta (materialistisesta) todellisuudesta.


      • Anonyymi
        Järkisyitä kirjoitti:

        "Esim. idealistit eivät kykene tuottamaan tieteellistä lisäarvoa edes ..."

        Pidän idealismia jonkinasteisena filosofisena hömppänä. Silti jos pitäydymme vain rationaaliseen päättelyyn idealismia on vaikea kumota ennen kuin ymmärrämme filosofian "vaikean kysymyksen" eli sen miten tietoisuus voi emergoitua fysikaalisesta (materialistisesta) todellisuudesta.

        Jkumous:"Esim. idealistit eivät kykene tuottamaan tieteellistä lisäarvoa edes ..."

        Eivät ontologiset kannanotot muutenkaan sellaisenaan tuota koskaan mitään lisäarvoa koska ne aina poikkeuksetta liittyvät jonkinlaiseen uskonnonkaltaiseen vakaumukseen.

        Fysikalismi/materialismi ovat ihan samanlaisia havainnon tai kokemuksen "päälle liimattuja" filosofisia luokitteluja kuin idealismikin ja kaikki muut mahdolliset ontologiset kategoriat jotka eivät sellaisenaan tietenkään tuota mitään lisäarvoa jos lisäarvolla tarkoitetaan jotain konkreettista elämisen kannalta tarpeellista hyödykettä kuten ravintoa, tekniikkaa, tiedettä tai taidetta.

        Sen sijaan yksilöinä idealistit voivat tietysti saada ihan yhtä paljon konkreettista lisäarvoa aikaan kuin materialistit tai vaikka raamattu-uskovaisetkin.

        Ontologiaa ei ylipäätänsä kannata sotkea tieteeseen koska se ei todellakaan tuota mitään konkreettista lisäarvoa tieteen tuloksille missään muodossa. Ei kenenkään kokemus muutu siitä paremmaksi eikä huonommaksi pelkästään sen takia että se tulkitaan idealismin tai fysikalismin puitteissa eikä ateisti tee sen parempaa tiedettä kuin teisti tai agnostikko.

        Tiede ei ole sama asia kuin fysikalismi vaikka ne usein assosioidaan toisiinsa koska suurin osa luonnontieteilijöistä lienee nykyään filosofis-uskonnolliselta vakaumukseltaan sekä fysikalisteja että ateisteja.

        Ontologisen sekoilun sijasta kaikki tiede voidaan tulkita tutkimuskohteen ontologian suhteen neutraalisti esim. jonkinlaisen systeemiteoreettisen mallin mukaisesti. Systeemiteoriassa ei oleteta reduktionismia eikä materialismia vaan todellisuus pyritään hahmottamaan systeemeinä ja niiden välisinä prosesseina.

        Belisario


      • Anonyymi
        Anonyymi kirjoitti:

        Jkumous:"Esim. idealistit eivät kykene tuottamaan tieteellistä lisäarvoa edes ..."

        Eivät ontologiset kannanotot muutenkaan sellaisenaan tuota koskaan mitään lisäarvoa koska ne aina poikkeuksetta liittyvät jonkinlaiseen uskonnonkaltaiseen vakaumukseen.

        Fysikalismi/materialismi ovat ihan samanlaisia havainnon tai kokemuksen "päälle liimattuja" filosofisia luokitteluja kuin idealismikin ja kaikki muut mahdolliset ontologiset kategoriat jotka eivät sellaisenaan tietenkään tuota mitään lisäarvoa jos lisäarvolla tarkoitetaan jotain konkreettista elämisen kannalta tarpeellista hyödykettä kuten ravintoa, tekniikkaa, tiedettä tai taidetta.

        Sen sijaan yksilöinä idealistit voivat tietysti saada ihan yhtä paljon konkreettista lisäarvoa aikaan kuin materialistit tai vaikka raamattu-uskovaisetkin.

        Ontologiaa ei ylipäätänsä kannata sotkea tieteeseen koska se ei todellakaan tuota mitään konkreettista lisäarvoa tieteen tuloksille missään muodossa. Ei kenenkään kokemus muutu siitä paremmaksi eikä huonommaksi pelkästään sen takia että se tulkitaan idealismin tai fysikalismin puitteissa eikä ateisti tee sen parempaa tiedettä kuin teisti tai agnostikko.

        Tiede ei ole sama asia kuin fysikalismi vaikka ne usein assosioidaan toisiinsa koska suurin osa luonnontieteilijöistä lienee nykyään filosofis-uskonnolliselta vakaumukseltaan sekä fysikalisteja että ateisteja.

        Ontologisen sekoilun sijasta kaikki tiede voidaan tulkita tutkimuskohteen ontologian suhteen neutraalisti esim. jonkinlaisen systeemiteoreettisen mallin mukaisesti. Systeemiteoriassa ei oleteta reduktionismia eikä materialismia vaan todellisuus pyritään hahmottamaan systeemeinä ja niiden välisinä prosesseina.

        Belisario

        Kyllähän oikea ontologinen olettama tuo lisäarvoa. Esimerkki tästä on sähkömagneettisten aaltojen olemassa olo, mikä ymmärrettiin oikean ontologian kautta ja niiden olemassa olo oli seurausta oikeista todellisuuden malleista.

        Idealistinen ontologia voisi tuoda lisäarvoa esimerkiksi, jos telepatia tai kyky vaikuttaa todellisuuteen ajatuksen voimalla, osoittautuisi olemassa olevaksi. Idealismin kautta ei kuitenkaan olla saatu aikaan juuri mitään hyödyllistä.


      • Anonyymi
        Anonyymi kirjoitti:

        Kyllähän oikea ontologinen olettama tuo lisäarvoa. Esimerkki tästä on sähkömagneettisten aaltojen olemassa olo, mikä ymmärrettiin oikean ontologian kautta ja niiden olemassa olo oli seurausta oikeista todellisuuden malleista.

        Idealistinen ontologia voisi tuoda lisäarvoa esimerkiksi, jos telepatia tai kyky vaikuttaa todellisuuteen ajatuksen voimalla, osoittautuisi olemassa olevaksi. Idealismin kautta ei kuitenkaan olla saatu aikaan juuri mitään hyödyllistä.

        "Kyllähän oikea ontologinen olettama tuo lisäarvoa. "

        Tuo liittyy tietysti siihen mitä kukin pitää arvokkaana tai arvokkaampana asiana kuin joku muu asia eli epäsuorasti tuolla voi olla vaikutusta siihen mihin ihmiset kuluttavat eniten aikaa ja vaivaa kussakin kulttuurissa.

        Voi olla että kaikki nykyinen tekniikka olisi jäänyt kehittämättä jos suurin osa länsimaisista ihmisistä olisi keskittynyt esim. kehonsa, tunteidensa tai älynsä kehittämiseen kuten muinaisessa hindukulttuurissa Intiassa sen sijaan että ovat kehittäneet laitteita, teollisuutta yms.

        Voi myös sitten olla että meillä ei nyt olisi sellaisia ympäristöongelmia, mielenterveysongelmia ym. yksipuolisen tieteellis-teknisen kulttuurin tuottamia ongelmia kuin nyt jos olisimme keskittyneet itsemme kehittämiseen eikä elämän mukavuuden ja helppouden näennäiseen lisäämiseen kuten tässä meidän kulttuurissa.

        Sinänsä tieteellisen metodin kannalta sillä ontologialla on hyvin vähän merkitystä.

        "Idealistinen ontologia voisi tuoda lisäarvoa esimerkiksi, jos telepatia tai kyky vaikuttaa todellisuuteen ajatuksen voimalla, osoittautuisi olemassa olevaksi. "

        Ne kyvyt ovat mahdollisia ja varsin todellisia mutta eivät onneksi kovin helposti eikä automaattisesti kehity niin hyviksi että niiden väärinkäytsötä olisi merkittävää haittaa tässä kulttuurissa,

        Materiaalisen tekniikan kehittäminen on helpompaa koska se ns. aine noudattaa aika selkeitä säännönmukaisuuksia eikä sillä ole juurikaan omaa tahtoa eikä varsinkaan tahdon vapautta :D

        Belisario


      • Anonyymi kirjoitti:

        "Kyllähän oikea ontologinen olettama tuo lisäarvoa. "

        Tuo liittyy tietysti siihen mitä kukin pitää arvokkaana tai arvokkaampana asiana kuin joku muu asia eli epäsuorasti tuolla voi olla vaikutusta siihen mihin ihmiset kuluttavat eniten aikaa ja vaivaa kussakin kulttuurissa.

        Voi olla että kaikki nykyinen tekniikka olisi jäänyt kehittämättä jos suurin osa länsimaisista ihmisistä olisi keskittynyt esim. kehonsa, tunteidensa tai älynsä kehittämiseen kuten muinaisessa hindukulttuurissa Intiassa sen sijaan että ovat kehittäneet laitteita, teollisuutta yms.

        Voi myös sitten olla että meillä ei nyt olisi sellaisia ympäristöongelmia, mielenterveysongelmia ym. yksipuolisen tieteellis-teknisen kulttuurin tuottamia ongelmia kuin nyt jos olisimme keskittyneet itsemme kehittämiseen eikä elämän mukavuuden ja helppouden näennäiseen lisäämiseen kuten tässä meidän kulttuurissa.

        Sinänsä tieteellisen metodin kannalta sillä ontologialla on hyvin vähän merkitystä.

        "Idealistinen ontologia voisi tuoda lisäarvoa esimerkiksi, jos telepatia tai kyky vaikuttaa todellisuuteen ajatuksen voimalla, osoittautuisi olemassa olevaksi. "

        Ne kyvyt ovat mahdollisia ja varsin todellisia mutta eivät onneksi kovin helposti eikä automaattisesti kehity niin hyviksi että niiden väärinkäytsötä olisi merkittävää haittaa tässä kulttuurissa,

        Materiaalisen tekniikan kehittäminen on helpompaa koska se ns. aine noudattaa aika selkeitä säännönmukaisuuksia eikä sillä ole juurikaan omaa tahtoa eikä varsinkaan tahdon vapautta :D

        Belisario

        En vetäisi yhtäläisyysmerkkiä fysikalismin ja materian arvostamisen välillä. Itse olen fysikalisti (=materialisti), mutta katson silti, että henkinen hyvinvointi on kaikkein arvokkainta ja materia on vain yksi pakollinen väline sen saavuttamisessa. En arvosta tuhlaus/kertakäyttökulttuuria vaan viittaan tässä vain selviytymisen kannalta pakollisiin hyödykkeisiin. Toki taide, musiikki yms ovat myös materialistisia asioita, jotka ovat mielen ruokaa.


      • Anonyymi
        Järkisyitä kirjoitti:

        En vetäisi yhtäläisyysmerkkiä fysikalismin ja materian arvostamisen välillä. Itse olen fysikalisti (=materialisti), mutta katson silti, että henkinen hyvinvointi on kaikkein arvokkainta ja materia on vain yksi pakollinen väline sen saavuttamisessa. En arvosta tuhlaus/kertakäyttökulttuuria vaan viittaan tässä vain selviytymisen kannalta pakollisiin hyödykkeisiin. Toki taide, musiikki yms ovat myös materialistisia asioita, jotka ovat mielen ruokaa.

        "En vetäisi yhtäläisyysmerkkiä fysikalismin ja materian arvostamisen välillä. "

        Joo ei se ihan niinkään ole. Sanotaan nyt vaikka niin että minusta näyttää että materialisti hahmottaa kokemaansa kielellisten käsitteiden avulla erilaisiin "laatikoihin" joista yhden laatikon kyljessä lukee "materia" ja toisen laatikon kyljessä lukee "henki" ja sinne materia-lootaan lajitellaan kaikki se kokemus mikä tuntuu konkreettiselta ja todelliselta ja sinne henki-lootaan taas kaikki se kokemus mikä tuntuu jotenkin epätodelliselta, haamumaiselta, abstraktilta yms.

        Idealisti taas käyttää vain yhtä laatikkoa koska kaikki mahdolinen on aina kokemusta tai datastreamiä tai mitä käsitettä tässä yhteydessä on tottunut käyttämään.

        Sitten on toisenlainen datastream (stream 2) jota kutsutaan "unimaailmaksi" sen takia että se on yleensä vähemmän pysyvä ja sen konteksti vaihtelee yleensä jokaisessa unessa vaikka joskus ainakin minä näen ns. jatkounia jotka jatkavat siitä mihin se edellinen uni jäi joskus aikaisemmin eikä välttämättä sen ykkös datastreamiin (valvekokemus) suhteutettuna ajallisesti peräkkäin seuraavana yönä vaan välissä voi olla viikkoja tai kuukausia.

        Molemmat datastreamit tuntuvat todelliselta ja molemmissa voi halutessaan hahmottaa osan sen datastreamin komponenteista "aineeksi" ja toisen osan "hengeksi".

        Mielestäni Tom Campbell osaa selittää tämän varsin hyvin omassa kaiken teoriassaan vaikka en olekaan ihan kaikesta täysin samaa mieltä.

        https://www.youtube.com/watch?v=zstsJ7-ULM0

        Mielestäni kaikissa mahdollisissa selitysmalleissa ovat heikkoutensa varsinkin jos ne yritetään ymmärtää jotenkin kirjaimellisesti. Minusta erilaiset mallit ovat lähinnä käsitteellisiä työkaluja joista kukin tavoittaa vain joitakin aspekteja ns. todellisuudesta.

        Itse kannatan tällä hetkellä kolmea tai neljää hieman erilaista mallia samanaikaisesti koska ne ovat toisiaan täydenteviä


        1) Tom Campbellin simulaatiomalli
        2) Bernardo Kastrupin moderni idealismi
        3( Gurdjieffin ja J.G. Bennettin suhteellinen materialismi
        4) Joseph P. Farrellin topologinen vertauskuva

        Gurdjieffiin ja Bennettiin perehdyin jo n. 40 vuotta ja noihin kahteen muuhun viimeisen viiden vuoden aikana. Farrelliin n. 10 v sitten ja noista kaikista minulla on melkein kaikki aineisto mitä ylipäätänsä on saatavilla (kymmeniä kirjoja, satoja videoita ja äänitteitä netin kautta kerättynä vuodesta 1995 ).

        Tieteestä aloitin n. 40 v sitten keräilemään lehtileikkeitä ja kirjoja ja tutkimaan varsinkin erilaisia outoja ja anomaalisia ilmiöitä jotka eivät mahdu valtavirtamalleihin. Samoihin aikoihin opettelin ohjelmoimaan ja harrastamaan tietotekniikkaa kausittain varsin intensiivisesti. Lopputulos on se mitä mieltä tällä hetkellä olen ja voi olla että sekin kanta muuttuu ainakin jonkin verran vielä niin kauan kuin kykenen oppimaan uutta ja perehtymään asioihin.

        Belisario


      • Anonyymi
        Anonyymi kirjoitti:

        Minusta ns. matemaattiset ja loogiset totuudet eivät mitenkään luonteeltaan poikkea muista ihmiskunnan keksinnöistä ja innovaatioista.

        Esim. polkupyörän idea (kun se on kerran keksitty) niin pysyy voimassa niin kauan kuin se idea säilyy ihmiskunnan kollektiivisessa tietotaidossa joko ihmismuistin tai kirjallisen (lasken tähön mukaan myös tietotekniset tallenteet)n aineiston tai fyysisten esimerkkikappaleiden avulla. Itse konkreettiset aktualisoidut polkupyörät sen sijaan vähitellen hajoavat ihan samalla tavalla kuin kaikki muutkin fysikaaliset objektit mutta ne tekniset innovaatiot kestävät kauemmin jos niitä kopioidaan tai niiden periaate säilytetään fysikaalisen informaation muodossa.

        Sama periaate pätee matemaattisten keksintöjen suhteen jotka äärimmäisen pitkälle yleistettyjä ja pelkistettyjä keksintöjä. Logiikka taas voi olla moniarvoista tai sumeaa logiikkaa joka soveltuu paremmin esim. robotiikkaan kuin 2-arvoinen jäykkä kyllä-ei kytkentä.

        Tässä minusta on ihan sama analogia kuin kaksoisrakokokeen viivästetyssä versiossa
        eli jokin on todellista vain jos se on mukana ihmiskunnan kollektiivisessa tietoisuudessa (se kollektiivi voi olla 2 tai useampia yksilöitä aina miljardeihin asti jossain sellaisessa "paikassa" lokalisoitunena jossa heillä on yhteinen ympäristö). Erilaiset tietoisten olioiden kollektiivit voivat tuottaa itselleen ihan omanlaisensa todellisuuden joka sen kollektiivin ulkopuolisen kannalta näyttää anomalialta tai huijaukselta/hulluudelta mutta on silti sen yhteisön toiminnan kannalta mielekäs kokonaisuus (esim. luonnonkansojen ja länsimaisenkin kansanperinteesen liittyvien yhteisöjen jäsenten merkittävät ja usein länsimaisten kannalta varsin merkilliset kyvyt esim. luonnonlääkinnässä ja viestinnässä ennen kännyköiden aikaa )

        Tämä selittäisi mm. jotkut ns. parapsykologiset ilmiöt kuten esim. levitaatiot joita on havaittu suhteellisen usein esim. Intiassa ja länsimaissakin muutaman kerran ( esim. Daniel Dunglas Home ja eräs keskiaikana elänyt munkki joka haltioitui messun aikana niin paljon että alkoi liidellä lähellä kirkon kattoa:D)

        Ilmaisen tämän (tietysti taas) aika kiistanalaisella tavalla koska minun kannattamassani ontologiassa ns. aine on alisteista ns. tietoisuudelle eli aineellinen maailma tavallaan sijaitsee tietoisuuden maailman sisällä eikä niin kun fysikalismissa jossa tietoisuus ja kaikki mahdollinen kokemus sijaitsee ihmisten kallon sisällä fysikaalisten aivojen neuronien keskinäisessä vuorovaikutuksessa ja rakenteessa.

        En kuitenkaan kannata puhdasta idealismia koska siinä tulee vastaan 'tietoisuuden vaikean ongelman' sijasta tietynlainen 'aineen vaikea ongelma' ainakin joissain tilanteissa mitkä sattumoisin ovat ihan samoja tilanteita missä fysikalisteillakin ilmenee samantapainen ongelma.

        Itse en usko superdeterminismiin ja hahmotan ns. aineenkin hierarkkisena ja laadullisesti tiettyjen aineen aspektien suhteen suhteellisena ontologiana jossa tietoisuus on korkeamman tasoista aine-energiaa jossa on matalampi entropia ja vähemmän säännönmukaisuuksia (=luonnonlakeja) kuin aistein havaittavissa olevalla aineella.

        Matematiikalla ja logiikalla on suurin hyöty ja arvo ainoastaan suhteessa niihin aineen säännönmukaisuuksiin ja se matematiikan hyöty vähenee sitä mukaa kuin ne säännönmukaisuudet vähenevät siirryttäessä mekaanis-automaattisesta ontologiasta intention (=tahdon) ohjaamaan ontologiaan eli tietoisen itseohjautuvuuden tasolle,

        Belisario

        Nimimerkki: Jumalan-kumous (ei-rekisteröitynyt professori)

        Belisario: "Minusta ns. matemaattiset ja loogiset totuudet eivät mitenkään luonteeltaan poikkea muista ihmiskunnan keksinnöistä ja innovaatioista."

        Ainakaan ne eivät poikkea toisistaan sen osalta, että niillä siis on fysikaalisia edellytyksiä ja mm. niiden havaittaviksi tuottaminen edellyttää myös tietyn tuotantotekniikan, vaikka tietysti nuo nimenomaiset edellytykset ja tekniikat ovat vähintään osin erilaiset. Paul Erdös muistaakseni totesi joskus mm., että matemaatikko on kahvia teoreemoiksi muuttava kone. :D

        "Tässä minusta on ihan sama analogia kuin kaksoisrakokokeen viivästetyssä versiossa […]".

        Olen samoilla linjoilla siltä osin, että erilaiset näkökulmat todellisuuteen voivat tosiaan erilaisissa tilanteissa olla selvästi toimivimpia. En nyt muista, että kuka sen sanoi, mutta idea meni jotenkin näin ikään, että: "An academic is a person who tries to create models describing reality each day meticulously from 8 AM to 4 PM and then uses rest of his/her time in trying to forget those very same models…" :D …

        En kuitenkaan suinkaan menisi niin pitkälle, että väittäisin, että todellisuus olisi olemassa *vain* tiedostettuna, vaan vain että tietyn toimijan itsensä kannalta *relevantteja* ovat vain ne todellisuuden osat/piirteet, jotka tai joiden vaikutukset hän; vähintään jossain vaiheessa, tiedostaa, minkä seikan oikeaan-osuva ennakointi voi tietenkin olla hyvinkin haasteellista…

        Itseasiassa, jos todellisuus on sellainen kone, joka käy läpi kaikki tilansa jossain järjestyksessä; eli "toteuttaa omat vapausasteensa", niin *kaikki* todellisuuden osat/piirteet ovat relevantteja kaikille tietoisille olennoille, vaikka ne; väistämättömistä kognitiivisista rajoitteistaan johtuen, eivät kykenekään välttämättä koskaan *yhdistämään* noita; tuossa suhteessa irrallisia tietoisia kokemuksiaan, toisiinsa ainakaan tietoisesti...

        "Tämä selittäisi mm. jotkut ns. parapsykologiset ilmiöt kuten esim. levitaatiot joita on havaittu suhteellisen usein esim. Intiassa ja länsimaissakin muutaman kerran ( esim. Daniel Dunglas Home ja eräs keskiaikana elänyt munkki joka haltioitui messun aikana niin paljon että alkoi liidellä lähellä kirkon kattoa:D)"

        Mietiskelinkin, että miksi "mielessäni pyöri" sana "pyhä", eli esim. "pyhän" Joosef Kupertinolaisen väitetään kyenneen levitoimaan, mutta tuollaisten ns. ihmeiksi tapahtuma-aikoinaan luokiteltujen tapahtumien raportoinnit näyttävät vähentyneen ajan funktiona, mikä voi olla selitettävissä yksinkertaisesti esim. sitä kautta, että tuontyyppisiä tapahtumia on aikoinaan *tulkittu* sen mukaan, mitä kulloinkin on pidetty mahdollisena.

        Jos tarkastelemme erilaisten selitysmallien paikkansapitävyyden suhteellista todennäköisyyttä, niin on ilmeistä, että on *huomattavasti todennäköisempää*, että tuonkaltaisissa tapahtumissa on kyse juurikin siitä, että *tulkitaan* fyysisiä vaikutuksia esim. lentäviksi munkeiksi, kuin se, että munkit lentäisivät *fyysisesti*.

        Eli, on ilmeistä, että vaikka esim. tiettyyn munkkiin liittyisikin mm. "outoja ja aivan reaalisiakin ja epätavallisia korrelaatteja"; kuten esim. minuun ja Lenin:iinkin liittyy, niin on silti huomattavasti paljon todennäköisempää esim. että joukko "havainnoitsijoita" näkee samanaikaisesti samansisältöistä unta siitä, että esim. tietty munkki lentää fyysisesti, kuin se, että kyseinen munkki lentäisi fyysisesti...

        "Ilmaisen tämän (tietysti taas) aika kiistanalaisella tavalla koska minun kannattamassani ontologiassa ns. aine on alisteista ns. tietoisuudelle eli aineellinen maailma tavallaan sijaitsee tietoisuuden maailman sisällä eikä niin kun fysikalismissa jossa tietoisuus ja kaikki mahdollinen kokemus sijaitsee ihmisten kallon sisällä fysikaalisten aivojen neuronien keskinäisessä vuorovaikutuksessa ja rakenteessa."

        Tuolta osin kannattaa huomata se, että mm. aivojen ja havaittavan todellisuuden rakenne yleisemmässäkin katsannossa on *äärimmäisen* epäsatunnainen, koska mitään aivoja ei ylipäätään keskimäärin pitäisi tilastollisesti olla havaittavissakiin ja se on relevanttien tekijöiden osalta itseasiassa *juuri* "oikeanlainen" mm., jotta juuri tämän ja vain tämän lauseen tuottaminen juuri tässä asiayhteydessä oli mahdollinen ja myös, jotta mm. sen sisällön tiedostaminen oli mahdollista…

        Itse edustan sitä kantaa, että oikeastaan kaikki havaitsemamme todellisuuden aspektit ovat; siis jo lähtökohtaisestikin, eli ennen tuota ns. "tietoisuuden kehittämistä", ns. "creme de la creme":ä, eli *äärimmäisen valikoituneita*. En usko, että tuohon valikoitumiseen tarvitaan tietoisuutta tai että tietoisuus kykenisi tuollaista valtavaa urakkaa ylipäätään suorittamaankaan, vaan tietoisuus on vain seuraus ja itseasiassa vain sivutuote tuosta *tavattomasta valikoituneisuudesta*, joskin *yksilöiden subjektiivisten kokemusten* kannalta tietysti eräs hyvin oleellisista aspekteista...


      • Anonyymi
        Anonyymi kirjoitti:

        Minusta ns. matemaattiset ja loogiset totuudet eivät mitenkään luonteeltaan poikkea muista ihmiskunnan keksinnöistä ja innovaatioista.

        Esim. polkupyörän idea (kun se on kerran keksitty) niin pysyy voimassa niin kauan kuin se idea säilyy ihmiskunnan kollektiivisessa tietotaidossa joko ihmismuistin tai kirjallisen (lasken tähön mukaan myös tietotekniset tallenteet)n aineiston tai fyysisten esimerkkikappaleiden avulla. Itse konkreettiset aktualisoidut polkupyörät sen sijaan vähitellen hajoavat ihan samalla tavalla kuin kaikki muutkin fysikaaliset objektit mutta ne tekniset innovaatiot kestävät kauemmin jos niitä kopioidaan tai niiden periaate säilytetään fysikaalisen informaation muodossa.

        Sama periaate pätee matemaattisten keksintöjen suhteen jotka äärimmäisen pitkälle yleistettyjä ja pelkistettyjä keksintöjä. Logiikka taas voi olla moniarvoista tai sumeaa logiikkaa joka soveltuu paremmin esim. robotiikkaan kuin 2-arvoinen jäykkä kyllä-ei kytkentä.

        Tässä minusta on ihan sama analogia kuin kaksoisrakokokeen viivästetyssä versiossa
        eli jokin on todellista vain jos se on mukana ihmiskunnan kollektiivisessa tietoisuudessa (se kollektiivi voi olla 2 tai useampia yksilöitä aina miljardeihin asti jossain sellaisessa "paikassa" lokalisoitunena jossa heillä on yhteinen ympäristö). Erilaiset tietoisten olioiden kollektiivit voivat tuottaa itselleen ihan omanlaisensa todellisuuden joka sen kollektiivin ulkopuolisen kannalta näyttää anomalialta tai huijaukselta/hulluudelta mutta on silti sen yhteisön toiminnan kannalta mielekäs kokonaisuus (esim. luonnonkansojen ja länsimaisenkin kansanperinteesen liittyvien yhteisöjen jäsenten merkittävät ja usein länsimaisten kannalta varsin merkilliset kyvyt esim. luonnonlääkinnässä ja viestinnässä ennen kännyköiden aikaa )

        Tämä selittäisi mm. jotkut ns. parapsykologiset ilmiöt kuten esim. levitaatiot joita on havaittu suhteellisen usein esim. Intiassa ja länsimaissakin muutaman kerran ( esim. Daniel Dunglas Home ja eräs keskiaikana elänyt munkki joka haltioitui messun aikana niin paljon että alkoi liidellä lähellä kirkon kattoa:D)

        Ilmaisen tämän (tietysti taas) aika kiistanalaisella tavalla koska minun kannattamassani ontologiassa ns. aine on alisteista ns. tietoisuudelle eli aineellinen maailma tavallaan sijaitsee tietoisuuden maailman sisällä eikä niin kun fysikalismissa jossa tietoisuus ja kaikki mahdollinen kokemus sijaitsee ihmisten kallon sisällä fysikaalisten aivojen neuronien keskinäisessä vuorovaikutuksessa ja rakenteessa.

        En kuitenkaan kannata puhdasta idealismia koska siinä tulee vastaan 'tietoisuuden vaikean ongelman' sijasta tietynlainen 'aineen vaikea ongelma' ainakin joissain tilanteissa mitkä sattumoisin ovat ihan samoja tilanteita missä fysikalisteillakin ilmenee samantapainen ongelma.

        Itse en usko superdeterminismiin ja hahmotan ns. aineenkin hierarkkisena ja laadullisesti tiettyjen aineen aspektien suhteen suhteellisena ontologiana jossa tietoisuus on korkeamman tasoista aine-energiaa jossa on matalampi entropia ja vähemmän säännönmukaisuuksia (=luonnonlakeja) kuin aistein havaittavissa olevalla aineella.

        Matematiikalla ja logiikalla on suurin hyöty ja arvo ainoastaan suhteessa niihin aineen säännönmukaisuuksiin ja se matematiikan hyöty vähenee sitä mukaa kuin ne säännönmukaisuudet vähenevät siirryttäessä mekaanis-automaattisesta ontologiasta intention (=tahdon) ohjaamaan ontologiaan eli tietoisen itseohjautuvuuden tasolle,

        Belisario

        Nimimerkki: Jumalan-kumous (ei-rekisteröitynyt professori)

        Belisario: "Itse en usko superdeterminismiin ja hahmotan ns. aineenkin hierarkkisena ja laadullisesti tiettyjen aineen aspektien suhteen suhteellisena ontologiana jossa tietoisuus on korkeamman tasoista aine-energiaa jossa on matalampi entropia ja vähemmän säännönmukaisuuksia (=luonnonlakeja) kuin aistein havaittavissa olevalla aineella."

        Yllättävää, että pidät siis tietoisuuttakin *aineena/energiana* ja tuo kohta kuulostaa mielenkiintoiselta, mm. koska jos tietoisuus on ainetta/energiaa, niin sitä voidaan tutkia ja mitata objektiivisesti, mikä tietenkin avaa hyvin mielenkiintoisia mahdollisuuksia tieteen edistämiseen. En hahmota, että mitä tarkoitat sillä, että tietoisuuteen liittyisi *sekä* matalampi entropia, että *vähemmän* säännönmukaisuuksia, koska entropian *puutehan* juuri tarkoittaa säännönmukaisuuksia.

        Mielestäni olet yksinkertaisesti väärässä siltä osin, jos/kun oletat, että todellisuudessa olisi olemassa jotain *todellisia* vapausasteita, koska kaikki on riippuvaista edellytyksistänsä, minkä olen sanonut lukemattomia kertoja ja tiedän, että toistaminen ei ole validi argumentointitapa, mutta kun muutkin toistavat omia teesejään, niin on vain tasapuolista, että minäkin toistan mm. tuota. Vapaus on valitettavasti niin epämääräinen käsite, että kun sitä käytetään, niin on helppo mennä "ihan metsään", ilman että sitä edes huomaakaan. Superdeterminismiin toisaalta päädytään sitä kautta, että suljetaan pois sellaisia mahdottomia/mielettömiä vaihtoehtoja joihin esim. sisältyy implisiittisesti tyhjästä syntyviä vaikutuksia.

        Mielestäni kannattaisikin kuviteltuun *vapauteen* "ripustautumisen* sijasta keskittyä tavoitteiden kannalta oleellisten *resurssien* kuvaamiseen, eli esim. niiden, joilla voi olla oleellista vaikutusta siihen, että missä määrin kykenee tiedostamaan todellisuutta… Kyse on siis pikemmin vain siitä, että mitkä omaan toimintaansa *vaikuttavat tekijät* kykenee tiedostamaan käytettävissään olevien resurssien puitteissa, jotka resurssit ovat mm. energiaan ja tietojenkäsittelyyn liittyviä. Lisäksi, koska noita vaikuttavia yksittäisiä tekijöitä on *loputtomasti*, niin tietoisuuden merkitys on aina rajallinen.

        "Matematiikalla ja logiikalla on suurin hyöty ja arvo ainoastaan suhteessa niihin aineen säännönmukaisuuksiin ja se matematiikan hyöty vähenee sitä mukaa kuin ne säännönmukaisuudet vähenevät siirryttäessä mekaanis-automaattisesta ontologiasta intention (=tahdon) ohjaamaan ontologiaan eli tietoisen itseohjautuvuuden tasolle,"

        Matematiikkaa käytetään mallinnuksessa, mutta paikkansapitävät todellisuuden mallit perustuvat empiirisiin havaintoihin/mittauksiin. Itseasiassa, matemaattiset prosessit, joissa kuvauksia muunnetaan muodosta toiseen, eivät tuota edes oikeasti lisää informaatiotakaan, vaan kyse on vain siitä, että syötetietoja muunnetaan sellaiseen muotoon, että tulokset ovat niiden tulkitsijoille jossain suhteessa *käyttökelpoisempia*.

        En kuitenkaan usko siihen, että olisi *välttämättä* estettä sille, etteikö myös noita ns. "tahdon tai tietoisen itseohjautuvuuden" prosesseja voitaisi kuvata matemaattisesti, vaan mallinnukseen liittyvä ongelma on tuolta osin ehkä ainakin ensisijaisesti se, että noita prosesseja kuvaavaa *tietoa* ei ole riittävästi käytettävissä.

        Käsitteistö pitäisi myös saada paljon yksiselitteisemmäksi tältä osin, sillä jos tahto määritellään esim. "järjestelmän kyvyksi jatketusti suhteellisen vakioituun käyttäytymiseen tiettyjen ympäristötekijöiden vallitessa", niin eihän se edes edellytä tietoisuutta tai itseohjautuvuutta, vaan on pikemminkin niiden negaatio, eli itseasiassa esim. "panssaroidulla leivänpaahtimella" on tuolloin paradoksaalisesti hyvinkin kehittynyt; tai siis jo alunperinkin "vahva", tahto. :D


      • Anonyymi
        Järkisyitä kirjoitti:

        ”Tietenkin totuudet; kuten kaikki muutkin asiat; ovat *vähintään myös* fyysisiä asioita,”

        Olen eri mieltä. Totuus on väittämä-lauseen ominaisuus eikä fyysinen asia. Esimerkiksi matemaattiset lauseet ovat joko tosia tai epätosia. Fyysisillä asioilla kuten vaikkapa kivellä ei ole totuusarvoa.

        Jos tarkoitit totuudella olemassa olemista, niin se on asia, joka meidän pitäisi määritellä. Kuten sanoin matemaattiset totuudet ovat tietyllä tavalla olemassa, mutta huomaan, että sinä määrittelet olemassa olon seuraavasti:
        Vain asiat joilla on energiaa (=kykyä tehdä työtä fyysisessä todellisuudessa) ovat olemassa.

        Tuon määritelmän mukaisesti matematiikkaa ja loogisia totuuksia ei ole olemassa, mutta minun mielestäni ne ovat kuitenkin olemassa, vaikka niiden olemassa olo ei olekaan fyysistä olemista.

        Jos määrittelemme olemassa olon asiaksi, jonka tietoiset olennot voivat havaita ja se on kaikille havaitsijoille samaa (ei siis vain oma kuvitelma), niin matematiikka on olemassa.

        Nimimerkki: Jumalan-kumous (ei-rekisteröitynyt professori)

        Jumalan-kumous: ”Tietenkin totuudet; kuten kaikki muutkin asiat; ovat *vähintään myös* fyysisiä asioita,”
        Järkisyitä: "Olen eri mieltä. Totuus on väittämä-lauseen ominaisuus eikä fyysinen asia. Esimerkiksi matemaattiset lauseet ovat joko tosia tai epätosia. Fyysisillä asioilla kuten vaikkapa kivellä ei ole totuusarvoa. Jos tarkoitit totuudella olemassa olemista, niin se on asia, joka meidän pitäisi määritellä."

        Keskustelumme voisi periaatteessa edetä optimaalisemminkin, jos siis sitä ei olisi determinoitu etenemään juuri kuten se on determinoitu etenemään - valitettavasti. Tarkoitukseni ei ole jäädä käymään "juupas-eipäs" -keskustelua *loputtomasti* mistään tietystä kysymyksestä, vaan lähinnä pyrkiä löytämään sellaisia *uusia* todellisuuden kuvauksia, joilla sitä saadaan kuvattua entistä paremmin, eli pyrkimyksenä on tältä osin tietynlainen dialektiikka, eli että kyettäisiin muodostamaan sellaista synteesiä, joka tuottaisi jonkinlaista *lisäarvoa* tuossa suhteessa...

        Mahdollisesti etenemme tuohon suuntaan, mutta mahdollisesti emme riittävän nopeasti, jotta pääsisimme eteenpäin yhtä nopeasti, kuin voisimme päästä joillakin sellaisilla alueilla, joilla näkemyksemme ovat vähemmän erilaiset tai keskusteltaessa sellaisten tahojen kanssa, joilla on vähemmän erilaiset näkemykset ko. asiasta.

        Eli, totuus tosiaan on *määritelmällisesti* esim. väittämä-lauseen ominaisuus, mutta mitään väittämä-lauseita; kuten mitään muitakaan asioita, ei voi olla olemassa, ilman että *myös* niiden edellytykset ovat olemassa, eikä ole olemassa mitään *empiiristä* näyttöä siitä, että jotain tai edes mitään, voisi olla olemassa ilman edellytyksiänsä, koska koskaan ei olla havaittu mitään ilman, että käytettävissä on ollut mm. havainnot mahdollistavat fyysiset edellytykset; eli esim. aistit, mittalaitteet, aivot jne. ja siitä taas, että emme edes esim. *tietäisi* sitä, että *mitkä* nuo edellytykset ovat; mitä ne sitten ovatkin, ei tietenkään seuraisi, etteikö niitä olisi. Sellaiset asiat, jotka olisivat kausaalisesti *täysin riippumattomia* siitä, että mitä havaitsemme, olisivat kannaltamme täysin merkityksettomiä.

        Toisaalta, en tietenkään tarkoita totuudella *olemassaolemista sinällään*, vaan päinvastoin totuuskaan; kuten mikään muukaan, ei voi olla olemassa ilman edellytyksiänsä ja jos taas sitä ei olisi olemassa, niin silloin se olisi kaiketi korkeintaan pelkkä illuusio, eli esim. "vapaan tahdon" kaltainen asia, mutta et ilmeisesti kuitenkaan edes yritäkään väittää, että se olisi vain illuusio…

        Filosofeilla on; luonnollisesti, totuudesta erilaisia näkemyksiä ja eräs on ns. totuuden korrespondenssiteorian mukainen, eli sen mukaan totuus on; karkeasti ilmaisten, "kuvauksen ja kuvauksen kohteen välisen suhteen sellainen vastaavuus, joka vastaavuus on jollakin tavalla käyttökelpoinen". Käytännössä tuo vastaavuus ei kuitenkaan koskaan voi olla täydellinen, koska kuvaus on *vain* kuvaus, eikä kuvauksen kohde; eli ne ovat *laadullisesti* erilaisia asioita, johon liittyen varsin tunnettu ja hauska esimerkki on se Rene Magritte:n piipun kuva. Tämä seikka liittyy myös tuohon mainittuun ns. "tietoisuuden vaikeaan ongelmaan".
        https://en.wikipedia.org/wiki/The_Treachery_of_Images

        "Kuten sanoin matemaattiset totuudet ovat tietyllä tavalla olemassa, mutta huomaan, että sinä määrittelet olemassa olon seuraavasti: Vain asiat joilla on energiaa (=kykyä tehdä työtä fyysisessä todellisuudessa) ovat olemassa."

        Aivan, mutta jos/koska matemaattiset totuudet ovat olemassa, niin *myös* niillä on; tietyssä merkityksessä energiaa, eli nekin ovat fyysisiä. Ne ovat kuitenkin eri asia kuin edellytyksensä, jotka nekin tietenkin ovat fyysisiä. Havaitsijat *tulkitsevat* fyysistä maailmaa tietyillä tavoilla, joista tavoista eräs voi olla esim. se, että *tulkitaan* tietty fyysinen asia; kuten esim. vaikka jokin tietty kivi, esim. "totuuden kuvaukseksi" tai jonkin "väitelauseen totuusarvoksi"... Hakoteille mennään siis silloin, jos oletetaan, että totuus, totuusarvot jne. voisivat olla olemassa *irrallaan* edellytyksistänsä; ne eivät voi olla, vaikka ne tietenkin onkin; mm. käyttötarkoituksestansa johtuvista esitysteknisistä ja abstrahointisyistä, usein *esitetty* irrallaan edellytyksistänsä...


      • Anonyymi
        Järkisyitä kirjoitti:

        "Fyysisen todellisuuden olemassaolo ei edellytä havaitsemista,"

        Samaa mieltä. Vain äärimmäiset idealistit eivät hyväksy tuota asiaa. Voihan olla niin, että fyysinen todellisuus mallinnetaan vasta siinä vaiheessa, kun havaitsija sitä katsoo - eikä fyysinen todellisuus ole olemassa havaintojen välillä. Se *vaikuttaa* jatkuvalta, muttei ole.

        "mutta matemaattisen totuuden; ja tietysti muidenkin *todellisuuden kuvausten* olemassaolo, edellyttää fyysistä todellisuutta"

        Tuo on pelkkä väittämä. Yhtä hyvin voin väittää Max Tegmarkin tavoin, että fyysisen todellisuuden olemassa olo edellyttää matemaattisen logiikan olemassa oloa.

        Otan vertauskuvan: Edellyttääkö tietokone ohjelman olemassa olo sitä, että on olemassa prosessori joka tulkitsee sitä? Nähdäkseni väität, että logiikka on olemassa vain, jos on olemassa fyysinen todellisuus, jossa sitä tulkitaan. Minä taas katson, että ohjelma/logiikka on olemassa myös silloin, kun sillä ei ole tulkitsijaa.

        Nimimerkki: Jumalan-kumous (ei-rekisteröitynyt professori)

        Jumalan-kumous: "mutta matemaattisen totuuden; ja tietysti muidenkin *todellisuuden kuvausten* olemassaolo, edellyttää fyysistä todellisuutta"

        Järkisyitä: "Tuo on pelkkä väittämä. Yhtä hyvin voin väittää Max Tegmarkin tavoin, että fyysisen todellisuuden olemassa olo edellyttää matemaattisen logiikan olemassa oloa. Otan vertauskuvan: Edellyttääkö tietokone ohjelman olemassa olo sitä, että on olemassa prosessori joka tulkitsee sitä? Nähdäkseni väität, että logiikka on olemassa vain, jos on olemassa fyysinen todellisuus, jossa sitä tulkitaan. Minä taas katson, että ohjelma/logiikka on olemassa myös silloin, kun sillä ei ole tulkitsijaa."

        Jokainen voi tietenkin väittää vaikka kuinka kummallisia väitteitä - *jos* fyysinen todellisuus sen sallii; tai siis itseasiassa, jos se siihen *pakottaa*…, mutta esim. tuohon Tegmark:in hypoteesiin liittyy lisäksi siis se kritiikki, johon annoin sen linkin, johon kritiikkiin kannattaa tutustua mm. koska se on varsin "painavaa"... ja lisäksi, koska; kuten jo totesin, koskaan ei siis olla *havaittu* todellisuuden kuvauksia irrallaan fyysisistä edellytyksistänsä, niin tuollaisten "irrallisten platonisten olioiden" olettaminen ei ole *käytännön kannalta*; ja siis myöskään tieteen edistämisen kannalta, perusteltua, mm. koska Occam:in periaatteen mukaisesti, todellisuutta selittävien mallien sisältämien ns. ylimääräisten entiteettityyppien määrä kannattaa minimoida, jos ja tässä tapauksessa siis myös kun, todellisuus on selitettävissä yhtä hyvin ilmankin noita; todellisuudesta *irrallisia* ja siten itseasiassa redundantteja entiteettityyppejä...

        Todelliset tietokoneohjelmat ovat *fyysisiä* ja niiden olemassaolo edellyttää mm. fyysisen muistivälineen olemassaolon, mutta ei ohjelman tulkitsijan olemassaoloa; joskin tietysti käytännössä nykyisenkaltaiset ohjelmat usein ovat ihmisten tekemiä, jolla seikalla tosin tässä ei ole varsinaista relevanssia. Oleellista tässä sen sijaan on se, että pelkkä *abstrakti* tietokoneohjelma/algoritmi ei kykenisi tuottamaan niitä toiminnallisuuksia, jotka ovat sen määritelmällisiä ominaisuuksia. Yleisemmässä katsannossa, olemassa ei ole myöskään mitään informaatiota, ilman fyysisiä edellytyksiänsä...

        Esim. totuuden korrespondenssiteorian mukaisen totuuden, eli kunkin: "kuvauksen ja kuvauksen kohteen välisen suhteen sellaisen vastaavuuden, joka vastaavuus on jollakin tavalla käyttökelpoinen", olemassaolo toisaalta kylläkin edellyttää myös todellisuuden tulkitsijaa, koska tuo käyttökelpoisuus edellyttää sen määrittelemistä, että mikä on käyttökelpoista, mikä taas edellyttää todellisuuden tulkitsijaa, eli käytännössä vähintään jotain sellaista *fyysistä prosessia*, joka kykenee havainnoimaan ja arvioimaan todellisuutta vähintään jossakin laajuudessa jne.


      • Anonyymi
        Anonyymi kirjoitti:

        "En vetäisi yhtäläisyysmerkkiä fysikalismin ja materian arvostamisen välillä. "

        Joo ei se ihan niinkään ole. Sanotaan nyt vaikka niin että minusta näyttää että materialisti hahmottaa kokemaansa kielellisten käsitteiden avulla erilaisiin "laatikoihin" joista yhden laatikon kyljessä lukee "materia" ja toisen laatikon kyljessä lukee "henki" ja sinne materia-lootaan lajitellaan kaikki se kokemus mikä tuntuu konkreettiselta ja todelliselta ja sinne henki-lootaan taas kaikki se kokemus mikä tuntuu jotenkin epätodelliselta, haamumaiselta, abstraktilta yms.

        Idealisti taas käyttää vain yhtä laatikkoa koska kaikki mahdolinen on aina kokemusta tai datastreamiä tai mitä käsitettä tässä yhteydessä on tottunut käyttämään.

        Sitten on toisenlainen datastream (stream 2) jota kutsutaan "unimaailmaksi" sen takia että se on yleensä vähemmän pysyvä ja sen konteksti vaihtelee yleensä jokaisessa unessa vaikka joskus ainakin minä näen ns. jatkounia jotka jatkavat siitä mihin se edellinen uni jäi joskus aikaisemmin eikä välttämättä sen ykkös datastreamiin (valvekokemus) suhteutettuna ajallisesti peräkkäin seuraavana yönä vaan välissä voi olla viikkoja tai kuukausia.

        Molemmat datastreamit tuntuvat todelliselta ja molemmissa voi halutessaan hahmottaa osan sen datastreamin komponenteista "aineeksi" ja toisen osan "hengeksi".

        Mielestäni Tom Campbell osaa selittää tämän varsin hyvin omassa kaiken teoriassaan vaikka en olekaan ihan kaikesta täysin samaa mieltä.

        https://www.youtube.com/watch?v=zstsJ7-ULM0

        Mielestäni kaikissa mahdollisissa selitysmalleissa ovat heikkoutensa varsinkin jos ne yritetään ymmärtää jotenkin kirjaimellisesti. Minusta erilaiset mallit ovat lähinnä käsitteellisiä työkaluja joista kukin tavoittaa vain joitakin aspekteja ns. todellisuudesta.

        Itse kannatan tällä hetkellä kolmea tai neljää hieman erilaista mallia samanaikaisesti koska ne ovat toisiaan täydenteviä


        1) Tom Campbellin simulaatiomalli
        2) Bernardo Kastrupin moderni idealismi
        3( Gurdjieffin ja J.G. Bennettin suhteellinen materialismi
        4) Joseph P. Farrellin topologinen vertauskuva

        Gurdjieffiin ja Bennettiin perehdyin jo n. 40 vuotta ja noihin kahteen muuhun viimeisen viiden vuoden aikana. Farrelliin n. 10 v sitten ja noista kaikista minulla on melkein kaikki aineisto mitä ylipäätänsä on saatavilla (kymmeniä kirjoja, satoja videoita ja äänitteitä netin kautta kerättynä vuodesta 1995 ).

        Tieteestä aloitin n. 40 v sitten keräilemään lehtileikkeitä ja kirjoja ja tutkimaan varsinkin erilaisia outoja ja anomaalisia ilmiöitä jotka eivät mahdu valtavirtamalleihin. Samoihin aikoihin opettelin ohjelmoimaan ja harrastamaan tietotekniikkaa kausittain varsin intensiivisesti. Lopputulos on se mitä mieltä tällä hetkellä olen ja voi olla että sekin kanta muuttuu ainakin jonkin verran vielä niin kauan kuin kykenen oppimaan uutta ja perehtymään asioihin.

        Belisario

        Nimimerkki: Jumalan-kumous (ei-rekisteröitynyt professori)

        Belisario: "Itse kannatan tällä hetkellä kolmea tai neljää hieman erilaista mallia samanaikaisesti koska ne ovat toisiaan täydenteviä 1) Tom Campbellin simulaatiomalli 2) Bernardo Kastrupin moderni idealismi 3( Gurdjieffin ja J.G. Bennettin suhteellinen materialismi 4) Joseph P. Farrellin topologinen vertauskuva"

        Gurdjieff mainittu ja "vapaalla vaihteella" siis vahvasti mennään, mutta "eksoottisia katsantokantoja" on niin monia erilaisia, että tässä on ns. "valinnan vaikeus"; tai siis ei oikeastaan ole, koska kaikkihan on determinoitua, mutta ymmärtänet, että mitä tarkoitan, eli että mitkä perusteet pakottavat valitsemaan tietyn "eksoottisen" näkökulman?

        Miten suhtaudut esim. Carlos Castaneda:n esiintuomaan maailmankuvaan ja menetelmiin? Menemme tässä "vähän" "sivuun ja yli" ketjun aiheesta ja oikeastaan "tosi reippaastikin", mutta tämä "eksoottisiin" maailmankuviin liittyvä teema on kiinnostava. CC:n esittämien menetelmien tarkoitushan on ilmeisesti sama kuin Gurdjieff:in, eli "tietoisuuden tason kehittäminen", joka toki on relevanttia, kunhan siis tuo asia määritellään jollain järkevällä tavalla; mikä tietysti voi olla haasteellista. Esim. itse olen kiinnostunut havaitsemisen edellytyksistä. CC:n esittämä maailmankuva vaikuttaa *hyvin* kattavalta ja ns. sisäisesti konsistentilta todellisuuden kuvaukselta; riippumatta siitä, että mitkä syyt/olosuhteet ovat johtaneet sen muodostukseen.

        Mielestäni se sopii yllättäen myös varsin hyvin yhteen ns. "empiirisen tieteellisen lähestymistavan" kanssa; joskin toki subjektiivisella kokemusperäisellä "eksoottisella twist":llä, mikä merkitsee sitä, että tuon aiemmin mainitsemani objektiivisen todentamisen osalta tarvittaisiin uudenlaisten menetelmien kehittämistä ja CC:n kirjoissa ei ilmeisesti myöskään esiinny mitään erityisiä matemaattisia ilmauksia; eli ilmaisutapa on sellainen, että vaikka sisältö olisikin adekvaattia, niin niiden oikea tulkintatapa voi edellyttää "konsultaatiota" monien "liittolaisten" kanssa tai jopa myös sienien käyttöä; kuten CC:n itsensä tapauksessa. CC:n menetelmien mukainen toimintatapa kuitenkin tarjoaisi ihmisille tietysti myös mm. kaipaamiasi "henkilökohtaista mukavuutta ja helppoutta karttavia virikkeitä"; lievästi ilmaisten. Lisäksi CC:n maailmankuvassa korostetaan tahdon/intention merkitystä; kuten mm. itsekin teet...

        Onkin varsin valitettavaa, että CC:n maailmankuvaan on kohdistettu niin paljon mm. "ad hominem" -tyyppistä; eli päättelyvirheisiin perustuvaa, epärelevanttia kritiikkiä mm. tiedeyhteisön taholta, eli mm. sen tutkimista, että oliko CC vai eikö hän ollut "käynyt kirjastossa" tiettynä päivänä, jolloin hän ilmoittikin olleensa jossakin muualla... Maailmankuva on; sarkastisesti ilmaisten, "tietenkin" "väärä", jos esim. tuossa suhteessa on havaittavissa "anomalioita" ja etenkin, jos ja tässä tapauksessa siis myös vahvasti kun, se ei ole ollenkaan yhteensopiva; edelleenkin, etenkin länsimaissa vallitsevan "populistisen länsimaisen ihmiskäsityksen" kanssa, vaikka ja siis myös kun se sisältää *mm.* käsityksen "reaalisesti vaikuttavasta bilokaatiosta"... :D
        https://en.wikipedia.org/wiki/Bilocation


      • Anonyymi
        Anonyymi kirjoitti:

        Nimimerkki: Jumalan-kumous (ei-rekisteröitynyt professori)

        Belisario: "Minusta ns. matemaattiset ja loogiset totuudet eivät mitenkään luonteeltaan poikkea muista ihmiskunnan keksinnöistä ja innovaatioista."

        Ainakaan ne eivät poikkea toisistaan sen osalta, että niillä siis on fysikaalisia edellytyksiä ja mm. niiden havaittaviksi tuottaminen edellyttää myös tietyn tuotantotekniikan, vaikka tietysti nuo nimenomaiset edellytykset ja tekniikat ovat vähintään osin erilaiset. Paul Erdös muistaakseni totesi joskus mm., että matemaatikko on kahvia teoreemoiksi muuttava kone. :D

        "Tässä minusta on ihan sama analogia kuin kaksoisrakokokeen viivästetyssä versiossa […]".

        Olen samoilla linjoilla siltä osin, että erilaiset näkökulmat todellisuuteen voivat tosiaan erilaisissa tilanteissa olla selvästi toimivimpia. En nyt muista, että kuka sen sanoi, mutta idea meni jotenkin näin ikään, että: "An academic is a person who tries to create models describing reality each day meticulously from 8 AM to 4 PM and then uses rest of his/her time in trying to forget those very same models…" :D …

        En kuitenkaan suinkaan menisi niin pitkälle, että väittäisin, että todellisuus olisi olemassa *vain* tiedostettuna, vaan vain että tietyn toimijan itsensä kannalta *relevantteja* ovat vain ne todellisuuden osat/piirteet, jotka tai joiden vaikutukset hän; vähintään jossain vaiheessa, tiedostaa, minkä seikan oikeaan-osuva ennakointi voi tietenkin olla hyvinkin haasteellista…

        Itseasiassa, jos todellisuus on sellainen kone, joka käy läpi kaikki tilansa jossain järjestyksessä; eli "toteuttaa omat vapausasteensa", niin *kaikki* todellisuuden osat/piirteet ovat relevantteja kaikille tietoisille olennoille, vaikka ne; väistämättömistä kognitiivisista rajoitteistaan johtuen, eivät kykenekään välttämättä koskaan *yhdistämään* noita; tuossa suhteessa irrallisia tietoisia kokemuksiaan, toisiinsa ainakaan tietoisesti...

        "Tämä selittäisi mm. jotkut ns. parapsykologiset ilmiöt kuten esim. levitaatiot joita on havaittu suhteellisen usein esim. Intiassa ja länsimaissakin muutaman kerran ( esim. Daniel Dunglas Home ja eräs keskiaikana elänyt munkki joka haltioitui messun aikana niin paljon että alkoi liidellä lähellä kirkon kattoa:D)"

        Mietiskelinkin, että miksi "mielessäni pyöri" sana "pyhä", eli esim. "pyhän" Joosef Kupertinolaisen väitetään kyenneen levitoimaan, mutta tuollaisten ns. ihmeiksi tapahtuma-aikoinaan luokiteltujen tapahtumien raportoinnit näyttävät vähentyneen ajan funktiona, mikä voi olla selitettävissä yksinkertaisesti esim. sitä kautta, että tuontyyppisiä tapahtumia on aikoinaan *tulkittu* sen mukaan, mitä kulloinkin on pidetty mahdollisena.

        Jos tarkastelemme erilaisten selitysmallien paikkansapitävyyden suhteellista todennäköisyyttä, niin on ilmeistä, että on *huomattavasti todennäköisempää*, että tuonkaltaisissa tapahtumissa on kyse juurikin siitä, että *tulkitaan* fyysisiä vaikutuksia esim. lentäviksi munkeiksi, kuin se, että munkit lentäisivät *fyysisesti*.

        Eli, on ilmeistä, että vaikka esim. tiettyyn munkkiin liittyisikin mm. "outoja ja aivan reaalisiakin ja epätavallisia korrelaatteja"; kuten esim. minuun ja Lenin:iinkin liittyy, niin on silti huomattavasti paljon todennäköisempää esim. että joukko "havainnoitsijoita" näkee samanaikaisesti samansisältöistä unta siitä, että esim. tietty munkki lentää fyysisesti, kuin se, että kyseinen munkki lentäisi fyysisesti...

        "Ilmaisen tämän (tietysti taas) aika kiistanalaisella tavalla koska minun kannattamassani ontologiassa ns. aine on alisteista ns. tietoisuudelle eli aineellinen maailma tavallaan sijaitsee tietoisuuden maailman sisällä eikä niin kun fysikalismissa jossa tietoisuus ja kaikki mahdollinen kokemus sijaitsee ihmisten kallon sisällä fysikaalisten aivojen neuronien keskinäisessä vuorovaikutuksessa ja rakenteessa."

        Tuolta osin kannattaa huomata se, että mm. aivojen ja havaittavan todellisuuden rakenne yleisemmässäkin katsannossa on *äärimmäisen* epäsatunnainen, koska mitään aivoja ei ylipäätään keskimäärin pitäisi tilastollisesti olla havaittavissakiin ja se on relevanttien tekijöiden osalta itseasiassa *juuri* "oikeanlainen" mm., jotta juuri tämän ja vain tämän lauseen tuottaminen juuri tässä asiayhteydessä oli mahdollinen ja myös, jotta mm. sen sisällön tiedostaminen oli mahdollista…

        Itse edustan sitä kantaa, että oikeastaan kaikki havaitsemamme todellisuuden aspektit ovat; siis jo lähtökohtaisestikin, eli ennen tuota ns. "tietoisuuden kehittämistä", ns. "creme de la creme":ä, eli *äärimmäisen valikoituneita*. En usko, että tuohon valikoitumiseen tarvitaan tietoisuutta tai että tietoisuus kykenisi tuollaista valtavaa urakkaa ylipäätään suorittamaankaan, vaan tietoisuus on vain seuraus ja itseasiassa vain sivutuote tuosta *tavattomasta valikoituneisuudesta*, joskin *yksilöiden subjektiivisten kokemusten* kannalta tietysti eräs hyvin oleellisista aspekteista...

        "En kuitenkaan suinkaan menisi niin pitkälle, että väittäisin, että todellisuus olisi olemassa *vain* tiedostettuna, vaan vain että tietyn toimijan itsensä kannalta *relevantteja* ovat vain ne todellisuuden osat/piirteet, jotka tai joiden vaikutukset hän; vähintään jossain vaiheessa, tiedostaa, minkä seikan oikeaan-osuva ennakointi voi tietenkin olla hyvinkin haasteellista… "

        Jos perimmäinen ontologia on jonkinlainen tietoisuus tai oikeastaan informaatio ("it from bit") niin ei se tarkoita että se on kokonaisuudessaan tiedostettua vaan aina ainoastaan vain hyvin pieneltä osalta. Itsetuntemus on aina paljon vaikeampaa kuin esim. jonkun toisen hyvien ja huonojen puolien tiedostaminen eli huomaamme toisissa helpommin sellaisia asioita joita emme tavallisesti huomaa itsessämme.

        Tarkkaavaisuus tai huomaamisen kyky on kuin lamppu joka valaisee vain osan mielemme ja ulkomaailman ominaisuuksista ja aspekteista. Etsimme usein kuvainnollisesti avaimiamme katulampun alta missä on valoisinta eikä sieltä minne oikeasti hukkasimme ne avaimet:D

        "Itseasiassa, jos todellisuus on sellainen kone, joka käy läpi kaikki tilansa jossain järjestyksessä; eli "toteuttaa omat vapausasteensa", niin *kaikki* todellisuuden osat/piirteet ovat relevantteja kaikille tietoisille olennoille."

        Evoluutiossa oikeastaan käydään läpi kaikki mahdolliset vaihtoehdot ja vain toimivat vaihtohdot jäävät jäljelle.

        Oikeastaan ainoat välttämättömät perusoletukset ovat että jonkinlainen tietoisuus on olemassa sekä evoluution prosessi. Se tietoisuus voi alkuvaiheessaan olla hyvin yksinkertainen (esim. nautintoon pyrkiminen ja epämukavuuden välttäminen) eli siis jonkinlainen prosessifraktaali joka vähitellen tuottaa yhä monimutkaisempia kokonaisuuksia ja vähitellen ne kokonaisuudet erilaistuvat ja muodostavat toisilleen erilaisia ympäristöjä.

        Tietoisuus on oikeastaan sisäisen ja ulkoisen maailman rajapinta josta sitten vähitellen muodostuu hermosto ja aivot jos asiaa tarkastellaan 3. persoonan näkökulmasta.

        "tuollaisten ns. ihmeiksi tapahtuma-aikoinaan luokiteltujen tapahtumien raportoinnit näyttävät vähentyneen ajan funktiona, mikä voi olla selitettävissä yksinkertaisesti esim. sitä kautta, että tuontyyppisiä tapahtumia on aikoinaan *tulkittu* sen mukaan, mitä kulloinkin on pidetty mahdollisena. "

        Tottakai ne ovat vähentyneet koska niihin on alettu suhtautua kielteisesti ja muutenkin pidetty täysin mahdottomana varsinkin 1800-luvun jälkeen länsimaissa. Harvemmin kukaan edes yrittää puhumattakaan sitä joskus jopa onnistuisi tekemään sellaista mikä on omasta ja enemmistön mielestä sekä mahdotonta että myös usein hyvin paheksuttavaa ja pelottavaa.

        "on silti huomattavasti paljon todennäköisempää esim. että joukko "havainnoitsijoita" näkee samanaikaisesti samansisältöistä unta "

        Kyse voi tietysti olla jonkinlaisesta joukkohypnoosista mutta minusta voi olla myös mahdollista että voidaan luoda ikäänkuin jonkinlainen todellisuuskupla jossa samanmieliset voivat tuottaa anomaalisia ilmiöitä.

        Kuten olen aikaisemmin maininnut niin tämä fysikaalinen maailma voidaan hahmottaa unen kaltaisena koska emme siinä unessa yleensä edes tiedosta olevamme unessa ennenkuin heräämme. Jos unennäkö (ja varsinkin ns. selkounet) on jonkinlainen simulaatio niin miksei tämä ns. fysikaalinen maailmakin voisi olla jonkinlainen unenkaltainen simulaatio mutta ei yksilön mielen tuottama vaan kollektiivisen tajunnan tuotos.

        "tietoisuus kykenisi tuollaista valtavaa urakkaa ylipäätään suorittamaankaan,"

        En mitenkään tietoisesti kykenisi tuottamaan sellaisia niin sisällöltään ja yksityiskohdiltaan rikkaita unia kuin usein näen. Ne unet ovat usein jopa todellisemman tuntuisia kuin suurin osa valve-elämästä.

        Belisario


      • Anonyymi kirjoitti:

        "En vetäisi yhtäläisyysmerkkiä fysikalismin ja materian arvostamisen välillä. "

        Joo ei se ihan niinkään ole. Sanotaan nyt vaikka niin että minusta näyttää että materialisti hahmottaa kokemaansa kielellisten käsitteiden avulla erilaisiin "laatikoihin" joista yhden laatikon kyljessä lukee "materia" ja toisen laatikon kyljessä lukee "henki" ja sinne materia-lootaan lajitellaan kaikki se kokemus mikä tuntuu konkreettiselta ja todelliselta ja sinne henki-lootaan taas kaikki se kokemus mikä tuntuu jotenkin epätodelliselta, haamumaiselta, abstraktilta yms.

        Idealisti taas käyttää vain yhtä laatikkoa koska kaikki mahdolinen on aina kokemusta tai datastreamiä tai mitä käsitettä tässä yhteydessä on tottunut käyttämään.

        Sitten on toisenlainen datastream (stream 2) jota kutsutaan "unimaailmaksi" sen takia että se on yleensä vähemmän pysyvä ja sen konteksti vaihtelee yleensä jokaisessa unessa vaikka joskus ainakin minä näen ns. jatkounia jotka jatkavat siitä mihin se edellinen uni jäi joskus aikaisemmin eikä välttämättä sen ykkös datastreamiin (valvekokemus) suhteutettuna ajallisesti peräkkäin seuraavana yönä vaan välissä voi olla viikkoja tai kuukausia.

        Molemmat datastreamit tuntuvat todelliselta ja molemmissa voi halutessaan hahmottaa osan sen datastreamin komponenteista "aineeksi" ja toisen osan "hengeksi".

        Mielestäni Tom Campbell osaa selittää tämän varsin hyvin omassa kaiken teoriassaan vaikka en olekaan ihan kaikesta täysin samaa mieltä.

        https://www.youtube.com/watch?v=zstsJ7-ULM0

        Mielestäni kaikissa mahdollisissa selitysmalleissa ovat heikkoutensa varsinkin jos ne yritetään ymmärtää jotenkin kirjaimellisesti. Minusta erilaiset mallit ovat lähinnä käsitteellisiä työkaluja joista kukin tavoittaa vain joitakin aspekteja ns. todellisuudesta.

        Itse kannatan tällä hetkellä kolmea tai neljää hieman erilaista mallia samanaikaisesti koska ne ovat toisiaan täydenteviä


        1) Tom Campbellin simulaatiomalli
        2) Bernardo Kastrupin moderni idealismi
        3( Gurdjieffin ja J.G. Bennettin suhteellinen materialismi
        4) Joseph P. Farrellin topologinen vertauskuva

        Gurdjieffiin ja Bennettiin perehdyin jo n. 40 vuotta ja noihin kahteen muuhun viimeisen viiden vuoden aikana. Farrelliin n. 10 v sitten ja noista kaikista minulla on melkein kaikki aineisto mitä ylipäätänsä on saatavilla (kymmeniä kirjoja, satoja videoita ja äänitteitä netin kautta kerättynä vuodesta 1995 ).

        Tieteestä aloitin n. 40 v sitten keräilemään lehtileikkeitä ja kirjoja ja tutkimaan varsinkin erilaisia outoja ja anomaalisia ilmiöitä jotka eivät mahdu valtavirtamalleihin. Samoihin aikoihin opettelin ohjelmoimaan ja harrastamaan tietotekniikkaa kausittain varsin intensiivisesti. Lopputulos on se mitä mieltä tällä hetkellä olen ja voi olla että sekin kanta muuttuu ainakin jonkin verran vielä niin kauan kuin kykenen oppimaan uutta ja perehtymään asioihin.

        Belisario

        ”Sanotaan nyt vaikka niin että minusta näyttää että materialisti hahmottaa kokemaansa kielellisten käsitteiden avulla erilaisiin "laatikoihin" joista yhden laatikon kyljessä lukee "materia" ja toisen laatikon kyljessä lukee "henki" ja sinne materia-lootaan lajitellaan kaikki se kokemus mikä tuntuu konkreettiselta ja todelliselta ja sinne henki-lootaan taas kaikki se kokemus mikä tuntuu jotenkin epätodelliselta, haamumaiselta, abstraktilta yms.”

        En tunnista tuosta kuvauksesta ollenkaan omaa ajatustapaani. Kuulostaa enemmänkin dualistiselta käsitykseltä.

        Materialistille kaikki koostuu materiasta, mutta joudumme käsittelemään asioita eri abstraktio-tasoilla. Esimerkiksi sinä koostut vain erillisistä atomeista, mutta tästä huolimatta ajattelemme sinua kokonaisena ihmisenä, joka on tuolle atomijoukolle annettu nimike tai oikeammin korkeamman tason malli: abstraktio. Emme juuri koskaan ajattele asioita atomitasolla, koska ymmärryksemme ei riitä. Aivan samoin emme voi ajatella kokemusmaailmaa neuroverkkona vaan joudumme käsittelemään sitä erilaisten abstraktioiden kautta. Kokemusmaailmaa käsittelemme omista kokemuksistamme käsin, vaikka uskommekin senkin olevan neuroverkon tuotos.


      • Anonyymi
        Anonyymi kirjoitti:

        Nimimerkki: Jumalan-kumous (ei-rekisteröitynyt professori)

        Belisario: "Itse en usko superdeterminismiin ja hahmotan ns. aineenkin hierarkkisena ja laadullisesti tiettyjen aineen aspektien suhteen suhteellisena ontologiana jossa tietoisuus on korkeamman tasoista aine-energiaa jossa on matalampi entropia ja vähemmän säännönmukaisuuksia (=luonnonlakeja) kuin aistein havaittavissa olevalla aineella."

        Yllättävää, että pidät siis tietoisuuttakin *aineena/energiana* ja tuo kohta kuulostaa mielenkiintoiselta, mm. koska jos tietoisuus on ainetta/energiaa, niin sitä voidaan tutkia ja mitata objektiivisesti, mikä tietenkin avaa hyvin mielenkiintoisia mahdollisuuksia tieteen edistämiseen. En hahmota, että mitä tarkoitat sillä, että tietoisuuteen liittyisi *sekä* matalampi entropia, että *vähemmän* säännönmukaisuuksia, koska entropian *puutehan* juuri tarkoittaa säännönmukaisuuksia.

        Mielestäni olet yksinkertaisesti väärässä siltä osin, jos/kun oletat, että todellisuudessa olisi olemassa jotain *todellisia* vapausasteita, koska kaikki on riippuvaista edellytyksistänsä, minkä olen sanonut lukemattomia kertoja ja tiedän, että toistaminen ei ole validi argumentointitapa, mutta kun muutkin toistavat omia teesejään, niin on vain tasapuolista, että minäkin toistan mm. tuota. Vapaus on valitettavasti niin epämääräinen käsite, että kun sitä käytetään, niin on helppo mennä "ihan metsään", ilman että sitä edes huomaakaan. Superdeterminismiin toisaalta päädytään sitä kautta, että suljetaan pois sellaisia mahdottomia/mielettömiä vaihtoehtoja joihin esim. sisältyy implisiittisesti tyhjästä syntyviä vaikutuksia.

        Mielestäni kannattaisikin kuviteltuun *vapauteen* "ripustautumisen* sijasta keskittyä tavoitteiden kannalta oleellisten *resurssien* kuvaamiseen, eli esim. niiden, joilla voi olla oleellista vaikutusta siihen, että missä määrin kykenee tiedostamaan todellisuutta… Kyse on siis pikemmin vain siitä, että mitkä omaan toimintaansa *vaikuttavat tekijät* kykenee tiedostamaan käytettävissään olevien resurssien puitteissa, jotka resurssit ovat mm. energiaan ja tietojenkäsittelyyn liittyviä. Lisäksi, koska noita vaikuttavia yksittäisiä tekijöitä on *loputtomasti*, niin tietoisuuden merkitys on aina rajallinen.

        "Matematiikalla ja logiikalla on suurin hyöty ja arvo ainoastaan suhteessa niihin aineen säännönmukaisuuksiin ja se matematiikan hyöty vähenee sitä mukaa kuin ne säännönmukaisuudet vähenevät siirryttäessä mekaanis-automaattisesta ontologiasta intention (=tahdon) ohjaamaan ontologiaan eli tietoisen itseohjautuvuuden tasolle,"

        Matematiikkaa käytetään mallinnuksessa, mutta paikkansapitävät todellisuuden mallit perustuvat empiirisiin havaintoihin/mittauksiin. Itseasiassa, matemaattiset prosessit, joissa kuvauksia muunnetaan muodosta toiseen, eivät tuota edes oikeasti lisää informaatiotakaan, vaan kyse on vain siitä, että syötetietoja muunnetaan sellaiseen muotoon, että tulokset ovat niiden tulkitsijoille jossain suhteessa *käyttökelpoisempia*.

        En kuitenkaan usko siihen, että olisi *välttämättä* estettä sille, etteikö myös noita ns. "tahdon tai tietoisen itseohjautuvuuden" prosesseja voitaisi kuvata matemaattisesti, vaan mallinnukseen liittyvä ongelma on tuolta osin ehkä ainakin ensisijaisesti se, että noita prosesseja kuvaavaa *tietoa* ei ole riittävästi käytettävissä.

        Käsitteistö pitäisi myös saada paljon yksiselitteisemmäksi tältä osin, sillä jos tahto määritellään esim. "järjestelmän kyvyksi jatketusti suhteellisen vakioituun käyttäytymiseen tiettyjen ympäristötekijöiden vallitessa", niin eihän se edes edellytä tietoisuutta tai itseohjautuvuutta, vaan on pikemminkin niiden negaatio, eli itseasiassa esim. "panssaroidulla leivänpaahtimella" on tuolloin paradoksaalisesti hyvinkin kehittynyt; tai siis jo alunperinkin "vahva", tahto. :D

        "Yllättävää, että pidät siis tietoisuuttakin *aineena/energiana* ja tuo kohta kuulostaa mielenkiintoiselta, mm. koska jos tietoisuus on ainetta/energiaa, niin sitä voidaan tutkia ja mitata objektiivisesti, mikä tietenkin avaa hyvin mielenkiintoisia mahdollisuuksia tieteen edistämiseen."

        Valtavirtatiede tuntee vain 4 vuorovaikutusta joista vain sähkömagnetismi ja gravitaatio ilmenevät arkikokemuksessa ja joissa on periaatteessa selkeä suunta verrattuna esim. lämpöön jossa hiukkaset poukkoilevat edestakaisin satunnaisesti ilman minkäänlaista organisaatiota. Sitten olemassa kemiallisia sidoksia jotka voidaan luokitella korkeamman laatuiseksi energiaksi kuin sähkömagnetismi ja jotka siis organisoivat sähkömagneettisia kokonaisuuksia ja muodostavat kiinteitä kappaleita.

        Biologian tasolla on geenit joiden organisointikyky on selkeästi suurempi kuin "elottoman" maailman kemiallisilla sidoksilla ja joka kykenee aineenvaihduntaan jolla eliö säilyttää rakenteensa ja organisaationsa vaikka kehon atomit ja molekyylit vaihtuvat jatkuvasti.

        Jos tarkkailee taas mielen ilmiöitä niin ainakin tarkkaavaisuus ja siihen liittyvä asioiden huomaaminen ja tiedostaminen ovat selkeästi jonkinlaista energiaa kuluttavia mielen toimintoja samoin oppiminen ja oppimansa ymmärtäminen. Sitten vaikuttaa myös siltä että älylliset, tunne ja tahdonalaisen fyysisen kehon prosessit kukin vaativat omanlaisensa energiaa eli vaikka en jaksaisi opiskella "älykeskuksellani" niin silti voisin jaksaa suorittaa jostain fyysistä askaretta. Sekä äly,tunne että fyysinen puoli voi toimia joko mekaanisesti tiedostamatta tai tietoisesti jolloin siihen tietoiseen toimintaan tarvittava energia on selkeästä korkeampilaatuista kuin siihen mekaanisen tason toimintaan tarvittava energia.

        Voin myös nostaa tarkkaavaisuuteni intensiteettiä jonkinlaisella meditoinnilla jonka ainoa tarkoitus on pysäyttää sisäinen kielellinen yksinpuhelu mahdollisimman pitkäksi aikaa yhteenmenoon jolloin samalla erotamme erilaatuiset psyyken energiat toisistaan.

        Huomaamisen energiaa voidaan kutsua sensitiivienergiaksi jonka yksi perusominaisuus on samaistuminen omiin ajatuksiin, tunteisiin ja omaan kehoon. Jos onnistun tyhjentämään mieleni ja lisäämään tarkkaavaisuutta niin kykenen toimimaan ns. tietoisuuden energialla joka itsessään tuottaa objektiivisia havaintoja eli näemme maailman sellaisena kuin se on eikä sellaisena kuin sensitiivienergian tuottama samaistuminen tuottaa. Tajunnanenergia on pohjimmiltaan passiivista kun taas vielä korkeamman tason luovuuden energia kykenee tuottaa objektiivisesti uutta todellisuuden sisältöä.

        Kaikkien mahdollisten energioiden toimivuudessa pitää ottaa huomioon tarvittava energian määrä, laatu ja intensiteetti (vrt veden keittäminen kiehuvaksi). Energiat laadut varsinkin elollisessa luonnossa voivat muuttua toisikseen eli keho tekee työtä jolloin ylempi laatuinen energia hajoaa alempilaatuiseksi. Jos ruoka, juoma ja happi ylläpitävät kehon fyysistä rakenetta niin aistimukset, kokemukset ja vastaava "hengenravinto" ylläpitää kehon psyykkistä rakennetta.

        Korkeamman tasoisten organisoivien energioiden mittaaminen voi olla hankalaa koska niitä energioita ei voi ainakaan toistaiseksi mitata suoraan vain ainoastaan epäsuorasti vertaamalla siihen mitä puhtaasti satunnainen prosessi kykenisi tuottamaan samassa ajassa. Koska nykyinen tiede ei tunne kuin ne 4 perusvuorovaikutusta niin ei ole ainakaan toistaiseksi olemassa mittausvälineitä jotka kykenisivät tunnistamaan kemiallisen, biologisen tai psyykkisen tason erilaatuisia energiamuotoja eli joudumme tyytymään pelkkään kokemusperäiseen kuvailuun.

        "En hahmota, että mitä tarkoitat sillä, että tietoisuuteen liittyisi *sekä* matalampi entropia, että *vähemmän* säännönmukaisuuksia, koska entropian *puutehan* juuri tarkoittaa säännönmukaisuuksia. "

        Jos entropian hahmottaa epäjärjestyksenä ja sen epäjärjestyksen lisääntymisenä ja vähemmät säännönmukaisuudet sillä että mekaaninen reagointi vaihtuu tiedostetuksi ja harkituksi toiminnaksi niin asia lienee jotenkin ymmärrettävissä.

        "Superdeterminismiin toisaalta päädytään sitä kautta, että suljetaan pois sellaisia mahdottomia/mielettömiä vaihtoehtoja joihin esim. sisältyy implisiittisesti tyhjästä syntyviä vaikutuksia."

        Superdeterminismiin ilmeisesti päädytään kun oletetaan reduktionismi eli hahmotetaan maailma ja tietoisuus "alhaalta nousevaksi" sen sijaan että se voitaisiin hahmottaa "ylhäältä laskeutuvaksi".

        "Mielestäni kannattaisikin kuviteltuun *vapauteen* "ripustautumisen* sijasta keskittyä tavoitteiden kannalta oleellisten *resurssien* kuvaamiseen, eli esim. niiden, joilla voi olla oleellista vaikutusta siihen, että missä määrin kykenee tiedostamaan todellisuutta…"

        Voi sen noinkin päin hahmottaa jos välttämätä haluaa.

        (jatkuu....)


      • Anonyymi
        Anonyymi kirjoitti:

        "Yllättävää, että pidät siis tietoisuuttakin *aineena/energiana* ja tuo kohta kuulostaa mielenkiintoiselta, mm. koska jos tietoisuus on ainetta/energiaa, niin sitä voidaan tutkia ja mitata objektiivisesti, mikä tietenkin avaa hyvin mielenkiintoisia mahdollisuuksia tieteen edistämiseen."

        Valtavirtatiede tuntee vain 4 vuorovaikutusta joista vain sähkömagnetismi ja gravitaatio ilmenevät arkikokemuksessa ja joissa on periaatteessa selkeä suunta verrattuna esim. lämpöön jossa hiukkaset poukkoilevat edestakaisin satunnaisesti ilman minkäänlaista organisaatiota. Sitten olemassa kemiallisia sidoksia jotka voidaan luokitella korkeamman laatuiseksi energiaksi kuin sähkömagnetismi ja jotka siis organisoivat sähkömagneettisia kokonaisuuksia ja muodostavat kiinteitä kappaleita.

        Biologian tasolla on geenit joiden organisointikyky on selkeästi suurempi kuin "elottoman" maailman kemiallisilla sidoksilla ja joka kykenee aineenvaihduntaan jolla eliö säilyttää rakenteensa ja organisaationsa vaikka kehon atomit ja molekyylit vaihtuvat jatkuvasti.

        Jos tarkkailee taas mielen ilmiöitä niin ainakin tarkkaavaisuus ja siihen liittyvä asioiden huomaaminen ja tiedostaminen ovat selkeästi jonkinlaista energiaa kuluttavia mielen toimintoja samoin oppiminen ja oppimansa ymmärtäminen. Sitten vaikuttaa myös siltä että älylliset, tunne ja tahdonalaisen fyysisen kehon prosessit kukin vaativat omanlaisensa energiaa eli vaikka en jaksaisi opiskella "älykeskuksellani" niin silti voisin jaksaa suorittaa jostain fyysistä askaretta. Sekä äly,tunne että fyysinen puoli voi toimia joko mekaanisesti tiedostamatta tai tietoisesti jolloin siihen tietoiseen toimintaan tarvittava energia on selkeästä korkeampilaatuista kuin siihen mekaanisen tason toimintaan tarvittava energia.

        Voin myös nostaa tarkkaavaisuuteni intensiteettiä jonkinlaisella meditoinnilla jonka ainoa tarkoitus on pysäyttää sisäinen kielellinen yksinpuhelu mahdollisimman pitkäksi aikaa yhteenmenoon jolloin samalla erotamme erilaatuiset psyyken energiat toisistaan.

        Huomaamisen energiaa voidaan kutsua sensitiivienergiaksi jonka yksi perusominaisuus on samaistuminen omiin ajatuksiin, tunteisiin ja omaan kehoon. Jos onnistun tyhjentämään mieleni ja lisäämään tarkkaavaisuutta niin kykenen toimimaan ns. tietoisuuden energialla joka itsessään tuottaa objektiivisia havaintoja eli näemme maailman sellaisena kuin se on eikä sellaisena kuin sensitiivienergian tuottama samaistuminen tuottaa. Tajunnanenergia on pohjimmiltaan passiivista kun taas vielä korkeamman tason luovuuden energia kykenee tuottaa objektiivisesti uutta todellisuuden sisältöä.

        Kaikkien mahdollisten energioiden toimivuudessa pitää ottaa huomioon tarvittava energian määrä, laatu ja intensiteetti (vrt veden keittäminen kiehuvaksi). Energiat laadut varsinkin elollisessa luonnossa voivat muuttua toisikseen eli keho tekee työtä jolloin ylempi laatuinen energia hajoaa alempilaatuiseksi. Jos ruoka, juoma ja happi ylläpitävät kehon fyysistä rakenetta niin aistimukset, kokemukset ja vastaava "hengenravinto" ylläpitää kehon psyykkistä rakennetta.

        Korkeamman tasoisten organisoivien energioiden mittaaminen voi olla hankalaa koska niitä energioita ei voi ainakaan toistaiseksi mitata suoraan vain ainoastaan epäsuorasti vertaamalla siihen mitä puhtaasti satunnainen prosessi kykenisi tuottamaan samassa ajassa. Koska nykyinen tiede ei tunne kuin ne 4 perusvuorovaikutusta niin ei ole ainakaan toistaiseksi olemassa mittausvälineitä jotka kykenisivät tunnistamaan kemiallisen, biologisen tai psyykkisen tason erilaatuisia energiamuotoja eli joudumme tyytymään pelkkään kokemusperäiseen kuvailuun.

        "En hahmota, että mitä tarkoitat sillä, että tietoisuuteen liittyisi *sekä* matalampi entropia, että *vähemmän* säännönmukaisuuksia, koska entropian *puutehan* juuri tarkoittaa säännönmukaisuuksia. "

        Jos entropian hahmottaa epäjärjestyksenä ja sen epäjärjestyksen lisääntymisenä ja vähemmät säännönmukaisuudet sillä että mekaaninen reagointi vaihtuu tiedostetuksi ja harkituksi toiminnaksi niin asia lienee jotenkin ymmärrettävissä.

        "Superdeterminismiin toisaalta päädytään sitä kautta, että suljetaan pois sellaisia mahdottomia/mielettömiä vaihtoehtoja joihin esim. sisältyy implisiittisesti tyhjästä syntyviä vaikutuksia."

        Superdeterminismiin ilmeisesti päädytään kun oletetaan reduktionismi eli hahmotetaan maailma ja tietoisuus "alhaalta nousevaksi" sen sijaan että se voitaisiin hahmottaa "ylhäältä laskeutuvaksi".

        "Mielestäni kannattaisikin kuviteltuun *vapauteen* "ripustautumisen* sijasta keskittyä tavoitteiden kannalta oleellisten *resurssien* kuvaamiseen, eli esim. niiden, joilla voi olla oleellista vaikutusta siihen, että missä määrin kykenee tiedostamaan todellisuutta…"

        Voi sen noinkin päin hahmottaa jos välttämätä haluaa.

        (jatkuu....)

        "Lisäksi, koska noita vaikuttavia yksittäisiä tekijöitä on *loputtomasti*, niin tietoisuuden merkitys on aina rajallinen."

        Tuo kai riippuu lähinnä siitä mihin haluaa kulloinkin keskittyä koska kaikkeen mahdolliseen keskittyminen samanaikaisesti on sekä mahdotonta ja yleensä myös tarpeetonta. Sillä on taas hyvinkin suuri merkitys minkälaatuisella aktiviteetilla kulloinkin keskittyy.

        Tietoinen mieli kykenee vain murto-osaan siitä mihin ns. alitajunta kykenee. Yleensä kannattaa vain kerätä kaikki mahdollinen saatavilla oleva aineisto ja sitten tyhjentää mielensä tai nukkua kunhan on ensin hahmottanut sen ongelman. Usein herätessä aamulla se ratkaisu putkahtaa mieleen ikäänkuin tyhjästä mutta sille ratkaisulle pitää antaa tilaa eli ei saa olettaa liikaa etukäteen sellaista mitä ei oikeasti tiedä tai edes voi tietää (kuten esim. tuo superdeterminismi ja multiversumimalli ja kaikkien mahdollisuuksien toteutuminen:D)

        "En kuitenkaan usko siihen, että olisi *välttämättä* estettä sille, etteikö myös noita ns. "tahdon tai tietoisen itseohjautuvuuden" prosesseja voitaisi kuvata matemaattisesti, vaan mallinnukseen liittyvä ongelma on tuolta osin ehkä ainakin ensisijaisesti se, että noita prosesseja kuvaavaa *tietoa* ei ole riittävästi käytettävissä. "
        Kai se on kuvattavissa matemaattisesti mutta ei välttämättä ihan samalla tavalla kuin atomeja ja alkeishiukkasia kuvaavaa matematiikka.

        esim. topologinen ja vektoripsykologia

        https://en.wikipedia.org/wiki/Field_theory_(psychology)


        "Käsitteistö pitäisi myös saada paljon yksiselitteisemmäksi tältä osin, sillä jos tahto määritellään esim. "järjestelmän kyvyksi jatketusti suhteellisen vakioituun käyttäytymiseen tiettyjen ympäristötekijöiden vallitessa",

        Tuo määritelmä kattaa kaiken sekä puhtaasti mekaanisen että tietoisen tahdon ja oikeastaan melkein kaiken muunkin mahdollisen.

        "esim. "panssaroidulla leivänpaahtimella" on tuolloin paradoksaalisesti hyvinkin kehittynyt; tai siis jo alunperinkin "vahva", tahto. :D"

        "Pientä" viilausta siis tarvitaan vielä.... :D

        Belisario


      • Anonyymi
        Anonyymi kirjoitti:

        Nimimerkki: Jumalan-kumous (ei-rekisteröitynyt professori)

        Belisario: "Itse kannatan tällä hetkellä kolmea tai neljää hieman erilaista mallia samanaikaisesti koska ne ovat toisiaan täydenteviä 1) Tom Campbellin simulaatiomalli 2) Bernardo Kastrupin moderni idealismi 3( Gurdjieffin ja J.G. Bennettin suhteellinen materialismi 4) Joseph P. Farrellin topologinen vertauskuva"

        Gurdjieff mainittu ja "vapaalla vaihteella" siis vahvasti mennään, mutta "eksoottisia katsantokantoja" on niin monia erilaisia, että tässä on ns. "valinnan vaikeus"; tai siis ei oikeastaan ole, koska kaikkihan on determinoitua, mutta ymmärtänet, että mitä tarkoitan, eli että mitkä perusteet pakottavat valitsemaan tietyn "eksoottisen" näkökulman?

        Miten suhtaudut esim. Carlos Castaneda:n esiintuomaan maailmankuvaan ja menetelmiin? Menemme tässä "vähän" "sivuun ja yli" ketjun aiheesta ja oikeastaan "tosi reippaastikin", mutta tämä "eksoottisiin" maailmankuviin liittyvä teema on kiinnostava. CC:n esittämien menetelmien tarkoitushan on ilmeisesti sama kuin Gurdjieff:in, eli "tietoisuuden tason kehittäminen", joka toki on relevanttia, kunhan siis tuo asia määritellään jollain järkevällä tavalla; mikä tietysti voi olla haasteellista. Esim. itse olen kiinnostunut havaitsemisen edellytyksistä. CC:n esittämä maailmankuva vaikuttaa *hyvin* kattavalta ja ns. sisäisesti konsistentilta todellisuuden kuvaukselta; riippumatta siitä, että mitkä syyt/olosuhteet ovat johtaneet sen muodostukseen.

        Mielestäni se sopii yllättäen myös varsin hyvin yhteen ns. "empiirisen tieteellisen lähestymistavan" kanssa; joskin toki subjektiivisella kokemusperäisellä "eksoottisella twist":llä, mikä merkitsee sitä, että tuon aiemmin mainitsemani objektiivisen todentamisen osalta tarvittaisiin uudenlaisten menetelmien kehittämistä ja CC:n kirjoissa ei ilmeisesti myöskään esiinny mitään erityisiä matemaattisia ilmauksia; eli ilmaisutapa on sellainen, että vaikka sisältö olisikin adekvaattia, niin niiden oikea tulkintatapa voi edellyttää "konsultaatiota" monien "liittolaisten" kanssa tai jopa myös sienien käyttöä; kuten CC:n itsensä tapauksessa. CC:n menetelmien mukainen toimintatapa kuitenkin tarjoaisi ihmisille tietysti myös mm. kaipaamiasi "henkilökohtaista mukavuutta ja helppoutta karttavia virikkeitä"; lievästi ilmaisten. Lisäksi CC:n maailmankuvassa korostetaan tahdon/intention merkitystä; kuten mm. itsekin teet...

        Onkin varsin valitettavaa, että CC:n maailmankuvaan on kohdistettu niin paljon mm. "ad hominem" -tyyppistä; eli päättelyvirheisiin perustuvaa, epärelevanttia kritiikkiä mm. tiedeyhteisön taholta, eli mm. sen tutkimista, että oliko CC vai eikö hän ollut "käynyt kirjastossa" tiettynä päivänä, jolloin hän ilmoittikin olleensa jossakin muualla... Maailmankuva on; sarkastisesti ilmaisten, "tietenkin" "väärä", jos esim. tuossa suhteessa on havaittavissa "anomalioita" ja etenkin, jos ja tässä tapauksessa siis myös vahvasti kun, se ei ole ollenkaan yhteensopiva; edelleenkin, etenkin länsimaissa vallitsevan "populistisen länsimaisen ihmiskäsityksen" kanssa, vaikka ja siis myös kun se sisältää *mm.* käsityksen "reaalisesti vaikuttavasta bilokaatiosta"... :D
        https://en.wikipedia.org/wiki/Bilocation

        "Miten suhtaudut esim. Carlos Castaneda:n esiintuomaan maailmankuvaan ja menetelmiin? "

        Itse asiassa Castaneda osuu asian ytimeen paljon paremmin kuin Gurdjieff tai Bennett ja monissa Gurdjieffin seuraajissa on havaittavissa aika paljon pedanttista "oikea-oppisuuden" vaalimista ja vähän kökköilyn puolelle menevää kaavamaista käytöstä eli esim. Gurdjieffin elämästä kertova "Meetings with remarkable men" elokuva on paikoitellen jopa tahallisen koomisen vaikutelman tuottava.

        Mielestäni G. oli myös väärässä aika monessa asiassa mutta hän oli ainakin alkuvaiheessa 1900-luvun alussa jopa melkein länsimaisen tieteen perinteen mukainen kokeellinen tietoisuuden tutkija ja monet hänen kokeiluistaan olivat ehkä ainakin jossain määrin epäonnistuneita.

        G itse oli varsin värikäs ja huumorintajuinen tyyppi ja keskittyi elämänsä myöhemmässä vaiheessa epäsuoraan zen-tyyppiseen metodiin (esim. maljoja idiooteille) ja aika epätavalliseen kirjoittamiseen (esim. Beelzebubs Tales) koska huomasi oppilaidensa omaksuvan opetukset liian mekaanisesti ja kaavamaisesti eli "koira piti haudata syvemmälle" :D

        https://www.satrakshita.com/the_science_of_idiotism.htm

        "Esim. itse olen kiinnostunut havaitsemisen edellytyksistä. CC:n esittämä maailmankuva vaikuttaa *hyvin* kattavalta ja ns. sisäisesti konsistentilta todellisuuden kuvaukselta; riippumatta siitä, että mitkä syyt/olosuhteet ovat johtaneet sen muodostukseen. "
        Näin on ja tässä on aika epäolennaista oliko "intiaaninoita" Don Juan kumppaneineen oikeasti olemassa. Tuolta löytyy olennaisimmat Castanedaan liityvät ideat:

        https://www.prismagems.com/castaneda/djintro.html

        Itse tutustuin Castanedan kirjoihin jo 1970-luvun lopulla ja ainakin minulle niistä oli suuri apu "oikeanlaisen" tietoisuudentilan ja elämänasenteen saavuttamisessa. Nimimerkkini on lainattu yhdestä Castanedan kirjasta jota pidän ehkä parhaana ja ne kannattaa lukea kaikki jos vähänkin kiinnostaa. Kyse ei ole tietenkään millään tavalla tieteellisestä tekstistä kun taas esim. Bennettin kirjoituksia voi pitää ainakin akateemiseen filosofian kanssa vertailukelpoisina tuotteina ellei nyt varsinaisesti tieteellisinä julkaisuina.

        "CC:n kirjoissa ei ilmeisesti myöskään esiinny mitään erityisiä matemaattisia ilmauksia; eli ilmaisutapa on sellainen, että vaikka sisältö olisikin adekvaattia, niin niiden oikea tulkintatapa voi edellyttää "konsultaatiota" monien "liittolaisten" kanssa tai jopa myös sienien käyttöä; kuten CC:n itsensä tapauksessa. "

        Niin ilmeisesti monet varsinkin länsimaiset tarvitsevat jonkinlaista psykoaktiivista kemiaa voidakseen irtautua kulttuuristaan. Itse jostain syystä olen kai aina ollut sen verran syntymähumalassa että en tarvitse lisäaineita siihen että menee ajattelun tasolla melko "lujaa" aika usein. :D

        "CC:n menetelmien mukainen toimintatapa kuitenkin tarjoaisi ihmisille tietysti myös mm. kaipaamiasi "henkilökohtaista mukavuutta ja helppoutta karttavia virikkeitä"; lievästi ilmaisten."

        Heh joo. Eräässä kirjassa Belisario (valepukuinen naguaali) höykytti Don Juania oikein urakalla ja Don Juan laittoi sitten "vahingon kiertämään" CC:n tapauksessa.

        "siis myös kun se sisältää *mm.* käsityksen "reaalisesti vaikuttavasta bilokaatiosta"... :D"

        Idealistisissa malleissa tuo on tietysti ihan mahdollista niin kuin kaikki muutkin ns. paranormaalit jutut kuten kaukonäkeminen, levitointi ja muut makropsykokinesiat :D

        Tajunnanvirta lähti näköjään aika vuolaasti virtaamaan matematiikan monenlaisuudesta....

        Belisario


      • Anonyymi
        Järkisyitä kirjoitti:

        ”Sanotaan nyt vaikka niin että minusta näyttää että materialisti hahmottaa kokemaansa kielellisten käsitteiden avulla erilaisiin "laatikoihin" joista yhden laatikon kyljessä lukee "materia" ja toisen laatikon kyljessä lukee "henki" ja sinne materia-lootaan lajitellaan kaikki se kokemus mikä tuntuu konkreettiselta ja todelliselta ja sinne henki-lootaan taas kaikki se kokemus mikä tuntuu jotenkin epätodelliselta, haamumaiselta, abstraktilta yms.”

        En tunnista tuosta kuvauksesta ollenkaan omaa ajatustapaani. Kuulostaa enemmänkin dualistiselta käsitykseltä.

        Materialistille kaikki koostuu materiasta, mutta joudumme käsittelemään asioita eri abstraktio-tasoilla. Esimerkiksi sinä koostut vain erillisistä atomeista, mutta tästä huolimatta ajattelemme sinua kokonaisena ihmisenä, joka on tuolle atomijoukolle annettu nimike tai oikeammin korkeamman tason malli: abstraktio. Emme juuri koskaan ajattele asioita atomitasolla, koska ymmärryksemme ei riitä. Aivan samoin emme voi ajatella kokemusmaailmaa neuroverkkona vaan joudumme käsittelemään sitä erilaisten abstraktioiden kautta. Kokemusmaailmaa käsittelemme omista kokemuksistamme käsin, vaikka uskommekin senkin olevan neuroverkon tuotos.

        "Materialistille kaikki koostuu materiasta, mutta joudumme käsittelemään asioita eri abstraktio-tasoilla. "

        Niin mutta ne abstraktiotasot eivät ole ainetta vaan lähinnä sen aineellisen tason kartta ja karttaa ei pidä sotkea maastoon. Itse asiassa nuo abstraktiotasot tuottavat sen dualismin suhteessa aineeseen useimmilla materialisteilla jotka luulevat olevansa monisteja eli yhden substanssin kannattajia.

        "Aivan samoin emme voi ajatella kokemusmaailmaa neuroverkkona vaan joudumme käsittelemään sitä erilaisten abstraktioiden kautta. Kokemusmaailmaa käsittelemme omista kokemuksistamme käsin, vaikka uskommekin senkin olevan neuroverkon tuotos."

        Neuroverkko suhteessa tietoisuuteen on pelkkä analogia ja on hyvin vaikea ymmärtää miten se neuroverkko sellaisenaan voisi tuottaa kokemuksellisen tietoisuuden eli kvalian.
        Minusta nuo neuverkot vaikuttavat lähinnä asssosiatiiivisilta tietokannoilta mikä voi muistuttaa sitä miten ihmisen muisti toimii mutta kaiken tietoisuuden selittäjäksi siitä neuroverkkomallista tuskin on.

        ....

        Tuli tässä aamulla mieleen aiheeeseen liittyen sellainenkin asia että intuitiolla on 2 eri merkitystä joista toista käytetään kun sanotaan esim. että kvanttifysiikan ilmiöt ovat intuition vastaisia jolla tarkoitetaan lähinnä sitä että se on ajatustottumustemme vastaista.

        Toinen intuition merkitys muistuttaa oivallusta ja inspiraatiota joka tulee syvältä tajunnasta kun se tajunnan sisäinen monologi keskeytetään tarpeeksi pitkäksi aikaa ja sille oivallukselle tehdään ensin riittävästi tilaa tietoisen mielen tasolla.

        Belisario


      • Anonyymi kirjoitti:

        "Materialistille kaikki koostuu materiasta, mutta joudumme käsittelemään asioita eri abstraktio-tasoilla. "

        Niin mutta ne abstraktiotasot eivät ole ainetta vaan lähinnä sen aineellisen tason kartta ja karttaa ei pidä sotkea maastoon. Itse asiassa nuo abstraktiotasot tuottavat sen dualismin suhteessa aineeseen useimmilla materialisteilla jotka luulevat olevansa monisteja eli yhden substanssin kannattajia.

        "Aivan samoin emme voi ajatella kokemusmaailmaa neuroverkkona vaan joudumme käsittelemään sitä erilaisten abstraktioiden kautta. Kokemusmaailmaa käsittelemme omista kokemuksistamme käsin, vaikka uskommekin senkin olevan neuroverkon tuotos."

        Neuroverkko suhteessa tietoisuuteen on pelkkä analogia ja on hyvin vaikea ymmärtää miten se neuroverkko sellaisenaan voisi tuottaa kokemuksellisen tietoisuuden eli kvalian.
        Minusta nuo neuverkot vaikuttavat lähinnä asssosiatiiivisilta tietokannoilta mikä voi muistuttaa sitä miten ihmisen muisti toimii mutta kaiken tietoisuuden selittäjäksi siitä neuroverkkomallista tuskin on.

        ....

        Tuli tässä aamulla mieleen aiheeeseen liittyen sellainenkin asia että intuitiolla on 2 eri merkitystä joista toista käytetään kun sanotaan esim. että kvanttifysiikan ilmiöt ovat intuition vastaisia jolla tarkoitetaan lähinnä sitä että se on ajatustottumustemme vastaista.

        Toinen intuition merkitys muistuttaa oivallusta ja inspiraatiota joka tulee syvältä tajunnasta kun se tajunnan sisäinen monologi keskeytetään tarpeeksi pitkäksi aikaa ja sille oivallukselle tehdään ensin riittävästi tilaa tietoisen mielen tasolla.

        Belisario

        Niin nykyinen ymmärryksemme neuroverkoista selittää ”vain” ihmisen autonomisen oppimisen: hahmon- ja puheentunnistuksen ja motoriikan. Se ei anna mitään selitystä tietoisuuden eli kvalia-kokemuksille. Neurologit ovat havainneet, että ajatellesamme kuvia tai ääntä samat neuroverkot aktivoituvat kuin millä analysoimme niitä. Näyttää siis, että neuroverkkomme mallintavat todellisuutta.

        Monimutkaisetkin rutiini-toimet tapahtuvat usein autonomisesti ilman tiedostamista. Tietoinen kokemus vaikuttaisi syntyvän vain silloin tällöin, kun jokin sitä vaatii. Dualistilta voisi kysyä, että olemmeko siis vain sieluttomia neuroverkon ohjaamia zombeja suurimman osan aikaa ja sielu vain välillä pistäytyy kokemassa jotakin.


      • Anonyymi kirjoitti:

        "Materialistille kaikki koostuu materiasta, mutta joudumme käsittelemään asioita eri abstraktio-tasoilla. "

        Niin mutta ne abstraktiotasot eivät ole ainetta vaan lähinnä sen aineellisen tason kartta ja karttaa ei pidä sotkea maastoon. Itse asiassa nuo abstraktiotasot tuottavat sen dualismin suhteessa aineeseen useimmilla materialisteilla jotka luulevat olevansa monisteja eli yhden substanssin kannattajia.

        "Aivan samoin emme voi ajatella kokemusmaailmaa neuroverkkona vaan joudumme käsittelemään sitä erilaisten abstraktioiden kautta. Kokemusmaailmaa käsittelemme omista kokemuksistamme käsin, vaikka uskommekin senkin olevan neuroverkon tuotos."

        Neuroverkko suhteessa tietoisuuteen on pelkkä analogia ja on hyvin vaikea ymmärtää miten se neuroverkko sellaisenaan voisi tuottaa kokemuksellisen tietoisuuden eli kvalian.
        Minusta nuo neuverkot vaikuttavat lähinnä asssosiatiiivisilta tietokannoilta mikä voi muistuttaa sitä miten ihmisen muisti toimii mutta kaiken tietoisuuden selittäjäksi siitä neuroverkkomallista tuskin on.

        ....

        Tuli tässä aamulla mieleen aiheeeseen liittyen sellainenkin asia että intuitiolla on 2 eri merkitystä joista toista käytetään kun sanotaan esim. että kvanttifysiikan ilmiöt ovat intuition vastaisia jolla tarkoitetaan lähinnä sitä että se on ajatustottumustemme vastaista.

        Toinen intuition merkitys muistuttaa oivallusta ja inspiraatiota joka tulee syvältä tajunnasta kun se tajunnan sisäinen monologi keskeytetään tarpeeksi pitkäksi aikaa ja sille oivallukselle tehdään ensin riittävästi tilaa tietoisen mielen tasolla.

        Belisario

        En näe noiden kahden intuitio esimerkin välillä kovin suurta eroa. Jaan päättelyn intuitiiviseen ja analyyttiseen. Käytän shakissa molempia.

        Analyyttinen ajattelu on vaihtoehtojen täsmällistä loogista pohtimista ja laskemista. Tätä voidaan soveltaa taktisesti, mutta myös strategisesti ymmärtäen, mitkä elementit tekevät jostakin asetelmasta paremman. Se tuottaa aina (tarkastusten jälkeen) oikean tuloksen.

        Intuitiivinen ajattelu on nopeaa ”sinne päin” ajattelua, missä käytämme kokemustamme ilman, että taustalla on syvää analyysiä. Jokin siirto voi esimerkiksi vaikuttaa intuitiivisesti hyvältä, vaikka osoittautuu virheeksi. Intuitiivinen ajattelu on kuitenkin tarpeellista, koska:
        1. Intuitiivinen ajattelu on paljon nopeampaa. Jos olet ajamassa kolarin, niin ei sinulla ole aikaa pohtia kaikkia vaihtoehtoja analyyttisesti.
        2. Käytännön ongelmat ovat usein niin monimutkaisia, ettei meillä ole kykyä niiden syvälliseen analyyttiseen tarkasteluun. Intuitio voittaa usein pinnallisen liian yksinkertaistetun analyyttisen mallin.

        Todellisuudessa käytämme usein intuitiivisen ja analyyttisen ajattelun sekotelmaa. Toisinaan olen ollut tilanteessa, missä intuitioni on analyyttisen ajatteluni vastainen. Näissä minulla on taipumus mennä kovan logiikan eli analyysin kautta, mutta iän kanssa olen oppinut, että intuitioni osuu useammin oikeanpaan (vaikka menee joskus täysin metsään).


      • Anonyymi
        Järkisyitä kirjoitti:

        Niin nykyinen ymmärryksemme neuroverkoista selittää ”vain” ihmisen autonomisen oppimisen: hahmon- ja puheentunnistuksen ja motoriikan. Se ei anna mitään selitystä tietoisuuden eli kvalia-kokemuksille. Neurologit ovat havainneet, että ajatellesamme kuvia tai ääntä samat neuroverkot aktivoituvat kuin millä analysoimme niitä. Näyttää siis, että neuroverkkomme mallintavat todellisuutta.

        Monimutkaisetkin rutiini-toimet tapahtuvat usein autonomisesti ilman tiedostamista. Tietoinen kokemus vaikuttaisi syntyvän vain silloin tällöin, kun jokin sitä vaatii. Dualistilta voisi kysyä, että olemmeko siis vain sieluttomia neuroverkon ohjaamia zombeja suurimman osan aikaa ja sielu vain välillä pistäytyy kokemassa jotakin.

        "Niin nykyinen ymmärryksemme neuroverkoista selittää ”vain” ihmisen autonomisen oppimisen"

        Niin se selittää mekaanisen oppimisen mutta ei sen opitun ymmärtämistä eikä tarkoituksenmukaista soveltamista mikä taas mielestäni toimii parhaiten analogioiden ja vertauskuvien avulla eikä pelkästään deduktion tai induktion kautta. Analoginen ajattelu mahdollistaa aihepiiristä ja kontekstista toiseen siirtymisen luontevasti.

        "Monimutkaisetkin rutiini-toimet tapahtuvat usein autonomisesti ilman tiedostamista."

        Kyllä koska ne ovat rutiinitoimia eikä niihin kannata tuhlata arvokasta tiedostamisen energiaa.

        "Dualistilta voisi kysyä, että olemmeko siis vain sieluttomia neuroverkon ohjaamia zombeja suurimman osan aikaa ja sielu vain välillä pistäytyy kokemassa jotakin."

        Kyllä koska "automaattienergia" on kustannustehokkaampaa käyttää kuin korkeampaan organisointikykyyn kykenevät muut energianlaadut. Kaikkein olennaisinta on kuitenkin huomata ajoissa milloin ja minkälaisissa tilanteissa sitä "sielua" tarvitaan. Tarpeellista ja hyödylllistä olisi myös luopua turhista rutiineista ja tavoista ja näin säästää ja kartuttaa korkeammanlaatuisia energioita jolloin sitä olisi saatavilla silloin kun sitä oikeasti kaivataan ja tarvitaan.

        Belisario


    • Anonyymi

      Itse näen matematiikan hieman erilaisena kuin aloittaja. Mielestäni matematikka voidaan jakaa perinteiseen matematiikkaan, puhtaaseen eli abstraktiin matematiikkaan ja soveltavaan matematiikkaan.

      Perinteisellä matematiikalla itse ymmärrän kaikkea matematiikkaa 1800-luvun loppupuolelle saakka, ja tuo matematiikka oli pääosin syntynyt luonnonilmiöiden, eritoten fysiikan ilmiöiden, pohdinnoista. 1800-luvun loppupuollella matematiikka alkoi jakaantua abstraktiin matematiikkaa ja soveltavaan matematiikkaa. On luonnollisesti vaikea täsmällisesti vetää rajaa abstraktin ja soveltavan matematiikan kanssa. Esim. todennäköisyyslaskenta on varmaankin molempia.

      Soveltava matematiikka on nykyään valtavan suurin kokonaisuus, esim. lähes kaikki huipputekniikka (kuten tekoäly, koneoppiminen, hahmontunnistus, kontrollitekniikka, robotiikka, informaatiotekniikka, jne.) on pohjimmiltaan soveltavaa matematiikkaa.

      Matematikkaan tulee suhtautua mielestäni varauksella. Matemaattisista todistuksista voidaan olla filosofisesti montaa mieltä. Olisi parempaa kielenkäyttöä sanoa, että matemaattisesti voidaan osoittaa tai johtaa, kuin käyttää niin voimakasta kieltä kuin todistaminen.

      Matematiikka kokonaisuudessaan voitaisiin tietysti kirjoittaa myös toisella tavalla. Esim. soveltavassa matematiikassa huomaamme toistuvasti, että 1 euro plus 1 euro ei tarkasti ottaen ole kaksi euroa (saman asian ilmaiseen myös joukko-oppi).

      Matematiikka kokonaisuutena ei ole ristiriidaton kokonaisuus. Esim. todennäköisyyslaskennassa on ikuisuuskiista, pitää todenäköisuudet analysoida deduktiivisesti vai induktiivesti - vai jotain siltä väliltä.

      Lähes kaikki modernissa yhteiskunnassa perustuu viime kädessä matematiikkaan, mutta en usko ajatukseen, että matematiikan totuus on ehdotonta. Asioita voi tarkastella monesta näkökulmasta.

    • Matematiikka on sisäisesti koherentti oppijärjestelmä. Mutta matematiikka saattaa sisältää sellaista jolla ei ole mitään vastinetta reaalitodellisuudessa.

    • "Jotkut filosofisesti orientoituneet ovat sitä mieltä., että on monenlaista matematiikkaa"

      Juu, on aritmetiikkaa, algebraa ja geometriaa.

    Ketjusta on poistettu 0 sääntöjenvastaista viestiä.

    Luetuimmat keskustelut

    1. 197
      2742
    2. Euroviisut fiasko, Suomen kautta aikain typerin esitys, jumbosija odottaa. Olisi pitänyt boikotoida!

      Tämän vuoden euroviisut on monella tapaa täydellinen fiasko. Ensinnäkin kaikkien itseään kunnioittavien eurooppalaisten
      Maailman menoa
      241
      2668
    3. Hei A, osaatko

      sanoa, miksi olet ihan yhtäkkiä ilmestynyt kaveriehdotuksiini Facebookissa? Mitähän kaikkea Facebook tietää mitä minä en
      Ikävä
      65
      2366
    4. Synnittömänä syntyminen

      Helluntailaisperäisillä lahkoilla on Raamatunvastainen harhausko että ihminen syntyy synnittömänä.
      Helluntailaisuus
      201
      1857
    5. Mitä tämä tarkoittaa,

      että näkyy vain viimevuotisia? Kirjoitin muutama tunti sitten viestin, onko se häipynyt avaruuteen?
      Ikävä
      41
      1364
    6. Tuollainen kommentti sitten purjehduspalstalla

      "Naisen pillu se vasta Bermudan kolmio on. Sinne kun lähdet soutelemaan niin kohta katoaa sekä elämänilo että rahat"
      Suhteet
      12
      1328
    7. Nukkumisiin sitten

      Käsittelen asiaa tavallani ja toiveissa on vielä että tästä pääsee hyppäämään ylitse. Kaikenlaisia tunteita on läpikäyny
      Ikävä
      4
      1307
    8. Syö kohtuudella niin et liho.

      Syömällä aina kohtuudella voi jopa laihtua.On paljon laihoja jotka ei harrasta yhtään liikuntaa. Laihuuden salaisuus on
      Laihdutus
      22
      1266
    9. Muistatko komeroinnin?

      Taannoin joskus kirjoitin aloituksen tänne komeroinnista eli hikikomoreista; syrjäytyneistä nuorista ihmisistä. Ehkä asu
      Suhteet
      48
      1234
    10. Nesteen bensapumput pois, tilalle latausasemat

      Näin se maailma muuttuu, kun Suomessakin liikenneasemat lopettavat polttoaineiden myynnin ja tarjoavat enää sähköä autoi
      Maailman menoa
      158
      1132
    Aihe